TOL Patent Bar Outline

132
Patent Bar Outline © theOtherLives * Important to know for the patent bar. 1 CHAPTER 100: SECRECY, ACCESS, NATIONAL SECURITY AND FOREIGN FILING 101: General Information NOT Available to the Public : Generally, no information concerning pending or abandoned applications may be made available to the public without authorization of the applicant, the assignee of record, or the agent or attorney of record. o Exceptions: Applications that have been published Reissue applications Reexamination proceedings Implication of Timing on Availability of Certain Information : If the publication or issue date is later than the current date, information about the publication date (or number) or the issue date may ONLY be given to the applicant, the assignee of record, or agent or attorney of record. 102: Information as to Status of an Application Status information of an application includes: o Whether the application is pending, abandoned or patented; o Whether the application has been published; o The application number or the serial number plus any one of: The filing date of the national application, The international filing date or The date of entry into the national stage; and o Whether another application claims the benefit of the application. A requester of status information may check the Patent Application Information Retrieval (PAIR) system on the USPTO website or contact the File Information Unit (FIU). If the application has not been published and is pending or abandoned (including provisional applications), the FIU will give status information only to the following ‘entitled’ individuals 1 : o The inventor; o An attorney or agent of record in the application; o An assignee of record in the application; or o A person with written authority from a, b, or c. If the requestor is not one of the four types of individual listed above, status information may still be granted if: o The application is identified by application number (or serial number and filing date) in a published patent document; or o It is an application claiming the benefit of the filing date of an application identified by application number (or serial number and filing date) in a published patent document. However, other information such as whether the application is a Continuation-in-Part (CIP), continuation or divisional application, the date of abandonment of the application or the issue date should only be given to one of the ‘entitled’ individuals identified above. Even if the requestor is not an ‘entitled’ individual but the application is in the national stage or any application claiming the filing date of a published international application where the US is a designated state, the requestor may receive status information for the national stage application, as well as any application claiming the benefit of the filing date of the published international application. The best place to start when seeking status information is Patent Application Information Retrieval (PAIR) system on the USPTO website. 103: Right of Public to Inspect Patent Files and Some Application Files (37 § CFR 1.11, 1.14) Since June 30, 2003, all new applications are stored as official records in an electronic system as an Image File Wrapper (IFW). If a patent application has been published, all documents relating to the file of the application (whether abandoned or pending) are open to the public upon request. If the published application is a redacted copy, then access will be limited to the redacted version. (37 CFR § 1.14; Patent Applications Preserved in Confidence) The following unpublished abandoned applications are available to the public: o An abandoned application referred to in a US patent application publication or a US patent; o A pending File Wrapper Continuation application filed under former 37 CFR § 1.62 (File Wrapper Continuing Procedure) of an abandoned application that meets the requirements of 37 CFR § 1.14. 1 ‘Entitled’ individuals is not a term used by the PTO, but rather it is used in this outline to identify the group: inventor, attorney or agent in the application, assignee of record, or a person granted permission from one of these three individuals.

Transcript of TOL Patent Bar Outline

Page 1: TOL Patent Bar Outline

Patent Bar Outline © theOtherLives

* Important to know for the patent bar. 1

CHAPTER 100: SECRECY, ACCESS, NATIONAL SECURITY AND FOREIGN FILING 101: General

Information NOT Available to the Public: Generally, no information concerning pending or abandoned applications may be made available to the public without authorization of the applicant, the assignee of record, or the agent or attorney of record.

o Exceptions: • Applications that have been published • Reissue applications • Reexamination proceedings

Implication of Timing on Availability of Certain Information: If the publication or issue date is later than the current date, information about the publication date (or number) or the issue date may ONLY be given to the applicant, the assignee of record, or agent or attorney of record.

102: Information as to Status of an Application

Status information of an application includes: o Whether the application is pending, abandoned or patented; o Whether the application has been published; o The application number or the serial number plus any one of:

• The filing date of the national application, • The international filing date or • The date of entry into the national stage; and

o Whether another application claims the benefit of the application. A requester of status information may check the Patent Application Information Retrieval (PAIR) system

on the USPTO website or contact the File Information Unit (FIU). If the application has not been published and is pending or abandoned (including provisional

applications), the FIU will give status information only to the following ‘entitled’ individuals1: o The inventor; o An attorney or agent of record in the application; o An assignee of record in the application; or o A person with written authority from a, b, or c.

If the requestor is not one of the four types of individual listed above, status information may still be granted if:

o The application is identified by application number (or serial number and filing date) in a published patent document; or

o It is an application claiming the benefit of the filing date of an application identified by application number (or serial number and filing date) in a published patent document.

However, other information such as whether the application is a Continuation-in-Part (CIP), continuation or divisional application, the date of abandonment of the application or the issue date should only be given to one of the ‘entitled’ individuals identified above.

Even if the requestor is not an ‘entitled’ individual but the application is in the national stage or any application claiming the filing date of a published international application where the US is a designated state, the requestor may receive status information for the national stage application, as well as any application claiming the benefit of the filing date of the published international application.

The best place to start when seeking status information is Patent Application Information Retrieval (PAIR) system on the USPTO website.

103: Right of Public to Inspect Patent Files and Some Application Files (37 § CFR 1.11, 1.14)

Since June 30, 2003, all new applications are stored as official records in an electronic system as an Image File Wrapper (IFW).

If a patent application has been published, all documents relating to the file of the application (whether abandoned or pending) are open to the public upon request. If the published application is a redacted copy, then access will be limited to the redacted version. (37 CFR § 1.14; Patent Applications Preserved in Confidence)

The following unpublished abandoned applications are available to the public: o An abandoned application referred to in a US patent application publication or a US patent; o A pending File Wrapper Continuation application filed under former 37 CFR § 1.62 (File Wrapper

Continuing Procedure) of an abandoned application that meets the requirements of 37 CFR § 1.14.

1 ‘Entitled’ individuals is not a term used by the PTO, but rather it is used in this outline to identify the group: inventor, attorney or agent in the application, assignee of record, or a person granted permission from one of these three individuals.

Page 2: TOL Patent Bar Outline

Patent Bar Outline © theOtherLives

* Important to know for the patent bar. 2

Where only part of an application is incorporated by reference, a non-‘entitled’ requestor must petition for access to or a copy of the incorporated material.

Any party may Petition for Access to a patent application. o Filing requirements for each application to which access is desired:

• Petition for Access – File under 37 CFR § 1.14(h), showing why access is desired and why petitioner believes s/he is entitled to access.

• Proof of Service – Proof that such petition has also been served to the applicant (or assignee or attorney/agent of record) or a duplicate of the petition that the PTO can forward to the applicant, and

• Filing fee Provisional applications are generally only available to a member of the public who has received

permission from an entitled person. However, like other applications, provisionals may be available through public PAIR if the application is relied on for priority in a US patent.

All reissue documents are open to public inspection under 37 CFR § 1.11(b) (Files Open to the Public). o The filing of the reissue application will be public in the Official Gazette.

Documents associated with Requests for Reexamination are made available to the public once they are scanned into the reexamination database.

o Completed requests for reexamination are published in the Official Gazette. The Board of Patent Appeals and Interferences (BPAI) handles all requests for application documents

involved in interferences. 106: Control of Inspection by Assignee (entire vs. partial interest)

An assignee of record of entire interest may appoint an attorney or agent of choice to prosecute the application (37 CFR § 3.71; Prosecution by Assignee).

o Further, an assignee may request that an inventor be denied access to the application, and such a request should be filed separately under 37 CFR § 1.14(c) and should be directed to the Office of Petitions2.

An assignee of record of partial interest does not have intervening rights in the prosecution of the application to the exclusion of the inventor, however s/he does have a right to inspect the application.

110: Confidential Nature of International Application

International applications are generally published 18 months after filing. However, publication of the application does not open up public inspection of the Home Copy or Search Copy (see MPEP 1800) except where the US has been designated and:

o The US served as the Receiving Office (for the Home Copy); or o The US served as the International Searching Authority (for the Search Copy).

SUMMARY OF THE SECRECY OF APPLICATIONS

The following files may be available to the public: Unpublished abandoned patents may be viewed by: •Issued patents or published Statutory Invention Registration (SIRs), pending or abandoned applications • All reissue applications • All requests for reexamination or papers related to reexamination proceedings • Files of any interference • Unpublished abandoned applications that are identified or relied upon • Unpublished pending application whose benefit is claimed Unpublished pending apps that are incorporated by reference or otherwise identified

• Absolutely NO members of the public, these patents have confidential status • Any of the inventors except in cases where the assignee has requested otherwise • Any inventor who was names, even if they did not sign the application • Any assignee (either partial or entire interest) • A licensee of entire interest (not partial interest) • An attorney or agent of record, or anyone given written authorization from an inventor

Only inventors (whether listed in the declaration or not) of a pending app. or those with the serial number and filing date of

an app. in the national stage may access:

A person without written authority may petition for access to any pending or abandoned app by filing a petition (with

fee) that includes: •Whether an application is pending, abandoned or patented • The application number and filing date • Whether one or more applicants claim the benefit of the filing date • Whether the application claims the benefit of a prior application

•Reasons why access is desired • Reasons why he or she believes they are entitled access (“special circumstances” must be present)

o If the owner is using the patent app to interfere with a competitor’s business o When a patent relies upon the application for priority o If the application is referred to in an issued patent

• A notice that a copy of the petition was given to the applicant/owner either directly or indirectly through the patent office (“servicing the applicant”)

2 Note that all issues of appeal go the BPAI and all procedural issues go to the Office/Director of Petitions. This distinction is likely to be tested on the Patent Bar.

Page 3: TOL Patent Bar Outline

Patent Bar Outline © theOtherLives

* Important to know for the patent bar. 3

120: Secrecy Orders

Secrecy orders are placed on applications that contain information sensitive to national security. If a secrecy order is place on an international application, the application will not be forwarded to the

International Bureau (see MPEP 1800) until the secrecy order is lifted. Secrecy orders are valid for one-year terms, however, they may be renewed as many times as

necessary. 140: Foreign Filing Licenses

Once an application has been filed in the US, an applicant may not file an application for the same invention in a foreign country within six months without obtaining a Foreign Filing License.

If an applicant files in a foreign country after the US filing but before the expiration of 6 months through error and without deceptive intent, s/he must file for a retroactive foreign filing license.

Petition for a retroactive license must include: o A list of all foreign countries the application was filed in; o The dates the application was filed in each country; o An oath indicating:

• That the subject matter is not under a secrecy order; • Evidence that the license has been diligently sought; and • Evidence that the illegal foreign filing occurred by mistake and without deceptive intent

o A penalty fee. A foreign filing license will not be granted if the invention was made outside of the US, even if the inventor

is American. Summary of Foreign Filing Requirements:

150: Statements to DOE and NASA

No patents for nuclear material or atomic energy will be issued except under special circumstances.

Page 4: TOL Patent Bar Outline

Patent Bar Outline © theOtherLives

* Important to know for the patent bar. 4

Page 5: TOL Patent Bar Outline

Patent Bar Outline © theOtherLives

* Important to know for the patent bar. 5

CHAPTER 200: TYPES, CROSS-NOTING, AND STATUS OF APPLICATION 201: Types of Applications

National v. International Applications o A national application is an application that has either been 1) filed with the USPTO under 35

USC § 111, or 2) filed as an international application that has entered the national stage. The filing date is the date of the US filing (either direct or through entrance to national stage).

o An international application is filed under Patent Cooperation Treaty (PCT) and has not yet entered national processing at the Designated Office Stage (prior to being processed by the US). Once it is processed by the USPTO, the application enters the National Stage.

National Application National Stage Application

Filed with the USPTO under 35 USC § 111

Filed as an international app. and in later processed by the US

Restriction practice applies (MPEP 800) Unity of Invention applies (MPEP 1800)

The filing date may be awarded without the fee or an oath or declaration

The filing date will not be awarded until the fee is paid and the oath or declaration is submitted

Correction of Inventorship in an Application

o 37 CFR § 1.48 may be used to correct inventorship under certain circumstances. Use the following filings to correct the associated errors:

• 1.48(a): May be used when an innocent error occurred and inventorship was wrong from the beginning in an oath or declaration in a non-provisional application.

• 1.48(b): May be used for a correction in the inventorship necessary during the prosecution (usually due to cancellation or amending of claims) of a non-provisional application and one or more inventors must now be deleted.

• 1.48(c): May be used for amending the claims during the prosecution of a non-provisional application if it results in the addition of an inventor.

• 1.48(d): May be used where an inventor needs to be added to a provisional. • 1.48(e): May be used where an inventor needs to be deleted from a provisional. • 1.48(f): Where an oath/declaration or cover sheet was not submitted, inventorship is

corrected when the appropriate form is submitted (unless the inventor has declared himself as the sole inventor).

o Alternatively, inventorship can be changed by filing a continuing application under 37 CFR § 1.53 (Application Number, Filing Date, and Completion of Application).

o Changing inventorship of an issued patent can be done pursuant to § 1.48(a) and § 1.48(b). Parent Application

o The term “parent” applies to an earlier filed application of which a later application (“offspring” application) claims the benefit under 35 USC § 120.

o A provisional application is never referred to as a “parent” application. Original Application

o An “original” application is one that is not a reissue; an “original” application may be an offspring application.

Non-provisional (Regular) Application o Non-provisionals are filed under 35 USC § 111(a). o The filing date is the date that the minimum required filings are received by the PTO. o The documents required for a complete non-provisional application include:

1. Specification; 2. Claims; 3. Drawings (if necessary); 4. Oath/declaration (not required to obtain filing date); and 5. Filing fee (not required to obtain filing date).

o They may claim the benefit of a provisional or another non-provisional. Provisional Application

o Provisionals are filed under 35 USC § 111(b) and 37 CFR § 1.53(c). o An applicant must specify clearly that the application is a provisional or it will be treated as a regular

application. o A provisional can be filed for a utility patent only.

Page 6: TOL Patent Bar Outline

Patent Bar Outline © theOtherLives

* Important to know for the patent bar. 6

o The required filings are provisional applications include: 1. Specification; 2. Drawings (if required); 3. Cover sheet; and 4. Filing fee.

o Provisionals are not entitled to the benefit of earlier filed applications. o Provisionals are not examined by the PTO, and they expire automatically after 12 months. o A Notice of Missing Parts will be sent out when: 1) The cover sheet is missing; 2) The cover sheet

is incomplete; or 3) The provisional filing fee is not paid. • If “missing parts” are not corrected within 2 months, the application will go abandoned.

o An English translation is not required for provisional applications filed in another language (because they are not reviewed by the PTO).

Converting a Provisional to a Non-provisional o The conversion must occur within 12 months, prior to the expiration of the provisional. o A request to convert must be accompanied by:

1. Conversion fee, and 2. An amendment with at least one valid claim.

o A converted application will have the filing date of the provisional application. o Since the patent term is based on the filing date of the provisional when an application is converted

to non-provisional, it is often wiser to file the non-provisional and claim the benefit of the provisional*. This way the patent term begins on the filing date of the non-provisional.

Converting a Non-provisional to a Provisional o The non-provisional must still be pending and not more than 12 months from date of filing. o To convert a non-provisional to a provisional, an applicant must submit:

1. A petition requesting the conversion, and 2. A fee.

Reissue Application* o A reissue application takes the place of an unexpired patent that is defective in one or more areas.

If issued, the reissue patent will expire when the original patent term ends. o A reissue is generally requested by the patent owner. o A reissue may be filed anytime during the lifetime of the patent. o A reissue filed within a maximum of 2 years may be use to broaden claims. However, any claims

that were canceled or any matter removed from the original application in order to gain acceptance cannot be recaptured in a reissue.

o All reissue applications for a single patent must issue simultaneously. Divisional Application

o If an application actually claims more than one invention, an applicant may have to remove the claims describing the other invention(s). The applicant may then put these removed claims into a separate, divisional application, which will have the same filing date as the original (parent) application.

o The divisional application may only contain subject matter included in the original application. o There must be one common inventor between original and divisional applications. o The original will not become statutory bar, as prior art, against other claims until one year from the

issuance of the original patent. o Divisionals may be filed under 37 CFR § 1.53(b) or (d), however:

• A utility or plant divisional application filed on or after May 29, 2000 must be filed under 37 CFR § 1.53(b).

• A utility, design, plant or the national stage of an international divisional application filed before May 29, 2000 may be filed under 37 CFR § 1.53(d).

o A new oath or declaration is only required if a new invention is added to the divisional. o A new assertion as to small entity status is required for divisional applications.

Continuation Application o Continuation is a second application for the same invention claimed in a prior non-provisional

application. o Continuations may be filed under 37 CFR § 1.53 (b) or (d). o They must be filed during the pendency of the parent application. o A continuation application must claim the benefit of the prior non-provisional application under 35

USC § 120. Note that the prior application is not abandoned. o No new oath or declaration is required. o A new assertion as to small entity status is required, when applicable. o A continuation must have at least one common inventor with the original application. o The disclosure must be the same as the original application, and no new matter may be presented.

Page 7: TOL Patent Bar Outline

Patent Bar Outline © theOtherLives

* Important to know for the patent bar. 7

Continued Prosecution Application (CPA) Practice o A CPA is only allowed for a design application (after July 14, 2003). o CPAs may be delivered by fax*. o CPAs may only be filed while the design application is still pending, which is prior to: 1) Payment of

the issue fee, 2) Abandonment or 3) Termination of proceedings. o A request for suspension (see MPEP 709) must be filed at the same time as the CPA, if desired. o The filing date is the date of the CPA, though it may claim the benefit of an earlier § 120 application;

the prior application is always abandoned (there is no co-pending application). o A CPA must identify the prior application, disclose the same subject matter, and name the same or

fewer inventors (no extra inventors are allowed). o Small entity status must be (re)established if applicable. o Any terminal disclaimers in the prior application will be carried over to the CPA.

Substitute Application o A substitute application is an entirely new application with the same disclosure as the earlier (now

abandoned) application. o It is an application that is in essence a duplicate of a prior (earlier filed) application by the same

applicant abandoned before the filing of the substitute (later filed) application o The substitute application has its own filing date and is never co-pending with the original

application. Claiming the Benefit of an Earlier Filing Date Under 35 USC § 120 and § 119(e)

o If the earlier application provides an enabling disclosure of the subject matter of the claim under 35 USC § 112, a claim in an application is entitled to the benefit of the filing date of an earlier provisional or non-provisional application.

o The following conditions must be met to receive the benefit of an earlier filing date under 35 USC § 120 (non-provisionals) or 35 USC § 119(e) (provisionals):

• Both applications must have the same disclosure; • Both applications must be co-pending; • A specific reference to the prior application(s), as well as their relationship to the new

application, must be made in the first paragraph of the specification or application data sheet;

• Both applications must have at least one inventor in common; • The claim must be made within the appropriate time period, or else with petition for

unintentional delay. The benefit claim must also be made within the later of 4 months from the actual filing date of the application or 16 months from the filing date of the prior application* (either under § 111(a) or the national stage application);

• For a benefit claim under 35 USC § 119(e), the later-filed application must be filed not later than 12 months after the filing date of the prior provisional application; and

• Provisional applications must be translated to English with a statement that the translation is accurate for each application claiming the benefit of the provisional application.

Title to an Application Claiming Benefit of an Earlier Application o “The assignment records of the USPTO will only reflect an assignment of a divisional application or

continuation application (or any other application) if a request for recordation in compliance with 37 CFR § 3.28 (Requests for Recording), accompanied by the required fee (37 CFR § 3.41; Recording Fees), is filed… When the assignment is in a provisional application, see MPEP § 306.01.” (MPEP 201.12)

Right of Priority of Foreign Application o Rules specific to filing for foreign priority:

• Foreign applications must be filed in “a foreign country which affords similar privileges in the case of apps filed in the US or to citizens or the US or in a WTO member country;”

• The two applications must have at least one inventor in common; • The domestic application must be filed within 12 months of the foreign application (six

months for design applications); • The two applications must be for the same invention; • For applications filed under § 111(a), the claim for priority must be presented during the

pendency of the application and within the later of 4 months form the actual filing date of the application or 16 months from the filing date of the prior foreign application*;

• For applications entering the national stage, the claim for priority must be presented during the pendency of the application and within the time limit set forth in the PCT (see MPEP 1800); and

• Where the basis of the claim is an application for the inventor’s certificate, the requirements of 37 CFR § 1.55(b) (Claim for Foreign Priority) must also be met.

Page 8: TOL Patent Bar Outline

Patent Bar Outline © theOtherLives

* Important to know for the patent bar. 8

Using Certificate of Correction to Perfect Claim for Priority Under 35 USC § 119(a)-(d) or (f) o A reissue application may be used to correct failure to perfect a claim to foreign priority benefit

prior to patent issuance (see MPEP 1400 for rules on reissues). o Under some circumstances deemed acceptable by the Commissioner, a certificate of correction

may also cure such a failure. o In addition to filing a reissue or a certificate of correction, an applicant seeking correction of priority

must also submit: 1. A petition to accept a delayed claim for priority under 35 USC § 119(a)-(d) or (f), 2. A fee, and 3. A statement that the delay was unintentional.

Incorporation by Reference Under 37 CFR § 1.57(a) o 37 CFR § 1.57(a) provides that, “if all or a portion of the specification or drawing(s) is inadvertently

omitted from an application, but the application contains a claim under 37 CFR § 1.55 for priority of a prior-filed foreign application, or a claim under 37 CFR § 1.78 for the benefit of a prior-filed provisional, non-provisional, or international application, that was present on the filing date of the application, and the inadvertently omitted portion of the specification or drawing(s) is completely contained in the prior-filed application, the claim for priority or benefit shall be considered an incorporation by reference of the prior-filed application as to the inadvertently omitted portion of the specification or drawings.” (MPEP 201.17)

o The following are requirements for such incorporation by reference: • The applications must have been filed on or after September 21, 2004; • Any missing portion(s) of the specification or drawing(s) must have been omitted

inadvertently; • A claim for priority of a prior-filed foreign application, or a claim for the benefit of a prior-

filed provisional, non-provisional, or international application, must have been present on the filing date of the application;

• The inadvertently omitted portion must be completely contained in the prior-filed application;

• An amendment must be filed to include the inadvertently omitted portion within any time period set by Office (and during pendency of the application);

• A petition (and fee) must also be filed under 37 CFR § 1.17(f) (Filing Fees) if the application is not otherwise entitled to a filing date;

• A copy of the prior application must be provided except where the prior applications is one filed under 35 USC § 111;

• An English translation must be supplied for any prior-filed application in another language; and

• The applicant must identify where the inadvertently omitted portion can be found in the prior-filed application.

Summary of Offspring Application Requirements Statute Inventors in

Common New Subject

Matter Prior App. Status New Documents

Continuation 1.53(b) & (d) At least one No Co-pending Small entity status

Divisional 1.53(b) & (d) At least one No, only part is included Co-pending Small entity status

Continuation in Part 1.53(b) At least one Adds some new, keeps some old Co-pending All new papers

Continued Prosecution Application 1.53(d) At least one, no

extra No Abandoned All new papers

202: Cross-Noting

Notation in File History Regarding Prior US Applications, Including Provisional Applications o The first page of a printed patent will identify all prior applications for which benefit is claimed. A

patent owner should ensure that this information is correct. However, the inclusion of this information does not necessarily mean that the claims are entitled to the benefit of the earlier filing date.

203: Status of Application

New

Page 9: TOL Patent Bar Outline

Patent Bar Outline © theOtherLives

* Important to know for the patent bar. 9

o A new application is a non-provisional that has not yet received an Office action by the examiner. An amendment filed prior to the first Office action does not alter the status of a “new” application.

Rejected o A non-provisional application that contains an unanswered examiner’s action and is designated as

a rejected application. Its status as a rejected application will continue until the applicant replies satisfactorily to the examiner’s action (within a given period of time) or until the application becomes abandoned.

Amended o An “amended” or “old” non-provisional application is one that has been acted on by the applicant in

response to an examiner’s action. Such a reply may be an attempt to overcome a rejection by an examiner or an amendment of the application.

o For amended applications, if the applicant files a reply to an Office action and 6 months have elapsed since the filing of the reply and no response from the Office is received, the applicant should inquire as to the status of the application to avoid potential abandonment.

Allowed or in Issue o An “allowed” non-provisional application or an application “in issue” is one that, after having been

examined, is passed to issue as a patent subject to payment of the issue fee. Its status as an “allowed” application continues from the date of the notice of allowance until it is withdrawn from issue or until it issues as a patent or becomes abandoned.

Abandoned o An abandoned application is one that is removed from the Office docket of pending applications for

one of the following reasons: • Through formal abandonment by the applicant or by the attorney or agent of record (express

abandonment); • Through failure of the applicant to take the appropriate action at some stage in the

prosecution of a non-provisional application (implied abandonment); • For failure to pay the issue fee; or • After the expiration of 12 months in the case of a provisional application.

Incomplete An incomplete application is one that is not entitled to a filing date (such as for failure to submit the

specification or drawing(s)).

Page 10: TOL Patent Bar Outline

Patent Bar Outline © theOtherLives

* Important to know for the patent bar. 10

Page 11: TOL Patent Bar Outline

Patent Bar Outline © theOtherLives

* Important to know for the patent bar. 11

CHAPTER 300: OWNERSHIP AND ASSIGNMENT Introduction:

Ownership belongs to whoever has the controlling interest in a patent or a patent application (corporation or individual(s)).

o Patents may be bought and sold like property. o There may be multiple assignees, and each must record ownership with the PTO in order to take action on

the patent. Inventorship is a legal definition of the true inventors of a patent and may only be one or more individuals (never a

corporation). o Inventorship can never be transferred; however, an inventor may lose control of a patent/application by

assigning his ownership to another entity. 301: Ownership/Assignability of Patents and Application

Ownership o Ownership of a patent gives one the right to exclude others from making, using, offering for sale, selling, or

importing the invention (it does not give one the right to do these things). o Ownership initially vests in the inventor(s) of the patented subject matter. o The patent or application may be transferred, like property, by written assignment.

Assignment o Under 37 CFR § 3.1 (Definitions), assignment of “patent rights” is defined as “a transfer by a party of all or

part of its right, title and interest in a patent or patent application…” o Either the entirety or a percentage of one’s patent rights may be assigned.

Licensing o Unlike assignment, licensing refers to less than the total bundle of patents rights. Licensing may have

restrictions on time, geography or area of use. Individual Ownership

o May be an individual or an entity. o An assignee of entire interest (such as a corporation) that has received the sum of an entire assignment

from more than one inventor (or other prior owners) is an individual owner. Joint Ownership

o More than one person or entity may share the interests in a patent/application. This occurs when: • There are multiple partial assignees of the patent property; • There are multiple inventors who have not assigned their right, title and interest; or • A combination of the above.

Making the Assignment of Record o An assignment can be made in two ways, for two different purposes:

• The assignment can be made of record in the PTO, and this provides public notice of the assignment.

Note: This act of recording is not a determination by the PTO of the validity of the assignment. Also, this act alone does not, by itself, give the assignee right to take action on a patent, application or other proceeding.

• The assignment can be made of record in the file of the patent, application, or other patent proceeding, and this step permits the assignee to take action in the patent, application or other proceeding (37 CFR § 3.73; Establishing Rights of an Assignee to Take Action).

Accessibility of Assignment Records o Assignment documents relating to patents and published patent applications are open to the public. o The Office will not open only certain parts of an assignment document to public inspection. If such a

document contains two or more items and one is open to inspection, then the entire document will be opened*.

o Under 37 CFR § 1.14, assignment records of an earlier application will be open to the public because access may be granted to the earlier application when the offspring patent (divisional, continuation or CIP) is issued.

302: Recording of Assignment Documents Recording is voluntary. Requirements for recordable documents3:

o Cover sheet, containing: • Name of the partying conveying the interest; • Name and address of the partying receiving the interest; • Description of the interest conveyed or transaction being recorded; • Identification of the interests involved; • Name and address of the part to whom correspondence concerning the request to record the

document should be mailed; • Date the document was executed; and • Signature of the party submitting the document.

o Application number or patent number and filing date; o Inventor(s); and

3 As a general note, all documents submitted to the USPTO should be on one-sided paper.

Page 12: TOL Patent Bar Outline

Patent Bar Outline © theOtherLives

* Important to know for the patent bar. 12

o Recording fee. A legal assignment does NOT need to be in English. It will be accepted if translated and signed by the translator. Each patent cover sheet should also contain the number of patent applications or patents identified in the cover sheet

and the total fee. One assignment document may be used for multiple patents/applications. A cover sheet used to record a government interest must also:

o Indicate that the document relates to a governmental interest; and o Indicate, if applicable, that the document to be recorded is not a document affecting title.

Some assignment documents can be sent to the PTO by fax, and these must include: o An identified application or patent number; o One cover sheet to record a single transaction; and o Payment of the recordation fee.

Assignment-related documents that cannot be sent to the PTO by fax include: o Assignments submitted concurrently with newly filed patent applications; o Documents with two or more cover sheets; o Requests for corrections to documents recorded previously; o Requests for “at cost” recordation services; and o Resubmission of a non-recorded assignment.

Assignments and other documents affecting title may also be submitted to the PTO electronically through the PTO’s Electronic Filing System (EFS).

306: Assignment of Division, Continuation, Substitute, and CIP in Relationship to Parent Application The official assignment records will only reflect an assignment of a division or continuing application if a request for

recordation is in compliance. New assignment papers must be recorded for non-provisional applications that claim new material not included in the

previously filed provisional application. 307: Issue to Assignee

A request for a patent to issue to an assignee must be filed with the PTO no later than the date on which the issue fee is paid, and the request must indicate that the assignment has been recorded with the Office.

If the request is made after the date of issuance, the request must include a request for a certificate of correction under 37 CFR § 1.323 (Certificate of Correction of Applicant’s Mistake). This request must state that the assignment was submitted for recording before issuance of the patent.

If multiple names are on the assignment document, only the first name will be printed on the patent. 309: Restrictions Upon Employees of US Patent and Trademark Office*

Officers and employees of the PTO may not apply for or acquire a patent (or any interest therein) during the period of their appointments and for one year thereafter except by inheritance or bequest.

For patents applied for after one year post-employment, they may not be entitled to any priority date earlier than one year after the termination of their appointment.

315: Indexing Against a Recorded Certificate To record a document with respect to other patents or applications (such as the filing of an additional patent related to

the first disclosure), the document must: o Be submitted to the Assignment Division; o Be an original document or copy; o Include a cover sheet; and o Include a fee.

317: Handling of Documents in the Assignment Division Assignments that are made conditionally (like on an agreement to get married) are regarded as absolute assignments

for Office purposes, as the Office does not determine whether such conditions have been fulfilled (and thus will record the assignment).

323: Procedures for Correcting Errors in Recorded Assignment Documen If there is an error in the recorded assignment document, a “corrective document” must be submitted to the PTO. A copy of the original assignment document with the correction (initialed and dated) will suffice and will require a new

Recordation Form Cover Sheet. 324: Establishing Right of Assignee to Take Action*

Ownership needs to be established when: Ownership does NOT need to be established when: • An assignee signs a request for prosecuting the

application if ownership has been changed • An assignee signs a request for the status of the

application or wants to allow someone to inspect the application

• The application becomes abandoned • A legal representative is appointed • A terminal disclaimer is signed • A reissue application is filed • Correction of inventorship occurs • An application under 37 CFR § 1.47(b) is filed • An Issue Fee Transmittal form is signed • A reply to an Office Action is signed

• There is an assignee for a continued prosecution application where ownership has not changed

• Signing a small entity statement • Signing a statement of common ownership between two

inventions • Signing a NASA or DOE property rights statement • Signing a certificate under 37 CFR § 1.8 • Filing a request for re-examination of a patent

Page 13: TOL Patent Bar Outline

Patent Bar Outline © theOtherLives

* Important to know for the patent bar. 13

CHAPTER 400: REPRESENTATIVE OF INVENTOR OR OWNER General

An inventor may elect to prosecute an application by him/herself (pro se), or s/he may obtain help from a registered patent attorney or agent who has not been suspended or excluded by the PTO.

Note that the registered attorney or agent may not appoint an unregistered associate to assist in prosecution (MPEP 402.02).

A Customer Number is used to designate the correspondence address of a patent application or patent for the purposes of correspondence, fees, and to associate power of attorney of the application.

o A customer number may be used to grant Power of Attorney (POA) to more than one person (i.e., multiple attorneys in a law firm). However, POA can only be granted to individuals and never to a law firm as an entity.

In the case of joint inventors, one inventor may grant POA to a co-inventor, though it requires a request for special recognition.

402: Power of Attorney (POA); Acting in a Representative Capacity

Revocation o POA may be revoked by the applicant at any time and for any reason.

• All the owners of a patent application must sign the papers giving or revoking the POA. o Revocation of POA is effective on the date that notice is received at the Office.

Attorney or Agent Withdraws o An attorney or agent may withdraw his POA with permission of the PTO or Commissioner at least

30 days before the last possible date for any required response. o Each attorney of record must sign the Notice of Withdrawal, and the withdrawal is only effective

once approved. Assignee can Revoke Power of Attorney (POA) of Applicant and Appoint New POA

o An assignee may revoke POA and/or exclude the inventor from access to the patent, unless an “irrevocable right to prosecute has been given.”

Appointment/Revocation by Less Than All Applicants or Owners o POA may be revoked by one joint inventor with submission of:

1. A § 1.183 petition, 2. Sufficient reason to explain the missing consent of the joint inventor(s), and 3. A fee.

403: Correspondence – With Whom Held

Correspondence Held With Associate Attorney o If there is a POA on record, correspondence will be held with him/her. o If there was no oath submitted with the application, the only individuals who may change the

correspondence address are the inventor or assignee who filed the application or the attorney or any registered practitioner named in the transmittal papers.

Rules for Correspondence o Correspondence will be held with the POA. If there is no POA, correspondence will be held with

the person/address identified on the application data sheet (ADS). o Double correspondence will not be undertaken. If there is an address provided with the

Customer Number, this address will be used. 408: Telephoning Patent Practitioner

It is not necessary for the practitioner to request a telephone conversation with an examiner. The practitioner may simply call the examiner if s/he feels the call will be beneficial to advance

prosecution of the application. 409: Death, Legal Incapacity, or Unavailability of Inventor

Death of Inventor; Prosecution by Administrator or Executor o An application may be filed and prosecuted by the heirs, administrators or assignees of the

inventor. However, a new POA is required. Unavailability of Legal Incapacity of Inventor

o 37 CFR § 1.43 may be available where an inventor is temporarily unavailable (e.g., due to hospitalization).

o Generally, petitions under 37 CFR § 1.47 (Filing When an Inventor Refuses to Sign or Cannot be Reached) are used when an inventor is incapacitated or cannot be reached*.

o A minor may sign an oath or declaration under as long as he or she understands the document that he or she is signing.

Page 14: TOL Patent Bar Outline

Patent Bar Outline © theOtherLives

* Important to know for the patent bar. 14

o A CEO as an officer of a corporation may sign the necessary oath or declaration under 37 CFR § 1.63 (Oath or Declaration) on behalf of the corporation as the 37 CFR § 1.47(b) applicant.

At Least One Joint Inventor Available o If an inventor refuses to sign the oath or declaration or is unavailable:

• All available inventors must make an oath or declaration on their own behalf and make an oath or declaration on behalf of a non-signing joint inventor;

• The application must be accompanied by proof that the non-signing inventor cannot be found or refuses to sign; and

• The last known address of the non-signing inventor must be listed. Rights of the Non-Signing Inventor

o The non-signing inventor is entitled to inspect any paper in the application, order copies thereof, and make his or her position of record in the file wrapper of the application, either by him/herself or with the assistance of a registered agent or attorney.

o If a non-signing inventor believes that s/he is the sole inventor, the non-signer should file his/her own application and request that it be put in interference with the 37 CFR § 1.47 application.

410: Representations to the USPTO

The POA may fill out and in some cases sign many of the patent related documents required during the prosecution of the application.

Separate verification statements are no longer needed for small entities, English translations, petitions to make special, or claims for foreign priority.

Page 15: TOL Patent Bar Outline

Patent Bar Outline © theOtherLives

* Important to know for the patent bar. 15

CHAPTER 500: RECEIPT AND HANDLING OF MAIL AND PAPERS 501: Filing Papers with the USPTO

Certain types of patent documents require expedited review by the patent office and should be hand-delivered or faxed. Generally, these include:

o Petitions for express abandonment to avoid publication under 37 CFR § 1.138(c); o Petitions to withdraw an application from issue under 37 CFR § 1.313(c); o Request for expedited examination of a design application (rocket docket); and o Papers required to be hand-carried or faxed to the Office of Patent Publication.

Generally, all patent-related papers may be hand-carried to the Customer Service Window at the PTO. 502: Depositing Correspondence

General o New applications – both provisional and non-provisional – may be deposited by mail, hand-

delivered or filed via the Office’s Electronic Filing System (EFS). o The Office requests that all deposited papers pertaining to a filed application be identified by the

following: • Application number; • Art Unit number (copied form the most recent Office communication); • Filing date; • Name of the examiner who prepared the most recent Office action; • Title of invention; and • Confirmation number.

o On the back of all drawing related materials, the applicant should list the: • Applicant’s name; • Docket number; and • Title of invention.

Correspondence Transmitted by Fax* o The date accorded to correspondence sent by fax is the date that all papers in the transmission

are received by the PTO. If the fax is received on a weekend or holiday, it will be accorded the next date that the Office is open.

o The following correspondence may NOT be sent by fax and will NOT be accorded a filing date if it is sent by fax:

• Correspondence requiring an original signature; • Certified documents; • Correspondence which cannot receive the benefit of the certificate of mailing or

transmission; The exception is a CPA, which may be sent by fax as long as it includes

authorization for payment*. • Color drawings; • A request for reexamination; and • Correspondence to be filed in a patent application subject to secrecy order and directly

related to the secrecy order content of the application. o The following correspondence may be sent by fax*:

• CPAs filed under 37 CFR § 1.53(d); • Amendments; • Declarations; • Petitions; • Information disclosure statements (IDS); • Terminal disclaimers; • Notices of appeal and appeal briefs; • Requests for continued examination (RCE); • Assignment documents; • Issue fee transmittals; and • Authorization to charge deposit account.

Correspondence Signature Requirements o Three types of signatures are accepted by the Office:

• Handwritten signature: Should be an original signature in dark ink. Copies of handwritten signatures may be accepted in some cases.

• “S-signature”: An S-signature is created by electronic or mechanical means and is placed between two forward slash marks.

Page 16: TOL Patent Bar Outline

Patent Bar Outline © theOtherLives

* Important to know for the patent bar. 16

• Electronic Filing System (EFS) character coded signature: The applicant’s EFS signature is a code consisting of only of letters of the English alphabet, or Arabic numerals, or both, with appropriate spaces and commas, periods, apostrophes, and hyphens as punctuation.

503: Application Number and Filing Receipt

Application numbers consisting of a series code (2 digits) and a serial number (6 digits) are assigned by the Office of Initial Patent Examination (OIPE) immediately after mail has been opened.

An applicant may submit a self-addressed, stamped postcard with their patent application, and the OIPE will return to the applicant the postcard with both the receipt date and the application number. The identifying data on the postcard should include:

o Applicant’s name(s); o Title of invention; o Number of pages of the specification, claims (for non-provisionals), and sheets of drawing; o Whether an oath or declaration is included; o A list of any additional forms included with the application; and o Amount and manner of paying the fee.

It is important for an applicant to itemize on the postcard all items being submitted, as the returned receipt of the postcard is evidence of what was received by the Office in the event that submitted paperwork is later lost.

504: Assignment of Application for Examination

“The Office of Initial Patent Examination assigns a non-provisional application to the art unit to which it appears to belong. Provisional applications will not be examined.” MPEP 504.

506: Completeness of Original Application

General o If a non-provisional application is submitted without at least an acceptable (coherent)

specification, one or more claims, and any required drawings, the application will be “incomplete” and will not receive a filing date. Rather, the incomplete application will be held in the OIPE, and the applicant will be sent a “Notice of Incomplete Application”.

o A provisional application must include at least a specification containing a description and a drawing (if necessary) in order to be accorded a filing date.

o An Informal application will not be accorded a filing date. An application is considered informal if it is typed on both sides of the paper or is not permanent, legible, or reproducible.

Review of Refusal to Accord Filing Date* o An applicant may petition for review of the refusal to grant a filing date to an application. The

petition must include a fee, as well as the applicant’s arguments that the items noted were not missing or that a filing date should be assigned in the absence of such items if they are believed to be unnecessary.

o If the applicant alleges that no defect exists, s/he may also include with the petition a request for reimbursement of the petition fee.

509: Payment of Fees

General o Fees should be 1) paid in advance, and 2) itemized so the PTO can determine for what the fees

are being paid. o The Office charges a fee for processing the fee paid by a refund check.

Rules for Various Filing Fees o A non-provisional application based on the conversion of a provisional application must include

the filing fee for the non-provisional application since the filing fee for the provisional does not cover the cost of the converted application.

o The fee that must accompany a request for continuing examination is the same amount as the applicant would pay for the basic filing of the utility application.

Filings that DO NOT Require Immediate Payment of Fee o Protest filing; o Petitions to make special; o Access for abandoned patents; o A timely response to an Official Action; o A timely filing of an IDS; o Request for an interview; o Certificate of Correction for error made by the PTO;

Page 17: TOL Patent Bar Outline

Patent Bar Outline © theOtherLives

* Important to know for the patent bar. 17

o A supplemental declaration or amendment; o A petition requesting the withdrawal of an abandonment decision; o A response to an examiner’s reasons for allowance; and o A citation of prior art under 35 USC § 301.

Deposit Accounts o An applicant may establish a deposit account from which the PTO can deduct fees. o A credit card may not be used to replenish the deposit account. o An applicant who authorizes a charge to a deposit account having insufficient funds will be

charged an additional fee, and the action associated with the payment of the original charge may be delayed (if the payment is required for action by the PTO).

Small Entity Status o An applicant may qualify for small entity status if s/he (or it, in the case of corporations) falls into

one of the following categories: • Independent inventors who have NOT assigned, granted, conveyed, or licensed the

invention; • Small businesses with less than 500 employees; or • Nonprofit organizations:

Universities or other institutions of higher education in any country, Organizations exempt from taxation, or Nonprofit scientific or educational organizations.

o If small entity status is properly established at the time of the filing, the inventor is entitled to maintain small entity status until any issue fee is due.

o Discounted fees*: Fees associated with the following filings will be discounted by 50% for small entities:

• Application filing fees; • Extension of time, revival, and appeal fees; • Patent issue fees; • Statutory disclaimer fees; and • Maintenance fees.

o Non-discounted fees: • Petition and processing fees; • Document supply fees; • Certificate of correction fees; • Request for reexamination fees; • Miscellaneous fees; and • International application fees.

Claiming Small Entity Status o Small entity status must be established before discounted fees will be allowed. o A registered attorney or agent may NOT sign the assertion of small entity status. o An applicant may receive a refund if the application was filed with non-small entity status initially

and later qualified for small entity status. This request must be placed within three months of the timely, fully-paid fee (non-extendable).

o Fees are never due retroactively if the inventor’s status changes from small entity to non-small entity during prosecution of the application. Rather, future payments are calculated at the non-small entity rate.

510: PTO business hours

The USPTO is open M-F from 8:30am – 5:00pm, except federal holidays. 512: Certificate of Mailing or Transmission

A Certificate of Transmission allows an applicant to state the date on which the paper will be transmitted by fax.

o The filing date is based on the time sent, never when it is received (faxes are recorded as being sent on the earlier date).

A Certificate of Mailing states the date that a document was mailed and may be used for filing: o A request for continued examination, or o A reply under 37 CFR § 1.111 in a non-provisional patent application.

Correspondence sent by Certificate of Mailing will be considered timely filed if: o It is addressed correctly and there is enough postage for first class mail status, and o There is a dated certificate of mailing for each piece of the correspondence.

Rules for using Certificate of Mailing: o Each certificate must bear the signature of the applicant, assignee or registered practitioner.

Page 18: TOL Patent Bar Outline

Patent Bar Outline © theOtherLives

* Important to know for the patent bar. 18

o Ideally, each certificate should be placed directly on the paper being submitted. However, it may be on a separate, securely attached page.

o Each certificate must include the serial number, filing date and the type of form being filed. 513: Deposit as Express Mail with US Postal Service (USPS)*

The only type of “Express Mail” service that the Office can receive is “Post Office to Addressee” service. This service provides for the use of a mailing label that clearly indicates the date of deposit.

“Express Mail” can always be used for filing documents with the PTO. However, an applicant may wish to fax or hand-deliver allowed documents that need urgent attention by the PTO.

A separate Certificate of Mailing does not need to be included when using “Express Mail”. The Express Mail mailing label number does not need to be placed on the correspondence prior to

mailing*. Date of Deposit: Any correspondence that is sent via Express Mail will be stamped, and considered as

filed on the date of deposit, regardless of the day that it falls on. The filing date is the “date in” on the Express Mail label, which is stamped by USPS (so using a dropbox is done so at the applicant’s risk).

o Parties using dropboxes should protect themselves by maintaining a log where notations are entered by the person depositing correspondence as Express Mail within 1 business day after deposit with the USPS. This evidence could be useful to support a petition for correction of date of deposit.

Petitions: If the date of filing at the USPS is recorded as something different than the “date in” on the Express Mail label, a prompt petition may be made, including:

o The number located on the Express Mail mailing label is placed on the paper or fee that constitutes the correspondence prior to the original mailing by Express Mail;

o A true copy of the Express Mail mailing label that shows the “date in” or any other official notation by the USPS earlier than the date the PTO recorded as the filing date; and

o A true copy of any returned postcard receipt to show the date of deposit. Summary of Deposit Options for Various Types of Correspondence

Type of Correspondence Fax Certificate of Mailing

“Express Mail”

Continued Prosecution Application Yes No Yes

Continuation Application No No Yes

Continuation in Part No No Yes

Request for Continued Examination Yes Yes Yes

Foreign Document Yes No Yes

International Application No No Yes

Reply under 37 CFR § 111 Yes Yes Yes *Remember that all correspondence may be deposited in person or by “Express Mail”.

Page 19: TOL Patent Bar Outline

Patent Bar Outline © theOtherLives

* Important to know for the patent bar. 19

CHAPTER 600: PARTS, FORM AND CONTENT OF APPLICATION 601: Content of Provisional and Non-Provisional Applications

Complete Application o General Application Requirements (for provisional and non-provisional)

• All sheets should be the same size (8.5 x 11 inches) • Margins: top ¾ inch, left 1 inch, right ¾ inch, bottom ¾ inch • No holes in the sheets • 1.5 or double spaced • The pages should be numbered in the middle and preferably at the bottom

o Non-Provisional Application Filed Under 35 USC § 111(a) • Order of Parts: The following is the preferred order of arrangement for the specification

parts. Each item should appear in upper case, without underling or bold type: Title of the invention Cross-reference to related application Statement regarding federally sponsored research or development The names of the parties to a joint research agreement Reference to a “Sequence Listing,” a table, or a computer program listing

appendix submitted on CD and an incorporation-by-reference of the material on the CD

Background of the invention o Field of the invention o Description of related art including information disclosed under 37 CFR §

1.97 (Filing of IDS) and 37 CFR § 1.98 (Content of IDS) Brief summary of the invention Brief description of the several views of the drawing Detailed description of the invention Claims (commencing on a separate sheet) Sequence listing (if on paper)

• 37 CFR § 1.53(b) may be used to file any original, reissue, or substitute non-provisional and any continuing application (continuation, divisional, or continuation-in-part).

Filing Date: A filing date will be assigned to a non-provisional application upon receipt of the specification containing a description and claim (at least one) and any necessary drawings.

• 37 CFR § 1.53(d) may be used to file either a continuation or a divisional application (but not a continuation-in-part) of a design application (the parent must be a design application). An application filed under § 1.53(d) must be filed during the pendency of the application.

• Any required fees (with a surcharge) and oath//declaration may be submitted after the filing date of a § 1.53(b) or (d) application; however, no new material may be added to the disclosure of the application.

o Provisional Application Filed Under 35 USC § 111(b) • The preferred order of parts for a provisional is the same as that for a non-provisional.

However, neither claims nor an oath or declaration are required for provisional applications.

• A filing date will be accorded to a provisional application when the written description and any necessary drawings are received by the Office. Remember that this submission must make clear that the application is a provisional or else it will be treated as a regular application.

• Unlike non-provisional, a provisional application requires a cover sheet with identifying information, including:

That it is for a provisional application; The residence(s) of the inventor(s); The title of the invention; and The name and registration number of the attorney or agent (if applicable), the

docket number used by the person filing the application (if applicable), and the correspondence address.

If there is a governmental interest in the invention/application, this must also be stated on the cover sheet.

• Provisionals should not include an IDS. The Office will not accept one. o Applications Filed Without Drawings

Page 20: TOL Patent Bar Outline

Patent Bar Outline © theOtherLives

* Important to know for the patent bar. 20

• An application filed without at least one drawing is initially inspected to determine whether 1) a drawing is referred to in the specification, or 2) a drawing is necessary to understand the invention.

• Drawings are generally considered unnecessary for: Applications containing at least one process or method claim; Composition applications; Coated articles or products; Articles made from a particular material or composition; Laminated structures; Articles, apparatus, or systems where sole distinguishing feature is presence of a

particular material. • If the Office determines that a drawing is required but one was not submitted with the

original application, a “Notice of Incomplete Application” will be sent to the applicant, and a filing date will not be awarded until the application is completed.

o Power of Attorney (POA) • Every POA must include the attorney’s or agent’s full mailing address, as well as the

telephone and fax numbers. • A POA may be incorporated in the oath/declaration form if the POA is given by the

inventor(s), or else a separate POA form may be used. o Bibliographic Information – Application Data Sheet (ADS)

• General An ADS contains bibliographic data with the following section headers with the

corresponding information under each: o Applicant information;

• Name, residence, mailing address, and citizenship of each applicant.

o Correspondence information; • Address to which correspondence is to be directed (may be

indicated by reference to a Customer Number). o Application information;

• Title of the invention, a suggested classification by class and sub-class, the Technology Center (TC) to which the subject matter of the invention is assigned, the total number of drawing sheets, a suggested drawing figure for publication (in a non-provisional application), any docket number assigned to the application, and the type of application (e.g., plant, reissue, etc.).

• May also include any contained information that pertains to a secrecy order.

o Representative information; • Registration number appointed with a POA in the application

(preferably by reference to a Customer Number). o Domestic priority information;

• Application number, filing date, status and relationship of each application for which benefit is claimed under 35 USC § 119(e), 120, 121 or 365(c).

o Foreign priority information; and • Application number, country and filing date of each foreign

application for which priority is claimed. o Assignee information.

• The name and address of the assignee of the entire right, title and interest in an application.

The name(s) of the inventor(s), as well as the corresponding citizenship(s), must be presented in the oath/declaration even if it is presented in the ADS.

• Supplemental ADS Requirements The supplemental ADS must be titled “Supplemental Application Data Sheet” (or

some close variation thereof); The supplemental ADS must be a full replacement copy of the original ADS; The supplemental ADS must be submitted with all changes indicated by

underlining for additions and strike-through or brackets for deletions; and The footer information should include the word “Supplemental” in place of “Initial”

and should also contain the Application Number and Filing Date. Incomplete Application

Page 21: TOL Patent Bar Outline

Patent Bar Outline © theOtherLives

* Important to know for the patent bar. 21

o Notice of Missing Parts – Gets filing date • Sent out when an application will receive a filing date but is missing the oath/declaration,

translation or fee. o Notice of Incomplete Application – Does NOT get filing date

• Sent out when further action by the applicant is necessary for the application to be accorded a filing date, such as when a non-provisional application is filed without at least one claim.

• The application will be processed as an incomplete application under 37 CFR § 1.53(e) (Failure to Meet Filing Date Requirements) if the period set in the notice passes and the applicant has either not completed the application or not filed a petition under 37 CFR § 1.53(e) with a petition fee.

o Notice to File Corrected Application Papers - Gets filing date • Sent out when the specification is non-complying with 37 CFR § 1.52. Will only affect the

filing date if the deficiency is not fixed in the time period specified in the notice. o Notice of Omitted Items – Filing date depends on form of correction

• Sent out if an application is missing a critical page; such as in an application with 10 pages that is missing page 9 or when not all figures of the drawings are present.

• An applicant may choose to send in the missing page (which will change the filing date of the application) or not respond to the notice (and therefore retain the original filing date but give up the right to ever submit the missing page).

If an applicant sends in the missing paper(s), s/he must do so within 2 months of the date posted in the notice along with an oath/declaration referring to the omitted page(s) and a petition under 37 CFR § 1.182. The filing date for the application will then be the date that the Office receives the missing paper(s), oath/declaration and petition.

If an applicant does not send in the missing paper(s) within 2 months, s/he will not be able submit the paper(s) under this application and will retain the original filing date.

o In this case, a preliminary amendment of the specification will be required in a non-provisional application to renumber the pages consecutively and cancel any incomplete sentences caused by the absence of the omitted page(s).

• If the applicant thinks the page was there when the application was mailed, he may, within 2 months from the date of Notice, file a petition with the fee under 37 CFR § 1.53(e) along with evidence of the deposit.

602: Original Oath or Declaration*

Requirements - 37 CFR § 1.63 o An oath or declaration must:

• Be executed – A person of any age may sign as long as they understand the document that they are signing;

• Identify each inventor by full name – last name and at least one given name along with any other given name or initial;

• Identify the country of citizenship of each inventor; • State that the person making the oath or declaration believes the named inventor or

inventors to be the original and first inventor or inventors of the subject matter which is claimed and for which a patent is sought;

• Identify the application to which it is directed; • State that the person making the oath/declaration has reviewed and understands the

contents of the application, including the claims, as amended by any amendment specifically referred to in the oath or declaration; and

• State that the person making the oath/declaration acknowledges the duty to disclose to the Office all information known to the person to be material to patentability as defined in § 1.56 (duty to disclose).

o If not provided in an ADS, the oath/declaration must also identify: • The mailing address and the residence if an inventor lives at a location that is different

from where the inventor customarily receives mail (for each inventor); and • Any foreign application for parent – or inventor’s certificate – for which a claim for priority

is made and any foreign application having a filing date before that of the application on which priority is claimed.

Oath Cannot be Amended o An oath/declaration may never be amended.

Page 22: TOL Patent Bar Outline

Patent Bar Outline © theOtherLives

* Important to know for the patent bar. 22

o If an error exists, a new oath/declaration must be filed. • Alternatively, some deficiencies may be cured by filing an ADS.

603: Supplemental Oath or Declaration*

General o The original disclosure cannot be altered by merely filing a subsequent oath/declaration that

refers to different papers. o When an inventor who executed the original declaration will not execute a required supplemental

declaration, it is possible for that particular requirement to be suspended or waived in accordance with 37 CFR § 1.183 (Suspension of Rules).

Supplemental Oath or Declaration Filed After Allowance o Since filing a supplemental oath or declaration is a matter of right, this may be done even after

allowance. 605: Applicant

Applicant’s Signature and Name o Applicant’s name may be signed in non-English characters. o A party with proprietary interest in the invention claimed in an application can sign on behalf of

the inventor if s/he is unable or refuses. Joint Inventors

o The order of patentee’s listed on an issued patent is the order in which they appear on the oath/declaration.

o In order to qualify as an inventor, one must have actually contributed to at least one of the claims. • In the case of a provisional where no claims are made, each listed inventors must have

actually contributed to the subject matter. o 35 UCS § 116 allows inventors to apply for a patent jointly even though:

• They did not physically work together or at the same time; • Each did not make the same type or amount of contribution; or • Each did not make a contribution to the subject matter of every claim of the patent.

606: Title of Invention

The title of the invention should be descriptive, technically accurate and fewer than 500 characters. Neither “a”, “an” nor “the” may start the title. If an examiner does not believe the title is descriptive enough, s/he may require a new one.

607: Filing Fee

Filing fee, search fee and examination fee are distinct fees. All three fees are due upon filing of a non-provisional application, however, they may be paid at a later

date accompanied by a surcharge (filing date will not be affected by failure to pay fees at time of filing). o Only a filing fee is required for provisional applications since there is no search or examination

involved. Application Size Fee

o An application size fee will be required in addition to the basic filing fee where the specification (including drawings but excluding sequence listing or computer program on an electronic medium source) exceeds 100 pages. The fee will be charged for every 50 pages over the 100-page limit.

o If the application is submitted via the EFS, the page number for the purpose of assessing fees will be considered 75% of the actual page number.

Excess Claims Fee o An additional fee will be charged for each independent claim in excess of three and each claim –

independent or dependent – in excess of twenty. o Excess claims charges are assessed only after taking into account claims that are canceled. o Fees for proper multiple dependent claims (MDCs) are calculated based on the number of claims

on which the MDC depends. An improper MDC will count as one claim. Unless specified by the applicant, the Office will apply fee payment in the following order:

o The basic filing fee; o The application size fee; o The late filing surcharge; o The processing fee for an application filed in a language other than English; o The search fee; o The examination fee; and o The excess claims fee.

Returnability of Fees*

Page 23: TOL Patent Bar Outline

Patent Bar Outline © theOtherLives

* Important to know for the patent bar. 23

o The Office may refund: • A fee paid by mistake; or • Any fee paid in excess of the amount required.

o Where an applicant takes an action (like filing a patent) by mistake and pays the required fee, this does NOT qualify as a “fee paid by mistake.”

o All requests for refunds must be filed within a two-year, non-extendable time limit. • However, requests for refunds based on later establishment of small entity status must

be filed with three months. 608: Disclosure

Specification o The specification is a written description of the invention explaining how to make it and use it.

The specification should distinguish the invention from prior inventions, as well as make clear all improvements in the art.

o The spec must commence on a separate sheet and the text must be written in a non-script typefont. British English spellings are acceptable.

o Hyperlinks • It is acceptable to have non-active hyperlinks only when the hyperlinks are part of the

invention. • However, it is improper to attempt to incorporate subject matter by reference using

hyperlinks. o The specification may contain tables and chemical formulas instead of formal drawings. o The written specification may not contain drawings, flowcharts or graphs. o The specification is limited to the known applications of the subject matter disclosed and may not

reserve claims to future applications of the subject matter. o Non-English Specification

• A specification may be filed in a language other than English as long as it is accompanied by a translation, which:

1. Includes a signed statement that the translation is accurate; 2. Is done only in an emergent situation, 3. Is filed within the specified time limit (37 CFR § 1.52), and 4. Is accompanied by a fee.

Arrangement of Application o The following order of arrangement of non-provisional application parts is preferable but not

required by the Office: • Title of invention; • Cross-references to related applications; • Statement regarding federally sponsored research or development; • The names of the parties to a joint research agreement; • Reference to a sequence listing, a table, or a computer program listing appendix

submitted on a CD and incorporated by reference. The total number of CDs, including duplicates, must be listed;

• Background of the invention; Field of the invention Description of the related art

• Brief summary of the invention; • Brief description of the several views of the drawings; • Detailed description of the invention; • Claim or claims; • Abstract of the disclosure; • Sequence listing (if on paper);

o The section headers should appear in upper case letters, without underlining or bolding. Abstract of the Disclosure

o The abstract should make clear to even a layperson the nature and gist of the disclosure and should make clear that which is new to art; however, the abstract should not compare the invention to the prior art.

o Where applicable, the abstract should include the following: • If the invention is a machine or apparatus, its organization and operation; • If the invention is an article, its method of making; • If the invention is a chemical compound, its identity and use; • It the invention is a mixture, its ingredients; • If the invention is a process, its steps.

Page 24: TOL Patent Bar Outline

Patent Bar Outline © theOtherLives

* Important to know for the patent bar. 24

o The abstract must commence on a separate sheet of paper. o The abstract should be a single paragraph with 50 – 150 words and no more than 15 lines. o The language should be clear and concise and not repeat any information provided in the title.

Background of the Invention o Consists of two parts:

• Field of the invention: Statement of the pertinent field of art. • Description of the related art: Brief description of the pertinent prior art, including

references where appropriate. Brief Summary of the Invention

o Should set out the exact nature, operation, and purpose of the invention, and should be consistent with the subject matter of the claims.

Brief Description of Drawings o A description of each distinguished part of each drawing must be provided. For example, if the

applicant labels a figure with parts 1A, 1B and 1C, s/he must provide a description for each of parts 1A, 1B and 1C as opposed to providing a description merely for Figure 1.

Detailed Description of the Invention o Any person skilled in the art or science should be able to make and use the invention from

reading the description of the invention, without doing extensive experimentation. o Tables may be included in the description portion of the specification; however, the same tables

must not be included in both the drawings and the description portion of the specification. Mode of Operation of Invention

o The best mode of operation for the invention known to the inventor at the time of filing must be disclosed.

Claims o Numbering of Claims

• The original numbering of claims must be preserved throughout prosecution. If claims are deleted and/or added during prosecution, they will be renumbered only after allowance.

o Statutory Requirement of Claims • “35 USC § 112 requires that the applicant shall particularly point out and distinctly claim

the subject matter which he or she regards as his or her invention.” MPEP 608.01(k) o Form of Claims

• Claims should be placed at the end of the specification. • If there are more than one claim, they should be separated by a line of indentation. • They should be arranged by breadth of scope, such that the first claim presented is the

broadest and the last is the most restrictive. • Claims of like species should be grouped together. • Process and product claims should be grouped separately. • Measurements should be provided first in metric units, followed by English units.* • Claims may contain tables only to the extent necessary to comply with specification

guidelines. • There should never be two claims covering the same subject matter. • The preamble of a claim contains a description of things or steps and a body that sets

forth the elements or steps and limitations of the claims. • Grammatical rules for claims

They should begin with “The invention claimed is…” or “I (or we) claim…” Each claim should begin with a capital letter and end with a period. Either a semicolon or a comma separates each paragraph, and the last step

should have an “and” following the semicolon or comma. If there are numerous claims, they should be numbered consecutively in Arabic

numerals. o Specific Types of Claims

• Markush Claims: A Markush group is used to define a claim limitation where there is no generic term to describe a group of things (like side groups of a compound series). A Markush group is an acceptable form of alternative expression, provided the introductory phrase “consisting of” and the conjunctive “and” is used, such as the delineation that a group consists of “groups A, B, C and D.”

• Jepson Claims: A Jepson claim has a preamble that defines what is old, a body defining what is new and a translational phrase such as “the improvement comprising of”. The “what is old” part is general taken as admitted prior art.

o Dependent Claims • Multiple Dependent Claims (MDCs)*

Page 25: TOL Patent Bar Outline

Patent Bar Outline © theOtherLives

* Important to know for the patent bar. 25

An MDC is a dependent claim that refers back in the alternative to more than one preceding independent or dependent claim. MDCs must be presented in alternative form.

An MDC may refer in the alternative to only one set of claims. o Example of proper MDC: “A device as in claims 1, 2, 3 or 4…” o Example of improper MDC: “A device as in claims 1, 2, 3 or 4 made by a

process of claims 5, 6, 7, or 8.” An MDC may NOT depend from another MDC. Examples of acceptable MDC language*:

o Claim 5: A gadget according to claims 3 or 4, further comprising… o Claim 5: A gadget as in any one of the preceding claims, in which… o Claim 8: A gadget as in any one of claims 4 -7… o Claim 10: A gadget as in any one of claims 1 – 3 or 7 – 9….

Examples of unacceptable MDC language*: o Claim 5: A gadget according to claim 3 and 4, further comprising... o Claim 9: A gadget according to claims 1 – 3, in which… o Claim 5: A gadget according to claims 6 or 7… (MDC may only refer to

preceding claims) • Calculation of fees

A multiple dependent claim is considered to be the number of dependent claims to which it refers.

• Infringement test A proper dependent claim may not be conceivably infringed upon by anything

that would not also infringe the basic claim. A dependent claim should include every limitation of the claim from which it

depends. Infringement will depend largely on language used, such as “comprising” vs.

“consisting”. o Comprising: The list of elements and limitations that follow the word

“comprising” defines the minimum characteristics for infringement, there can still be infringement if an alleged infringing thing or method has more protected characteristics.

o Consisting: The list that follows the word “consisting” defines the exact combination of elements that must be present in order for another product to be infringing.

• Claim form and arrangement Any claim depending from a dependent claim should not be separated from that

claim by any claim that does not also depend from it. • Rejection and objection

If a base claim is canceled, any claims depending from it should be rejected. If a base claim is rejected, any claims depending from it should be objected to

(rather than rejected) if it is otherwise allowable. Generally this means that the claim may be rewritten in independent form and will be allowed.

Completeness o A disclosure in an application, to be complete, must contain such description and details as to

enable any person skilled in the art or science to which the invention pertains to make and use the invention as of its filing date (See MPEP 608.01)

Derogatory Remarks About Prior Art in Specification o An applicant may refer to the general state of prior art in the field of the invention. However, s/he

may not make derogatory remarks about such art. Trademarks and Names Used in Trade

o Trademarks may be used as long as the product to which the trademark refers is presented in such language that its identity is clear.

o Names used in trade are proprietary names by which a product is known in the art but not necessarily known by the public. Nonetheless, these may be used if:

• Their meanings are established by an accompanying definition that is sufficiently precise and definite to be made a part of a claim, or

• Their meanings are well-known and satisfactorily defined in the literature in this country. Drawings

o Drawings are generally required to understand the invention. However, the following types of inventions do NOT require drawings in order to be understood:

• Coated articles or products;

Page 26: TOL Patent Bar Outline

Patent Bar Outline © theOtherLives

* Important to know for the patent bar. 26

• Articles made form a particular material or composition; • Laminated structures; • Article, apparatus, or systems where the sole distinguishing feature is the presence of a

particular material. o If a drawing is considered necessary to understand the invention, an application will not be

accorded a filing date unless is contains the drawing(s) – or until the drawings are submitted. o Drawings

• Drawings must show every feature of the invention specified in the claims, as well as any claimed improvements over prior art.

• Symbols and other labeling may be used were appropriate. • Projection lines should not be used, and drawings should not contain center lines. • Arrows may be used as long as their meaning is clear. • Indications of “actual size” or presentation of a scale should not be used, as the size of

the drawing may change with reproduction. • Drawings must be submitted on paper that is flexible, strong, white, smooth, non-shiny,

and durable. • All drawings sheets must be the same size and the sheets must be:

21.0 cm. by 29.7 cm. (DIN size A4), or 21.6 cm. by 27.9 cm. (8 1/2 by 11 inches).

• Color drawings: In order to submit color drawings, an applicant must petition the Office. The

petition must include: o A fee o 3 sets of color drawings o An amendment to the specification that inserts language directing the

reader to the fact that color photos or drawings exist and that there are copies filed with the Office.

• Example of drawings from US Patent D282096:

Page 27: TOL Patent Bar Outline

Patent Bar Outline © theOtherLives

* Important to know for the patent bar. 27

o Photographs

• Black and White: Where drawings are incapable of accurately depicting the invention, black and

white photos or micrographs may be used in place of final drawings. Photos must be developed on paper that is DIN size A4 or 8.5 by 11 inches and

they must meet the margin requirements. • Color:

Color photos will be accepted if the requirements for both black and white photos and color drawings have been met.

• Example of photograph from US Patent PP8414:

Page 28: TOL Patent Bar Outline

Patent Bar Outline © theOtherLives

* Important to know for the patent bar. 28

o New Drawing

• All drawings submitted after the original application filing must be marked as “Replacement Sheet” or “New Sheet.”

o Correction of Drawings • If a drawing containing new matter is added after the beginning of prosecution, it will not

be entered. • A “correct drawing” is one that includes corrections of informalities and changes

approved by the examiner. • An “informal drawing” is one that does not comply with the drawing requirements, and the

examiner may require corrections to make it acceptable. o Cancelation of Figures

• If a drawing figure is cancelled during prosecution, a replacement sheet must be submitted. Where the canceled figure is the only figure in the drawing, applicant should submit a marked-up copy of the drawing with annotation stating the cancelation.

Tables o If a lengthy table is submitted on a CD or in e-form, the table will NOT be published as part of the

patent document. Rather, it will be published on the sequence homepage of the USPTO web site and the patent document will include a standardized “lengthy table” statement, directing the reviewer to the location of the table.

o Tables will be assumed to be presented in portrait view unless they are indicated as landscape. Models, Exhibits, Specimens

o These are generally not accepted by the Office unless they are requested by the Office. o For biotech applications, specimens are often required to be submitted so that they are available

for examination/use by the public. New Matter

o Matter not in original specification, claims or drawings • Any matter added to claims or drawings that was not adequately disclosed in the original

specification will be considered new matter, and such claims or drawings will be rejected. • New matter cannot be added through amendment. • Remember that the only way to add new matter after the filing date is through a

Continuation-in-Part (CIP) application. • Where an examiner requires cancelation of new matter that is entered as an amendment

to the specification, an applicant may petition for review of such decision (not appeal).* • Where an examiner rejects new matter as affecting claims, the applicant may appeal (not

petition).* o New matter by preliminary amendment

• A preliminary amendment that is present on the date of filing is considered part of the original disclosure.

It is the applicant’s responsibility to ensure that a supplemental oath or declaration is filed supporting preliminary amendment submitted on the date of

Page 29: TOL Patent Bar Outline

Patent Bar Outline © theOtherLives

* Important to know for the patent bar. 29

filing if it contains new matter (matter not presented in the specification and drawings of the application). This may be done later.

• A preliminary amendment submitted after the date of filing will NOT be considered part of the original disclosure, and new matter will be restricted as such.

609: Information Disclosure Statement General

o In order to be considered, an IDS must fully comply with both the timing requirements of 37 CFR § 1.97 and the content requirements of 37 CFR § 1.98.

o IDSs are not permitted in provisional applications. o For non-provisional applications, applicants and other individuals substantially involved in the

application have a duty to submit information to the Office that is material to patentability. • This duty to disclose applies at the time that information is known/discovered and cannot

be postponed. • Typists, accountants, and other persons not substantially involved in the preparation or

prosecution of the application do not have such a duty. o Third parties may not submit IDSs. They are only allowed to submit patents and publications (see

MPEP 1134). o Submission of an IDS by the applicant does not serve as a representation that a patentability

search has been performed. Information Disclosure Statements (ADS) in Continued Examinations or Continuing Applications

o When filing a continuing application under 35 USC § 120 that claims benefit to a parent application (other than an international application) a new IDS listing prior art cited by the examiner in examination of the parent is not required.

o Such an IDS will be required for a continuing application under 35 USC § 120 claiming the benefit to a parent international application.

o The examiner will consider information that has been considered by the Office in a parent application when examining a continuation, a divisional or a CIP application filed under 37 CFR § 1.53(b).

• However, if the IDS has not yet been considered in the parent at the time the offspring application is filed, the IDS must be resubmitted.

Content Requirements for an IDS o Patents listed in an IDS must be identified by:

• Patentee • Patent number • Issue date

o Publications listed in an IDS must be identified by: • Author • Title • Relevant pages • Date and place of publication

o If a non-English reference is submitted, the applicant shall include a copy of the translation if an English-language translation is available.

• If an English translation is not available, the Office will accept a concise explanation of the relevance to the information submitted in the application.

• If the concise explanation is part of the specification, the IDS listing should include the page or line numbers where the concise explanation is located in the specification.

Timing Requirements for an IDS * o There are three time periods where an IDS will be considered by the examiner, and there are

different requirements for submission during each period: (1) Within 3 months of filing or before the first Office Action

There are no additional requirements; these will always be accepted.

(2) After the time stated in (1) but before the final Office action or Notice of Allowance

Requires accompaniment of 37 CFR § 1.97(e) statement, OR the 37 CFR § 1.17(p) fee

(3) After the final Office action or Notice of Allowance but before payment of the issue fee.

Requires 1) 37 CFR § 1.97(e) statement, 2) petition, AND 3) a petition fee.

o Generally, the timing for submitting an IDS cannot be extended. o IDSs submitted after payment of the issue fee will be placed in the file jacket but will never be

considered. • The only option at this point for an applicant to have the IDS considered is to withdraw

the application from issue and file a continuing application (or a CPA in the case of design application). The IDS will automatically be reconsidered in the continuing or CPA application.

Page 30: TOL Patent Bar Outline

Patent Bar Outline © theOtherLives

* Important to know for the patent bar. 30

o If a bono fide attempt to submit a compliant IDS is made but part of the requirement is omitted, the applicant may be allowed time to correct the error.

Non-complying IDS o IDSs that do not comply with 37 CFR § 1.97 (timing requirements) or 37 CFR § 1.98 (content

requirements) will be placed in a file but not considered by the Office, and a record of the noncompliance will be sent back to the applicant.

o The requirements of §§ 1.97 and 1.98 do not apply to information provided by the applicant in response to an Office action.

Page 31: TOL Patent Bar Outline

Patent Bar Outline © theOtherLives

* Important to know for the patent bar. 31

CHAPTER 700: EXAMINATION OF APPLICATIONS 701: Statutory

The main conditions precedent to the grant of a patent to an applicant are set forth in 35 USC § 101, 102 and 103.

702: Requisites of the Application

The application is first reviewed for informalities (such as lacking MPEP 600 requirements) and for sufficiency of disclosure. If the application is ready for its first Office Action but it is then discovered to be impractical to give a complete action on the merits because of an informal or insufficient disclosure, the following procedure may be followed:

o A reasonable search should be made of the invention so far as it can be understood from the dis-closure, objects of invention, claims and any apparently pertinent art cited. In the rare case in which the disclosure is so incomprehensible as to preclude a reasonable search, the Office action will inform applicant that no search was made;

o Informalities noted by the Office of Initial Patent Examination (OIPE) and deficiencies in the drawing should be pointed out by means of attachments to the Office action;

o A requirement should be made that the specification be revised to conform to idiomatic English and US patent practice;

o The claims should be rejected as failing to define the invention in the manner required by 35 USC § 112 if they are informal. A blanket rejection is usually sufficient.

704: Search and Requirements for Information

Search o After reading the specification and claims and fully understanding the invention, the examiner

searches the prior art. Requirements for Information – 37 CFR § 1.105

o An examiner may require from individuals identified under 37 CFR § 1.56(c), or any assignee, the submission of such information as may be reasonably necessary to properly examine or treat a matter in a pending or abandoned application filed under 35 USC § 111, in a pending or abandoned application that has entered the national stage under 35 USC § 371, in a patent, or in a reexamination proceeding.

o Information required may be that which is reasonably necessary to properly examine an application. Such a request must have a reasonable basis and the scope must be narrowly defined.

o Where the factual information requested related to the subject application, and details thereof, applicant would be expected to make a reasonable inquiry under the circumstances to find the factual information requested.

o Applicant need not, however, derive or independently discover a fact, such as by experimentation, in response to a requirement for information.

o A requirement for information may be made before, with, or after an Office action on the merits. Replies to a Requirement for Information

o Replies to requirements for information must be complete and filed within the time period set including any extensions.

o Failure to reply within the time period set will result in the abandonment of the application. All replies for a request for information should be checked for completeness.

o Any incomplete reply can be completed within the original time period set including any extensions.

o Supplemental replies filed after the expiration of the original period for reply including any extensions of time must comply with all other rules for submissions of information.

o A reply that indicates the information requested is unknown may constitute a complete reply.* There is no requirement to perform a search for information.

• There is no requirement for the applicant to show that the required information was not, in fact, readily attainable, but applicant is required to make a good faith attempt to obtain the information and to make a reasonable inquiry once the information is requested.

o The duty of candor and good faith under 37 § CFR 1.56 applies to the applicant’s reply to a requirement for information under 37 CFR § 1.105, and requires that the applicant reply to a requirement under 37 CFR § 1.105 with information reasonably and readily available.

Time Periods for Reply o A requirement sent with an Office action on the merits will be allowed the same period for reply as

that set for the Office action.

Page 32: TOL Patent Bar Outline

Patent Bar Outline © theOtherLives

* Important to know for the patent bar. 32

o A requirement sent separately from an Office action will be given a shortened statutory period of two months for reply. Applicant may extend this period up to six months.

706: Rejection of Claims – 37 CFR § 1.104

Introduction o Once the examiner performs a prior art search, s/he will analyze the application in conjunction

with the state of the prior art to determine whether the claims define a useful, novel, nonobvious, and enabled invention that is clearly described in the specification.

• The examiner will articulate any applicable rejection(s) and/or objection(s) as clearly as possible.

• All requirements for patentability must be met before a claim will be allowed. o Remember that “rejections” are made based on the merits of a claim, and “objections” are made

based on form. Rejections are appealable, and objections are petitionable.* 35 USC § 102: Conditions for patentability; novelty and loss of right to patent – KNOW THIS STATUTE

o A person shall be entitled to a patent unless – • 102(a) the invention was known or used by others in this country, or patented or

described in a printed publication in this or a foreign country, before the invention thereof by the applicant for patent, or

• 102(b) the invention was patented or described in a printed publication in this or a foreign country or in public use or on sale in this country, more than one year prior to the date of the application for patent in the United States, or

• 102(c) he has abandoned the invention, or • 102(d) the invention was first patented or caused to be patented, or was the subject of an

inventor's certificate, by the applicant or his legal representatives or assigns in a foreign country prior to the date of the application for patent in this country on an application for patent or inventor's certificate filed more than twelve months before the filing of the application in the United States, or

• 102(e) the invention was described in - (1) an application for patent, published under section 122(b), by another filed in the United States before the invention by the applicant for patent or (2) a patent granted on an application for patent by another filed in the United States before the invention by the applicant for patent, except that an international application filed under the treaty defined in section 351(a) shall have the effects for the purposes of this subsection of an application filed in the United States only if the international application designated the United States and was published under Article 21(2) of such treaty in the English language; or

• 102(f) he did not himself invent the subject matter sought to be patented, or • 102(g)(1) during the course of an interference conducted under section 135 or section

291, another inventor involved therein establishes, to the extent permitted in section 104, that before such person's invention thereof the invention was made by such other inventor and not abandoned, suppressed, or concealed, or 102(g)(2) before such person's invention thereof, the invention was made in this country by another inventor who had not abandoned, suppressed, or concealed it. In determining priority of invention under this subsection, there shall be considered not only the respective dates of conception and reduction to practice of the invention, but also the reasonable diligence of one who was first to conceive and last to reduce to practice, from a time prior to conception by the other.

35 USC § 103 Conditions for patentability; non-obvious subject matter – KNOW THIS STATUTE o 103(a) A patent may not be obtained though the invention is not identically disclosed or described

as set forth in § 102 of this title, if the differences between the subject matter sought to be patented and the prior art are such that the subject matter as a whole would have been obvious at the time the invention was made to a person having ordinary skill in the art to which said subject matter pertains. Patentability shall not be negatived by the manner in which the invention was made.

o 103(b) • (1) Notwithstanding subsection (a), and upon timely election by the applicant for patent to

proceed under this subsection, a biotechnological process using or resulting in a composition of matter that is novel under section 102 and nonobvious under subsection (a) of this section shall be considered nonobvious if-

(A) claims to the process and the composition of matter are contained in either the same application for patent or in separate applications having the same effective filing date; and

Page 33: TOL Patent Bar Outline

Patent Bar Outline © theOtherLives

* Important to know for the patent bar. 33

(B) the composition of matter, and the process at the time it was invented, were owned by the same person or subject to an obligation of assignment to the same person.

• (2) A patent issued on a process under paragraph (1)- (A) shall also contain the claims to the composition of matter used in or made by

that process, or (B) shall, if such composition of matter is claimed in another patent, be set to

expire on the same date as such other patent, notwithstanding section 154. • (3) For purposes of paragraph (1), the term "biotechnological process" means-

(A) a process of genetically altering or otherwise inducing a single- or multi-celled organism to-

o (i) express an exogenous nucleotide sequence, o (ii) inhibit, eliminate, augment, or alter expression of an endogenous

nucleotide sequence, or o (iii) express a specific physiological characteristic not naturally

associated with said organism; (B) cell fusion procedures yielding a cell line that expresses a specific protein,

such as a monoclonal antibody; and (C) a method of using a product produced by a process defined by subparagraph

(A) or (B), or a combination of subparagraphs (A) and (B). o 103(c)

• (1) Subject matter developed by another person, which qualifies as prior art only under one or more of subsections (e), (f), and (g) of section 102 of this title, shall not preclude patentability under this section where the subject matter and the claimed invention were, at the time the claimed invention was made, owned by the same person or subject to an obligation of assignment to the same person.

• (2) For purposes of this subsection, subject matter developed by another person and a claimed invention shall be deemed to have been owned by the same person or subject to an obligation of assignment to the same person if –

(A) the claimed invention was made by or on behalf of parties to a joint research agreement that was in effect on or before the date the claimed invention was made;

(B) the claimed invention was made as a result of activities undertaken within the scope of the joint research agreement; and

(C) the application for patent for the claimed invention discloses or is amended to disclose the names of the parties to the joint research agreement.

• (3) For purposes of paragraph (2), the term "joint research agreement" means a written contract, grant, or cooperative agreement entered into by two or more persons or entities for the performance of experimental, developmental, or research work in the field of the claimed invention.

Rejection on Prior Art – 37 CFR § 1.104 o Prior art rejections based on anticipation are made under 35 USC § 102, and prior art rejections

based on obviousness are made under 35 USC § 103. • For anticipation under §102, the reference must teach every aspect of the claimed

invention either explicitly or impliedly, and any feature not directly taught must be inherently present.

• For an obviousness rejection under §103, a reference must be modified – or several references combined – in order to meet the claims. The modification must be one that would have been obvious to one of ordinary skill in the art at the time the invention was made.

Remember that this applies to the time the invention was made, not at the time of examination.

o Reliance on Certain Types of Prior Art as Basis for Rejection • Abstracts and Foreign Language Documents

A document published in a foreign language may be relied upon only if an English version/translation is obtained.

Citation of and reliance upon an abstract without citation of and reliance upon the underlying scientific document is generally inappropriate where both the abstract and the underlying document are prior art.

In limited circumstances, it may be appropriate for the examiner to make a rejection in a non-final Office action based in whole or in part on the abstract only without relying on the full text document. In such circumstances, the full text

Page 34: TOL Patent Bar Outline

Patent Bar Outline © theOtherLives

* Important to know for the patent bar. 34

document and a translation (if not in English) may be supplied in the next Office action.

• Admitted Prior Art Any statement that an applicant makes in the specification or during prosecution

identifying other work as prior art may be used to support an anticipation or an obviousness rejection, even where the prior art may not have been supportive of the rejection if discovered by the examiner independently.

Determining the Effective Filing Date of the Application o If the application is a continuation or a divisional of one or more earlier US or international

applications and if the requirements for the original filing have been satisfied, the effective filing date is the same as the earliest filing date in the line of continuation or divisional applications.

o If the application is a CIP of an earlier US or international application, any claims supported by the original specification will have the original filing date, and any claims supported only by the new matter will have the filing date of the CIP.

o The US application date of an application claiming foreign priority is the effective filing date. However, the foreign filing date may be used to overcome certain references.

o If the application properly claims the benefit of a provisional application, the provisional application’s filing date will be the effective filing date.

o See section 1800, below, for rules for determining the effective filing date of a PCT application. Rejections Under § 102(a), (b) or (e); Printed Publication or Patent

o Once the examiner finds a piece of prior art, s/he must first determine the effective filing date of the application and compare that to the date of the reference.

o Keep in mind that a reference may qualify as prior art under more than one 102 section, and the reference date may vary under different sections. Therefore, always analyze the reference under the rules of each 102 section and don’t stop at the first one that applies.

o § 102(a) • For § 102(a) to apply, the reference must:

Have an earlier publication date than the effective filing date of the application, and

Must NOT be the applicant’s own work. o § 102(b)

• If the reference qualifies under § 102(b) and its reference or issue date is more than one year prior to the effective filing date of the application, the reference qualifies as prior art under § 102(b).

• Where the last day of the year from the reference or issue date falls on a weekend or Federal holiday, the publication is NOT statutory bar if the applicant files on the following business day.

• A magazine, for example, is effective as a printed publication under § 102(b) as of the date it reaches the addresses, NOT the date it is printed or placed in the mail.

o § 102(e) • In order for § 102(e) to apply, the inventive entity of the application must be different than

that of the reference. Remember that where there are multiple inventors, only one inventor need be

different for the inventive entities to differ. • Under § 102(e), an international filing date that is on or after 29 November 2000 is a US

filing date if: The international application designated the US, and It was published by the World Intellectual Property Organization (WIPO) under

the PCT in the English language. • If the above listed conditions are met for the international filing date, the date of reference

of prior art under § 102(e) may actually be earlier if priority or benefit to an earlier US was properly claimed.

• Publications of international applications filed before 29 November 2000 do not have a § 102(e) date at all (however, such publications are available as prior art under 35 USC 102(a) or (b) as of the publication date).

Overcoming a § 102 Rejection Based on a Printed Publication or Patent* o § 102(a): A rejection based on § 102(a) can be overcome by:

• Persuasively arguing that the claims are patentably distinguishable from the prior art; • Amending the claims to patentably distinguish them over the prior art; • Filing an affidavit or declaration under 37 CFR § 1.131 showing prior invention (“swearing

back”), if the reference is not a US patent or a US patent application publication claiming the same patentable invention;

Page 35: TOL Patent Bar Outline

Patent Bar Outline © theOtherLives

* Important to know for the patent bar. 35

• Filing an affidavit or declaration under 37 CFR § 1.132 showing that the referenced invention is not by “another;”

• Perfecting a claim to priority under 35 USC § 119(a)-(d) within the allowable time period or filing a grantable petition. The foreign priority filing date must antedate the reference and be perfected;

• Perfecting benefit under § 119(e) or § 120 within the allowable time (or by petition if outside the allowable time), by amending the specification of the application to contain a specific reference to a prior application or by filing an ADS containing a specific reference to a prior application satisfying the enablement and written description requirements of § 112, first paragraph.

o § 102(b): A rejection based on § 102(b) can be overcome by: • Persuasively arguing that the claims are patentably distinguishable from the prior art; • Amending the claims to patentably distinguish over the prior art; • Perfecting benefit under 35 USC § 120 within the time period granted or petitioning for

amendment of the specification of the application to contain a specific reference to a prior application or by filing an ADS containing a specific reference to a prior application that satisfies the enablement and written description requirements of § 112, first paragraph;

• Perfecting benefit claim under § 119(e). o § 102(e): A rejection based on § 102(e) can be overcome by:

• Persuasively arguing that the claims are patentably distinguishable from the prior art; • Amending the claims to patentably distinguish over the prior art; • Filing an affidavit or declaration under 37 CFR § 1.132 showing that the reference

invention is not by “another;” • Filing an affidavit or declaration under 37 CFR § 1.131 showing prior invention (“swearing

back”), if the reference is not a US patent or a US patent application publication claiming the same patentable invention;

• Perfecting a claim to priority under 35 USC § 119(a)-(d) within the allowable time period or filing a grantable petition. The foreign priority filing date must antedate the reference and be perfected;

• Perfecting benefit under § 119(e) or § 120, as described above for overcoming a § 102(a) rejection.

o See summary chart for quick reference to these issues at the end of this section. Rejections Under 35 USC § 102(a) or (b); Knowledge by Others or Public Use or Sale

o Evidence of knowledge of others or public use or sale may come from admissions by the applicant or by personal knowledge of the examiner.

o The language “in this country” means in the US and does NOT include WTO or NAFTA member countries.

o If the activity being used as prior art is by someone other than the inventor, both § 102(a) and (b) may apply.

• However, no rejection under § 102(a) should be made if there is evidence that the applicant only disclosed the invention to another in less than one year prior to the application filing date.

o If there is not enough information on which to base a public use or on sale rejection, the examiner may make a requirement for more information.

Rejections Under 35 USC § 102(c) o Under § 102(c), abandonment of the invention results in loss of right to a patent.

Rejections Under 35 USC § 102(d) o If the following four conditions are present, there will be a statutory bar against the granting of a

patent in this country: • The foreign application must be filed more than 12 months before the effective filing date

of the US application. • The foreign and US applications must be filed by the same applicant or, his/her legal

representatives or assigns. • The foreign application must have actually issued as a patent or inventor’s certificate

before the US filing date. It need not be published, but the patent rights granted must be enforceable.

• The same invention must be involved. Rejections Under 35 USC § 102(e)

o To apply a reference under § 102(e), the inventive entity must be different. o The following are required of § 102(e) references:

• The potential reference must be a US patent, a US patent application publication or a WIPO publication of an international application.

Page 36: TOL Patent Bar Outline

Patent Bar Outline © theOtherLives

* Important to know for the patent bar. 36

• If the potential reference resulted from, or claimed the benefit of, an international application, the international filing date is the effective US filing date for § 102(e) purposes if:

The international application meets the following three conditions: o An international filing date on or after 29 November 2000; o Designated the US; and o Published under PCT Article 21(2) in English.

o For a concise flowchart on determining § 102(e) reference dates, see MPEP Chapter 700, page 40.

o Provisional Rejections Under § 102(e); Reference is a Copending US Patent Application • If an earlier filed, copending, and unpublished US patent application discloses subject

matter which would anticipate the claims in a later filed pending US application having a different inventive entity, a provisional § 102(e) rejection of the later filed application may be made.

• A provisional rejection is appropriate where if the earlier filed application were published or issued as a patent, it would become actual prior art.

• A provisional rejection of the later filed application should be made if: At least one common inventor exists between the applications or the applications

are commonly assigned, and The effective filing dates are different.

• A provisional rejection under § 102(e) can be overcome by the same methods as overcoming a regular § 102(e) rejection.

o Copending Applications Having No Common Inventor or Assignee • If there is no common assignee or common inventor and the application was not

published pursuant to 35 USC § 122(b), the confidential status of applications under 35 USC § 122(a) must be maintained and no rejection can be made relying on the earlier filed, unpublished application, or subject matter not supported in a redacted application publication, as prior art under § 102(e).

• If the filing dates of the applications are within 6 months of each other, an interference may be proper.

Rejections Under 35 USC § 102(f) o § 102(f) bars the issuance of a patent where an applicant did not invent the subject matter being

claimed and sought to be patented. Rejections Under 35 USC § 102(g)

o § 102(g) bars the issuance of a patent where another made the invention in the US before the applicant and had not abandoned, suppressed or concealed it.

o Applying a reference under § 102(g) may form the basis for an interference. Contents of a 35 USC § 103 Rejection

o An office action rejecting a claim for obviousness under § 103 will contain: • The relevant teachings of the prior art relied upon; • The difference or differences in the claim over the applied reference(s); • The proposed modification of the applied reference(s) necessary to arrive at the claimed

subject matter; and • An explanation as to why the claimed invention would have been obvious to one of

ordinary skill in the art at the time the invention was made. Provisional Rejection Under § 103

o § 103(c): Relating to all patents granted on or after 10 December 2004, the subject matter developed by someone other than the present applicant will be treated as owned by the same person or subject to an obligation of assignment to the same person for purposes of determining obviousness if three conditions are met:

• The claimed invention was made by or on behalf of parties to a joint research agreement that was in effect on or before the date the claimed invention was made;

• The claimed invention was made as a result of activities undertaken within the scope of the joint research agreement; and

• The application for patent for the claimed invention discloses or is amended to disclose the names of the parties to the joint research agreement.

o Provisional rejections of the obviousness type under § 103 based on provisional prior art under § 102(e) are rejections applied to copending applications having different effective filing dates where each application has a common assignee or a common inventor.

o Where two applications of different inventive entities are co-pending, not published, and the filing dates differ, a provisional rejection under § 103 based on provisional prior art under § 102(e)

Page 37: TOL Patent Bar Outline

Patent Bar Outline © theOtherLives

* Important to know for the patent bar. 37

should be made in the later filed application unless the application has been excluded under § 103(c), as described above.

• An actual rejection may be made if the first patent then issues. o The § 103 rejection based on § 102(e) prior art can be overcome by:

• Arguing patentability over the earlier filed application; • Combining the subject matter of the copending applications into a single application

claiming benefit under 25 USC § 120 of the prior applications and abandoning the copending applications;

• Filing an affidavit or declaration under 37 CFR § 1.132 showing that any unclaimed invention disclosed in the copending application was derived from the inventor of the other application and is thus not an invention “by another;”

• Filing an affidavit or declaration under 37 CFR § 1.131 showing a date of invention prior to the effective US filing date of the copending application; or

• For an application that is pending on or after 10 December 2004, a showing that: The prior art and the claimed invention were, at the time the invention was made,

owned by the same person or subject to an obligation of assignment to the same person; or

The subject matter is disqualified under § 103(c). o Where the applications are claiming the same patentable invention, a terminal disclaimer and an

affidavit or declaration under 37 CFR § 1.130 may be used to overcome a rejection under § 103 in a common ownership situation if the earlier filed application has been published or matured into a patent.

o For instructive examples, see MPEP Chapter 700, § 706.02(k). o Be sure to know: In order to be disqualified as prior art under § 103(c), the subject matter that

would otherwise be prior art to the claimed invention and the claimed invention must be commonly owned, or subject to an obligation of assignment to the same person, at the time that the invention was made.

o Where prior art is disqualified under §103(c), a double patenting rejection may be proper if there is subject matter claimed in the new application that would be given extended protection if awarded.

Biotechnology Process Applications o Rejections under § 103(b) apply only to claims of biotechnology processes. o § 103(b) requires that:

• The biotechnological process and composition of matter be contained in either the same application or in separate applications having the same effective filing date;

• Both the biotechnological process and composition of matter be owned or subject to an assignment to the same person at the time the process was invented;

• A patent issued on the process also contain the claims to the composition of matter used in or made by the process, or, if the process and composition of matter are in different patents, the patents expire on the same date;

• The biotechnological process falls within the definition set forth in 35 USC 103(b) – see above; and

• A timely election be made to proceed under the provisions of 35 USC 103(b), which is made by petition.

Rejections NOT Based on Prior Art o Rejections Under 35 USC § 101

• Subject Matter Eligibility: § 101 permits patents to be granted only for “any new and useful process, machine, manufacture, composition of matter, or material.”

The term “process” means process, art or method, and includes a new use of a known process, machine, manufacture, composition of matter or material.

Examples of subject matter NOT patentable under § 101: o Printed matter – such as the mere arrangement of printed matter. o Naturally occurring article – a thing in nature that is substantially

unaltered, like a shrimp with its head and digestive tract removed. o Scientific Principle – a scientific principle divorced from any tangible

structure. • Utility: A rejection may be made under § 101 for lack of utility.

o Rejections Under 35 USC § 112 - Discussed in Section 2100, below. o Duplicate Claims

• Courts have decided that an applicant may restate his claims in a reasonable number of ways. However, claims that are clearly duplicative may result in one copy being objected to as a copy of the allowed claim.

Page 38: TOL Patent Bar Outline

Patent Bar Outline © theOtherLives

* Important to know for the patent bar. 38

o New Matter • No new matter can be added to the specification during prosecution. Therefore, in the

case of amendments, any claim submitted that that is supported only by new matter introduced through amendment will be rejected (as will the new matter).

• If subject matter capable of illustration is originally claimed and it is not shown in the drawing, the claim will not be rejected, but the applicant will be required to add it to the drawing.

o Disclaimer • Claims may be rejected on the ground that the applicant has disclaimed the subject

matter involved, such as a disclaimer arising from an applicant’s failure to make claims suggested in an interference proceeding.

o Reissue • Claims in a reissue application that enlarge the scope of the original patent will be

rejected. • Additionally, a defective reissue oath affords a ground for rejecting all the claims in the

reissue application. Final Rejections

o When Proper on Second Action • The second or any subsequent actions on the merits will be final except where the

examiner introduces a new ground of rejection that is neither necessitated by amendments to the claims nor based on information submitted in an IDS.

• A second or subsequent action on the merits should NOT be final if it includes a rejection based on prior art not of record of any claim amended to include limitations that should reasonably have been expected to be claimed.

o When Proper on First Action • Claims may be finally rejection in the first Office action of a continuing application where:

The new application is a continuing application of, or a substitute for, an earlier application, and

All claims of the new application: o Are drawn to the same invention claimed in the earlier application, and o Would have been properly finally rejected on the grounds and art of

record in the next Office action if they had been entered in the earlier application.

• The claims for an application for which an RCE has been filed may be finally rejected in the action immediately subsequent to the RCE filing.

• A final rejection in a first Office action is NOT proper in a continuing or substitute application or an RCE where that application contains material that was presented in the earlier application after final rejection or closing of prosecution but was denied entry because:

New issues were raised that required further consideration and/or search, or The issue of new matter was raised.

• A final first action would NOT be proper in a CIP application where any claim includes subject matter not present in the earlier application.

o Time for Reply to Final Rejections • A reply to the final rejection is due 3 months from its mailing date.

o Avoiding Abandonment Due to Final Rejection • In order to avoid abandonment due to a final rejection one of the following actions must

be taken: A Notice of Appeal may be filed. A response may be filed within 3 months that places the claim in condition for

allowance. An RCE may be filed as long as it includes a bona fide attempt at advancing

examination. A response may be filed within 3 months contending the appropriateness of the

final action, and if the examiner agrees, s/he may convert it to a non-final action. Request for Continued Examination (RCE) Practice – 37 CFR § 1.114

o § 1.114 Request for continued examination. • If prosecution in an application is closed, an applicant may request continued

examination of the application by filing a submission and the fee set forth in § 1.17(e) prior to the earliest of:

Payment of the issue fee, unless a petition under § 1.313 is granted; Abandonment of the application; or

Page 39: TOL Patent Bar Outline

Patent Bar Outline © theOtherLives

* Important to know for the patent bar. 39

The filing of a notice of appeal to the US Court of Appeals for the Federal Circuit under 35 USC § 141, or the commencement of a civil action under 35 § USC 145 or 146, unless the appeal or civil action is terminated.

• “Prosecution in an application is closed” as used in this section means that the application is under appeal, or that the last Office action is a final action (§ 1.113), a notice of allowance (§ 1.311), or an action that otherwise closes prosecution in the application.

• A submission as used in this section includes, but is not limited to, an information disclosure statement, an amendment to the written description, claims, or drawings, new arguments, or new evidence in support of patentability. If reply to an Office action under 35 USC § 132 is outstanding, the submission must meet the reply requirements of § 1.111.

• If an applicant timely files a submission and fee set forth in § 1.17(e), the Office will withdraw the finality of any Office action and the submission will be entered and considered. If an applicant files a request for continued examination under this section after appeal, but prior to a decision on the appeal, it will be treated as a request to withdraw the appeal and to reopen prosecution of the application before the examiner. An appeal brief (§ 41.37 of this title) or a reply brief (§ 41.41 of this title), or related papers, will not be considered a submission under this section.

• The provisions of this section do not apply to: A provisional application; An application for a utility or plant patent filed under 35 USC § 111(a) before

June 8, 1995; An international application filed under 35 USC § 363 before June 8, 1995; An application for a design patent; or A patent under reexamination.

o An applicant may file an RCE to request continued examination after prosecution is closed. o An RCE is properly filed after:

• Receiving a final Office action; • A Notice of Appeal has issued; • An Office action under Ex Parte Quayle has occurred; • An application has been taken under appeal by the BPAI.

o An RCE is NOT properly filed in applications after: • It has been abandoned • The issue fee has been paid • The applicant has filed a Notice of Appeal to the Federal Circuit and it is still active.

o An RCE cannot be filed for: • Design applications • Provisional applications • Reexamination patents

o RCE submission requirements • Fee • Request for RCE, including a bona fide attempt at advancing the application, such as

new arguments or evidence in support of patentability or an IDS. o Other RCE details

• An applicant in a utility application originally filed on or after June 8, 1995 and before May 29, 2000 may obtain further examination either by timely filing an RCE, a proper submission and a requisite fee, or by timely filing a CPA.

• An improper RCE will not toll the running of any time period set in the previous Office action for reply to avoid abandonment of the application.

708: Order of Examination

Special cases o Generally, applications are examined in the order received. However, there are various “special”

cases where an application is reviewed out of turn, including reissues and applications that are “made special” through petition.

Petition to Make Special – See 37 CFR § 1.102, Advancement of Examination o Generally, new applications are reviewed in the order received. However, there are

circumstances under which an applicant can qualify for expedited review through submission of a petition to “make special.”

o A petition to make special must be filed by an applicant, assignee or an attorney/agent registered to practice before the office.

Page 40: TOL Patent Bar Outline

Patent Bar Outline © theOtherLives

* Important to know for the patent bar. 40

o Circumstances qualifying application to be “made special”: • Prospective Manufacture: Where the manufacturer is prepared to produce the

prospectively patented invention and is obligated to do so upon issuance. • Infringement: Where there is an infringing product or method in use and the claims are

unquestionably infringed. • Health: Where an applicant’s health will prevent him from assisting in the prosecution. • Age: Where the applicant is 65 or older. • Accelerated Examination of New Applications: Where all of the claims are directed to the

same invention and where the applicant does a pre-examination search and submits to the examiner a copy of each reference accompanied by a detailed discussion of the reference.

• Invention is of High Importance to Mankind where it: Directly impacts environmental quality Contributes to the discovery, utilization or development of energy Involves superconductivity materials Relates to HIV/AIDS or cancer Involves recombinant DNA Is involved in countering terrorism Is filed by a qualifying small entity and relates to biotech, and:

o The subject of the patent application is a major asset, and o The development of the technology will be significantly impaired if

examination of the patent application is delayed. o Any non-reissue utility or design application filed on or after August, 25 2006 is eligible for

accelerated examination, except: • Plant applications • Applications entering the national stage from an international application • Reexamination proceedings • RCEs (unless the application was previously granted special status under the program)

o Generally, a fee is required to accompany a petition to make special. A fee is NOT required where the subject of the petition is due to:

• Illness/health or age of applicant • Environmental contributions • Enhancement of energy resources • Countering terrorism

o All petitions to make special require a statement of the type of research involved, and other evidence may be required (such as a birth certificate) to establish qualification.

709: Suspension of Action – 37 CFR § 1.103

Suspension of action should not be confused with extension of time for reply. Suspension of action applies to an impending Office action by the examiner, whereas an extension of time for reply applies to action by the applicant.

A suspension of action will cause a reduction in patent term adjustment, if any, and the reduction is equal to the number of days beginning on the date a request for suspension of action was filed and ending on the date of the termination of the suspension.

An action cannot be suspended in an application that contains an outstanding Office action or a request awaiting a reply by the applicant.

Request, 37 CFR Section Requirement Max. Length of Suspension

1.103(a) Petition with a showing of good and sufficient cause; a fee 6 months 1.103(b) Request at the time of filing a CPA; a fee 3 months 1.103(c) Request at the time of filing an RCE, a fee 3 months 1.103(d) Request for deferral of examination 3 years from earliest filing date for which

a benefit is claimed under Title 35 710: Period for Reply

See: o 35 USC 133. Time for prosecuting application o 37 CFR 1.135. Abandonment for failure to reply within time period

Statutory Period

Page 41: TOL Patent Bar Outline

Patent Bar Outline © theOtherLives

* Important to know for the patent bar. 41

o The maximum statutory period for reply to an Office action is 6 months, but shortened periods (not less than 30 days) are used in almost all cases as time limits for reply given by examiners in Office actions.

o Alternatively to giving a shortened statutory period for reply, an examiner might require response before a specific date, called a time limit action.

Shortened Statutory Periods (SSPs) o The length of the shortened statutory period to be used depends on the type of reply required. o Failure to reply with a SSP results in abandonment of the application.

• Abandonment of an application is not appealable, but a petition to revive may be granted if the delay was unavoidable or unintentional.

• Further, extensions to the SSP may be obtained provided the extension does not extended beyond the 6-month statutory period from the date of the Office action.

o 1-Month SSPs Apply to: • Requirements for restriction or election of species only (no action on the merits). • When a reply by an applicant for a non-final Office action is bona fide but includes an

inadvertent omission that must be corrected. o 2-Month SSPs Apply to:

• The winning party in an interference to reply to an unanswered Office action. • An applicant who needs to reply to an Ex Parte Quayle Office action. • A multiplicity rejection where there are no other rejections.

o 3-Month SSPs Apply to: Any Office action on the merits. SSP versus a Specified Time Limit

o Where an examiner specifies a time limit for reply, failure to take action within the specified time results in loss of rights in regard to the particular matter. However, a failure to reply within a SSP results in abandonment of the entire application.

o A specified time limit for reply may be less than the 30-day minimum set for SSPs. o Where an applicant replies a day or two after a specified time limit for reply, the examiner may

excuse the delay if s/he finds that it is excusable. However, any delay in responding to an Office action with a SSP – even one day – results in abandonment of the application.

Extensions of Time o 37 CFR § 1.136 provides for two distinct procedures to extend the period for action or reply in

particular situations. o While the SSP may be extended within the limits of the statutory 6 months period, no extension

can operate to extend the time beyond the 6 months. o § 1.136(a): Automatic Extensions

• An automatic extension permits an applicant to file a petition for extension of time and a fee up to 5 months after the end of the time period set to take action except:

Where prohibited by statute; Where prohibited by one of the items listed in the rule; or Where applicant has been notified otherwise in an Office action.

• The petition and fee must be filed within the requested extended time period for reply and the fee MUST be paid or the extension will not apply and the application will be abandoned.

o § 1.136(b): Extensions for Cause • Provides for requests for extensions of time upon a showing of sufficient cause when the

procedure of § 1.136(a) is not available. • A petition for cause under § 1.136(b) must be filed during the set period for reply. • A request for extension under §1.136(b) must state a reason in support of the request.

o Extensions of time with payment under § 1.136(a) are possible in reply to most Office actions, except:

• All extensions in a reexamination proceeding; • All extensions during an interference proceeding; • Those specified situations where an Office action states that the provisions of § 1.136(a)

are not applicable; • Those limited instances where the applicant is given a specified time limit to take certain

actions. o Fees for extensions of time qualify for 50% reduction for applicants with small entity status. o An extension is not possible after payment of the issue fee. o For non-statutory actions – such as appeals and a notice to file missing parts – an automatic

extension may be obtained for up to 5 months beyond the reply period, even if that makes the total period for reply more than 6 months (the statutory maximum for reply).

Period Ending on a Weekend or Holiday

Page 42: TOL Patent Bar Outline

Patent Bar Outline © theOtherLives

* Important to know for the patent bar. 42

o Where the last day for taking any action or paying any fee in the USPTO falls on a weekend or Federal holiday, the action may be taken or fee may be paid on the next succeeding secular or business day.

o When a Federal holiday falls on a Saturday, the preceding Friday is considered to be a Federal holiday and the PTO will be closed.

Situations When Reply Period is Reset or Restarted o Where an Office action contains an error that affects the ability of the applicant to reply to the

action, the applicant can call the error to the attention of the examiner within one month and request that the period set for reply be restarted from the date the error is corrected.

o If the error is called to the examiner’s attention after one month but within the period for reply, the applicant can request that a new deadline be set substantially equal to the time remaining in the period for reply.

o Any error brought to the Office’s attention after the period for reply will not be restarted and any appropriate extension fees will apply.

o Petitions to Reset Due to Late Receipt - The PTO will grant a petition asking to restart the period for reply to begin from the date of receipt of the Office action at the correspondence address when the petition is filed within 2 weeks of the date of receipt of the PTO action at the correspondence address.

• The petition must include evidence of the address, a copy of the envelope with the date received, and a statement.

o Petitions to Reset Due to Postmark Dated Later than the Date on the Office Action – The PTO will grant a petition asking to restart the period for reply to run from the postmark date shown on an Office action when the petition is filed within 2 weeks of the date of receipt of the OA at the correspondence address and the reply period was for payment of the Issue Fee, or the reply period set was 1 month or 30 days, and the petition includes evidence of the address, a copy of the envelope with the date received, and a statement.

711: Abandonment and Revival of Patent Application

Abandonment of Patent Application o An abandoned application loses all patent rights and is one that is removed from the Office’s

docket of pending applications through: • Formal abandonment

By the applicant (acquiesced in by the assignee, if one exists); By the attorney or agent of record; or By a registered attorney or agent acting in a representative capacity when filing a

continuing application • Failure of applicant to take appropriate action within a specified time at some stage in the

prosecution of the application. o A letter of abandonment properly signed becomes effective when an appropriate Office official

takes action thereon. o An allowed application will not be abandoned unless:

• There is a mistake on the part of the Office; • There is an illegality in the application; • The invention is unpatentable; • There is an interference; or • It is abandoned in order to permit the consideration of an IDS in a continuing application.

o An allowed application may only be abandoned by filing a form for express abandonment and a petition to withdraw from issue under 37 CFR § 1.313, including payment of a fee.

o Note: Abandonment may result from a situation where the applicant’s reply is within the reply period but is not fully responsive to the Office action.

Termination of Proceedings o Proceedings are terminated when:

• The issue fee is not paid; • The application loses an interference regarding all the claims it contained; • The Board determines the proceedings are closed; or • The court determines the proceedings are closed.

Revival of Patent Application o Petition to Withdraw Holding of Abandonment Based on Failure to Receive Office Action:

May include allegations that an Office action was never received and must show that abandonment was either unintentional or unavoidable.

o Treatment of Untimely Petition to Withdraw Holding of Abandonment

Page 43: TOL Patent Bar Outline

Patent Bar Outline © theOtherLives

* Important to know for the patent bar. 43

• A petition to withdraw holding of abandonment not filed within two months of the action complained of may be dismissed as untimely. Alternatively, the Office may require a terminal disclaimer as a condition of granting an untimely petition to withdraw the holding of abandonment.

o Petition to Revive an Abandoned Application or Accept Late Payment of Issue Fee • Under specific circumstances, an abandoned application may be revived if the

abandonment was caused by unavoidable or unintentional delay. Intentionally abandoned applications cannot be revived.

• Applications that are successfully revived may require terminal disclaimers. • A petition to revive must be filed within one year of abandonment or the Office will require

evidence as to the delay in discovering the abandonment. • 37 CFR § 1.137 provides for the revival of abandoned applications and lapsed patents for

the failure: To timely reply to an Office requirement in a provisional application; To timely prosecute in a non-provisional application; To timely pay the issue fee for a design application; To timely pay the issue fee for a utility or plant application; and To timely pay any outstanding balance of the issue fee (lapsed patents).

• Petitions for Unavoidable delay: An unavoidable delay may be proved by showing:

o The error was the cause of the delay at issue; o There was in place a business routine for performing the clerical function

that could reasonably be relied upon to avoid errors in its performance; and

o An employee was sufficiently trained and experienced with regard to the function and routine for its performance that reliance upon such employee represented the exercise of due care.

A petition under 37 CFR § 1.137(a) requires: o The required reply, unless previously filed; o The petition fee; o A showing to the satisfaction of the Director of the USPTO that the entire

delay in filing the required reply from the due date for the reply until the filing of a grantable petition pursuant to 37 CFR § 1.137(a) was unavoidable; and

o Any terminal disclaimer required pursuant to 37 CFR § 1.137(d). • Petitions for Unintentional delay:

An acceptable unintentional delay may NOT be proved by intentional actions, such as:

o The applicant does not consider the claims to be patentable over the references relied upon in an outstanding Office action;

o The applicant does not consider the allowed or patentable claims to be of sufficient breadth or scope to justify the financial expense of obtaining a patent;

o The applicant does not consider any patent to be of sufficient value to justify the financial expense of obtaining the patent;

o The applicant does not consider any patent to be of sufficient value to maintain an interest in obtaining the patent; or

o The applicant remains interested in eventually obtaining a patent but simply seeks to defer patent fees and patent prosecution expenses.

A petition under 37 CFR § 1.137(b) requires: o The required reply, unless previously filed; o The petition fee; o A statement that the entire delay in filing the required reply from the due

date for the reply until the filing of a grantable petition pursuant to 37 CFR § 1.137(b) was unintentional; and

o Any terminal disclaimer required pursuant to 37 CFR § 1.137(d). • Unavoidable Versus Unintentional Delay

If the delay is unintentional, the fee is generally large, but only a statement as to the reason for delay is required.

If the delay is unavoidable, the fee is generally small, but proof that the delay was unavoidable is required.

Terminal Disclaimers (TDs)

Page 44: TOL Patent Bar Outline

Patent Bar Outline © theOtherLives

* Important to know for the patent bar. 44

o See: 37 CFR § 1.137 o A terminal disclaimer is required for a utility or plant application filed on or after June 8, 1995, but

before May 29, 2000, where the application became abandoned during appeal, interference, or while under a secrecy order.

o TDs occur when part of the patent term must be dedicated to the public, such as to avoid double patenting.

o TDs under 37 CFR § 1.137(a) or (b) are used in: • Design applications; • Non-provisional, utility, or plant applications filed before 8 June 1995; • Utility or plant apps filed on or after June 8, 1995, but before May 29, 2000 where the

application became abandoned during appeal, interference or while under secrecy order (these are eligible for Uruguay Round Agreements Act – URAA).

o Utility or plant applications filed on or after May 29, 2000 do not require a TD since the period of abandonment is now reduced from the Patent Term Adjustment provisions (see MPEP Chapter 2700, below).

Request for Reconsideration o A request for the reconsideration or review of a decision refusing to revive an abandoned

application or lapsed patent must be filed within 2 months of the decision refusing to revive the application and may be extended under 37 CFR § 1.137 unless otherwise stated.

713: Interviews

An interview is defined as any personal appearance of an applicant, attorney, or agent before the examiner or a telephone conversation or video conference or email between such parties presenting matters for the examiner’s consideration.

Interviews Prior to First Official Action o Request for interviews are generally granted prior to the first Office action in continuing or

substitute applications. o In all other applications, requests are granted when the examiner determines that such an

interview would advance prosecution. Recordation

o The substance of any interview must be made part of the application. o It is the responsibility of the applicant or his attorney to make sure the substance of an interview

is made of record, except where the interview was initiated by the examiner. The examiner should always verify accuracy.

Prohibited Interviews or Granted, Special Situations o Except in unusual situations, no interview is permitted after the brief on appeal is filed or after an

application has been passed to issue. o Interviews normally should not be granted unless the requesting party has authority to bind the

principal concerned. Finally Rejected Applications

o One interview is generally permitted after final rejection. o A second or further interview may be held if the examiner believes it will expedite prosecution.

714: Amendments, Applicant’s Action

See: 37 CFR § 1.121. Manner of making amendments in application When Applicant May Amend:

o Before or after the first examination and action and also after the second or subsequent examination or reconsideration as specified in 37 CFR § 1.112 (Reconsideration before final action);

o After final rejection, if the amendment meets the criteria of 37 CFR § 1.116 (Amendments and affidavits or other evidence after final action and prior to appeal);

o After the date of filing a notice of appeal if the amendment meets the criteria of 37 CFR § 41.33 (Amendments and affidavits or other evidence after appeal); and

o When and as specifically required by the examiner. New Matter: may not be added by amendment.

o New matter includes*: • Matter that is wholly unsupported by the original disclosure; • Adding specific percentages or compounds when the original disclosure was broader;

and • The omission of a step from a method.

o The only time that new matter may be added to a patent application is through the filing of a CIP application.

Page 45: TOL Patent Bar Outline

Patent Bar Outline © theOtherLives

* Important to know for the patent bar. 45

Amendments to Claims o Each amendment document that includes a change to an existing claim, including the deletion of

an existing claim, or submission of a new claim, must include a complete listing of all claims ever presented (including previously canceled and non-entered claims) in the application.

o After each claim number, the status identifier of the claim must be presented in a parenthetical expression, and the text of each claim under examination as well as all withdrawn claims (each with markings if any, to show current changes) must be presented.

o The listing will serve to replace all prior versions of the claims in the application. o Status Identifiers: The current status of all claims must be given using one of the following

identifiers: (original), (currently amended), (previously presented), (canceled), (withdrawn), (new), or (not entered).

• The status identifier (withdrawn – currently amended) is also acceptable for a withdrawn claim that is being currently amended.

• Claims added by preliminary amendment must always have the identifier (new) instead of (original).

• The identifier (not entered) is used for claims that were previously proposed in an amendment that was denied entry.

o Markings to Show the Change: All claims being currently amended must be presented with markings to indicate the changes that have been made relative to the immediate prior version.

• Use strikethrough for deleted matter (strikethrough) and underlining for added matter (underlining) with two exceptions:

For deletion of five or fewer consecutive characters, double brackets may be used (e.g. [[eroor]]);

If strikethrough cannot be easily perceived, double brackets must be used. • Any full claims added by amendment must be identified as (new) and must not be

underlined. o Claim Text: The text of all pending claims under examination and withdrawn claims must be

submitted each time any claim is amended. o Claim Numbering: All of the claims in each amendment paper must be presented in ascending

numerical order, and consecutive canceled or not entered claims may be aggregated into one statement (e.g., Claims 1-5 (canceled)).

• A canceled claim can only be reinstated by subsequent amendment, adding it as a new claim and giving it a new, consecutive number.

• The original numbering of claims must be preserved throughout the entire document, and they will be renumbered when the patent issues.

Amendments to Drawings o Amendments to drawings must be submitted on a replacement sheet of drawings, even if no

substantive changes are being made, and it must be labeled as a “Replacement Sheet.” o A new drawing must be submitted on a new drawing sheet and be labeled as “New Sheet.” o A marked-up copy of the original drawing may be included and labeled as “Annotated Sheet.”

However, a clean amended version must also be submitted. Examiner’s Amendments*

o Prosecution may be expedited through amendments to the specification made by the examiner. o If a non-compliant amendment would otherwise place the application in condition for allowance,

the examiner may enter the non-compliant amendment and provide an examiner’s amendment to correct the non-compliance (such as an incorrect status identifier).

Signatures to Amendments o An amendment must be signed by a person having authority to prosecute the application. o An unsigned or improperly signed amendment will not be entered. Where the application is not

under final rejection, the applicant will be given one month to correct this defect. Amendments Before First Office Action (Preliminary Amendments)*

o See: 37 CFR § 1.115. Preliminary amendments. o For applications filed on or after 21 September 2004, a preliminary amendment that is present on

the filing date of the application is part of the original disclosure of the application. For those filed before 21 September 2004, it is only considered part of the original disclosure if the amendment is referred to in the first executed oath or declaration filed with the application.

o Any preliminary amendment must comply with 37 CFR § 1.121 (Manner of making amendments in applications).

o Any amendment filed after the mailing date of the first Office action is NOT a preliminary amendment.

o A preliminary amendment will be disapproved by the Director if it cancels all of the claims in the application without presenting any new or substitute claims.

Page 46: TOL Patent Bar Outline

Patent Bar Outline © theOtherLives

* Important to know for the patent bar. 46

o Once the examiner has started to prepare a first Office action, entry of a preliminary amendment may be disapproved if the preliminary amendment unduly interferes with the preparation of the first Office action.

o Entry of a preliminary amendment will NOT be disapproved for undue delay of the Office action if it is filed no later than:

• 3 months from the filing date of the application under 37 CFR § 1.53(b); • 3 months from the date the national stage is entered as set forth in 37 CFR § 1.491

(National stage commencement and entry) in an international application; • The filing date of a CPA under 37 CFR § 1.53(d) in a design application; or • The last day of any suspension period requested by applicant under 37 CFR § 1.103

(Suspension of action by the Office). Supplemental Amendment

o See: 37 CFR § 1.111. Reply by applicant or patent owner to a non-final Office action. o A supplemental reply may be necessary where the applicant does not include a complete, fully

responsive reply to an outstanding Office action the first time. o Supplemental replies are not entered as a matter of right, except in limited cases such as where

the applicant is requesting suspension of action when filing an RCE. o The Office may enter a supplemental reply if the reply is clearly limited to:

• Cancellation of a claim; • Adoption of the examiner’s suggestions; • Placement of the application in condition of allowance; • Reply to an Office requirement made after the first reply was filed; • Correction of informalities (e.g., typographical errors); or • Simplification of issues for appeal.

Amendments Made After Final Action or Appeal o Once the final rejection has been entered, the applicant no longer has the right to further

prosecution. At this point, an amendment that will place the application in condition for allowance or in better form for an appeal may be entered at the discretion of the examiner.

o Amendments filed after the final rejection may be kept on file, but these are not considered a matter of right.

o The reply requirements of 37 CFR § 1.116(b) require that the reply must tie up loose ends by amendment.

o Replies presenting the argument in a more defensible light that adds additional claims will NOT be allowed.

o Replies amending claims into process claims will NOT be allowed. o Replies amending all individual claims will NOT be allowed.

Amendments After Notice of Allowance (NoA) o These amendments are not entered as a matter of right but rather at the Office’s discretion; any

amendments considered necessary by the applicant should be completed before the NoA is issued.

o Amendments must be filed on or before the date the Issue fee is paid, never after. o If the applicant plans to send an amendment after the NoA has already been issued the applicant

must also establish: • Why the amendment(s) were not presented earlier; • Why the amendment(s) is needed; • Why no new search or consideration is required; and • Why any amendment or new claims are patentable.

715: Swearing Back of Reference – Affidavit or Declaration Under 37 CFR § 1.131* See: 37 CFR § 1.131. Affidavit or declaration of prior invention. “Swearing back” occurs when the applicant claims that the invention date was actually before the date of

reference, usually after receiving a rejection based on prior art. Where evidence is submitted to traverse a rejection but does not qualify under § 1.131, it may be

submitted in an affidavit or declaration under § 1.132 (Affidavits or declarations traversing rejections or objections).

Swearing back under § 1.131 may be used to overcome rejections where the prior art used to establish the rejection was not by “another” but was filed by the same inventor and is most commonly used to overcome 35 USC § 102(a) or (e) and § 103 rejections.

Examples of When Affidavits or Declarations can be Used under § 1.131 o To antedate a reference or activity that qualifies as prior art under 35 USC § 102(a) and not under

§ 102(b), but if the prior art reference under § 102(a) is a US patent (or application publication) the reference cannot be antedated if it claims the same patentable invention.

Page 47: TOL Patent Bar Outline

Patent Bar Outline © theOtherLives

* Important to know for the patent bar. 47

o To antedate prior art under § 102(e) where the reference has a prior art date under § 102(e) prior to the applicant’s effective filing date, and shows but does not claim the same patentable invention.

Affidavits or Declarations under § 1.131 are inappropriate where the prior art is statutory bar, such under § 102(b), (d), (f) or (g).

Swearing back is also inappropriate where the reference is a prior US patent to the same entity, claiming the same invention, as this involves “double patenting.”

The Date to Overcome is the effective date of the reference as prior art. o Domestic Patents and Applications are available as of their publication date.

• The effective date of a domestic patent when used as a reference is not the foreign filing date to which the application for patent may have been entitled under 35 USC § 119(a) during examination. The US filing date is the date to overcome.

o Foreign Patents: The date that a foreign patent is effective as a reference is usually the date patent rights are formally awarded to its applicant.

o Printed Publications are available as prior art as of its publication date, NOT as of the date it was received by the publisher.

o Activities: An admission by the applicant or knowledge or use of the invention by others in this country is available as prior art from the date the activity was first known to have occurred.

Where Reference is Joint to Applicant and Another o When subject matter disclosed but not claimed in a patent or application publication filed jointly by

applicant and another is claimed in a later application filed by applicant, the joint patent or application publication is a valid reference under 35 USC § 102(a) or (e) unless:

• Overcome by affidavit or declaration under 37 CFR § 1.131; or • An unequivocal declaration under 37 CFR § 1.132 by applicant that s/he conceived or

invented the subject matter disclosed in the patent or application publication and relied on in the rejection.

Where Reference and Application Have Common Assignee o An affidavit or declaration under § 1.131 must be filed even when the reference disclosing the

invention and the application claiming the invention are subject to common ownership in the absence of a showing under § 1.132 that the patentee derived the subject matter relied on from the applicant.

Where Reference is Publication of Applicant’s Own Invention o Unless it is a statutory bar, a rejection based on a publication may be overcome by a showing

that it was published either by applicant himself/herself or on his/her behalf. Use § 1.132 affidavit or declaration since the invention doesn’t need to antedate the reference.

Activities Applied Against the Claims o Unless the activity is statutory bar, § 1.131 may be used to establish date of invention prior to the

invention being used or known. o Alternatively and where applicable, the applicant can use § 1.132 to declare that the activity was

done by the applicant. General Rule as to Generic Claims

o A reference or activity applied against generic claims may (in most cases) be antedated as to such claims by an affidavit or declaration under 37 CFR § 1.131 showing completion of the invention of only a single species, within the genus, prior to the effective date of the reference or activity (assuming, of course, that the reference or activity is not a statutory bar or a patent, or an application publication, claiming the same invention).

When Interference is Necessary o Where the reference is a US patent or application publication claiming the same invention as the

applicant and the publication date is less than one year prior to the applicant’s presentation of claims, the applicant’s remedy is NOT by way of § 1.131 but rather through interference.

Who May Make an Affidavit or Declaration o The following parties may make an affidavit or declaration under § 1.131:

• All inventors of the subject matter claimed; • Less than all named inventors of an application only where it is shown that less than all

named inventors of an application invented the subject matter of the claim or claims under rejection;

• A legal representative where they have been granted authority; • The assignee or other party in interest when it is not possible to produce the affidavit or

declaration of the inventor. Requirements of a § 1.131 Showing

o Affidavit or declaration in compliance with § 1.131; and o Evidence showing facts (not conclusions) of the invention’s date (such as a lab notebook entry).

Page 48: TOL Patent Bar Outline

Patent Bar Outline © theOtherLives

* Important to know for the patent bar. 48

• Secondary considerations may also be presented, such as unexpected results, commercial success, long-felt but unresolved need, failure of others or skepticism of experts.

Three Ways to Show Prior Invention o There are three ways in which an applicant can establish prior invention of the claimed subject

matter. The showing of facts must prove: • Actual reduction to practice of the invention prior to the effective date of the reference;

Generally, an existing, working model is required, but photos may suffice for very simple inventions.

• Conception of the invention prior to the effective date of the reference coupled with due diligence from prior to the reference date to a subsequent (actual) reduction to practice; or

• Conception of the invention prior to the effective date of the reference coupled with due diligence from prior to the reference date to the filing date of the application (constructive reduction to practice).

716: Affidavits or Declarations Traversing Rejections, 37 CFR § 1.132*

General Criteria Applicable to All Evidence for Traversing Rejections o Timeliness: Evidence traversing rejections is considered timely if submitted:

• Prior to a final rejection; • Before appeal in an application not having a final rejection; • After final rejection, but before or on the same date of filing an appeal, upon a showing of

good and sufficient reasons why the affidavit or other evidence is necessary and was not earlier presented in compliance with 37 CFR § 1.116(e) (Amendments and affidavits or other evidence after final action and prior to appeal); or

• After prosecution is closed if applicant files the affidavit or other evidence with a request for RCE or a CPA.

o Consideration of Evidence • The examiner will review all evidence submitted and provide a detailed explanation of

his/her findings in the next Office action. o Objective Evidence of Nonobviousness

• Evidence timely submitted must be considered, even “evidence rising out of the so-called ‘secondary considerations’ must always when present be considered en route to a determination of obviousness.”

• Examiners must consider comparative data in the specification, which is intended to illustrate the claimed invention in reaching a conclusion with regard to the obviousness of the claims.

• The lack of objective evidence of nonobviousness does not weigh in favor of obviousness.

• However, where a prima facie case of obviousness is established, the failure of applicant to provide rebuttal evidence is dispositive.

o Other Considerations • Any secondary evidence must be related to the claimed invention. • Any objective evidence should be supported by actual proof. • Attorney arguments cannot take the place of evidence. • Although factual evidence is preferable to opinion testimony, such testimony is entitled to

consideration and some weight so long as the opinion is not on the ultimate legal conclusion at issue.

• There is no requirement that a publication must describe something that has actually been reduced to practice before the publication can act as a prior art reference.

• Prior art can include subject matter described in a printed publication in a foreign country before the filing date of the application.

Rules for Secondary Considerations o Unexpected Results

• Where the applicant alleges unexpected results, s/he MUST show unexpected results to be evidence of nonobviousness.

Greater than expected results are evidence of nonobviousness. Superiority of a property shared with the prior art is evidence of nonobviousness. Presence of an unexpected property is evidence of nonobviousness. Absence of an expected property is evidence of nonobviousness. The burden is always on the applicant to show that the results are significant

through presenting and explaining the data.

Page 49: TOL Patent Bar Outline

Patent Bar Outline © theOtherLives

* Important to know for the patent bar. 49

Expected beneficial results are evidence of obviousness. Showing unexpected results over one of two equally close prior art references

will not rebut prima facie obviousness unless the teachings of the prior art references are sufficiently similar to each other that the testing of one showing unexpected results would provide the same information as to the other.

To establish unexpected results over a claimed range, applicants should compare a sufficient number of tests both inside and outside the claimed range to show the criticality of the claimed range.

o Commercial Success • There must be sufficient evidence between the claimed invention and evidence of

commercial success presented by the applicant, and the evidence of success must be commensurate in scope with the claimed invention.

• Further, the commercial success must be derived from the claimed invention and must flow from the functions and advantages disclosed or inherent in the specification description.

o Long-Felt Need and Failure of Others • Evidence presented to support this secondary consideration must show that the claimed

invention satisfies a long-felt need that was recognized, persistent, and not solved by others.

• The long-felt need is measured from the date a problem is identified and efforts are made to solve it.

• Other factors contributing to the presence of a long-felt need must be considered, such as a lack of interest or appreciation of an invention’s potential or marketability rather than want of technical know-how.

o Skepticism of Experts • It has been held that disbelief by experts can constitute strong evidence of

nonobviousness. 718: Affidavit or Declaration to Disqualify Commonly Owned Patent or Published Application as Prior Art, 37 CFR § 1.130

See: 37 CFR § 1.130. Affidavit or declaration to disqualify commonly owned patent or published application as prior art.

37 CFR § 1.130(a) may be used when: o The rejection in an application or patent under reexamination is a rejection under 35 USC § 103

in view or a US patent application or publication which is not prior art under 35 USC § 102(b). o The inventions defined by the claims in the application or patent under reexamination and by the

claims in the US patent are not identical, but are not patentably distinct, and are owned by the same party.

37 CFR § 1.130(b) states that: A double patenting rejection will be made when the claims in the reexamination and the prior art in question are not patentably distinct from one another and the same invention is claimed, but this may be resolved by adding a terminal disclaimer.

The phrase “prior art inventor under 35 USC § 104” requires that the inventor named in the application or patent be the prior inventor within the meaning of 35 USC § 104 in that an applicant or patent owner may not:

o Establish a date of invention in a foreign country other than a NAFTA or WTO member country; o Establish a date of invention in a WTO member country other than a NAFTA country earlier than

January 1, 1996; or o Establish a date of invention in a NAFTA country other than the US earlier than December 8,

1993. 724: Trade Secret, Proprietary, and Protective Order Materials

Any information that is material to patentability must be submitted to the Office, even if the information is desired to be kept as trade secret or otherwise confidential. Such submissions may be edited to remove confidential information so long as all information material to patentability is included (but not necessarily that which is merely favorable to patentability).

Information that is considered by the party submitting the same to be either trade secret material or proprietary material, and any material subject to a protective order, must be clearly labeled as such and be filed in a sealed, clearly labeled, envelope or container.

Each document or item must be clearly labeled as a “Trade Secret” document or item, a “Proprietary” document or item, or as an item or document “Subject To Protective Order.”

Page 50: TOL Patent Bar Outline

Patent Bar Outline © theOtherLives

* Important to know for the patent bar. 50

Table Quick Guide to §102

§ 102 Sub § Person Relevant Action Date of Significance Relevant

Location How to Overcome the Rejection

Third-party knew of or used invention before the date of claimed invention USA • Prove that the disclosure was derived

from the applicant’s own work.

a. Third-party

patented/described invention in printed

publication

before the date of claimed invention

anywhere in the world

• Argue distinguishability over prior art. • Amend the claims to be

distinguishable over prior art. • File an affidavit or declaration under

37 CFR § 1.131. or 37 CFR § 1.132. • Perfect a claim to priority under 35

USC § 119(a)-(d). • Perfect benefit under § 119(e) or §

120.

Inventor or Third-party

patented/described invention in printed

publication

more than 1 year before the filing date of the patent

application

anywhere in the world

• Argue distinguishability over prior art. • Amend the claims to be

distinguishable over prior art. • Perfect benefit under § 119(e) or §

120. b.

Inventor or Third-party

offered for sale, sold, or publicly used invention

more than 1 year before the filing date of the patent

application USA

• Provide sufficient evidence of experimental use.

c. Inventor abandons invention anytime USA • Prove due diligence from conception to application filing.

d. Inventor patents invention more than one year prior

to the US application filing date

foreign country

• Show the foreign application was filed less than 12 months prior

• Show the foreign and US applications were filed by separate applicants

• Show the foreign application was not issued prior to the US filing date

• Show that a different invention was involved.

e. Third-party

describes invention in patent application that

ultimately issues or PCT application is published in

English

files before the date of the claimed invention

anywhere in the world

• Argue distinguishability over prior art • Amend the claims to be

distinguishable over prior art • File an affidavit or declaration under

37 CFR § 1.131 or 37 CFR § 1.132 • Perfect a claim to priority under 35

USC § 119(a)-(d) • Perfect benefit under § 119(e) or §

120

f. Inventor derived invention from a third-party

before the date of the claimed invention

anywhere in the world

• Prove that the applicant is responsible for the invention.

g. Third-party invented invention before the date of the claimed invention USA • Prove applicant was first to conceive

and diligently reduced it to practice = Third-party = Inventor or Third-party = Inventor

Page 51: TOL Patent Bar Outline

Patent Bar Outline © theOtherLives

* Important to know for the patent bar. 51

CHAPTER 800: RESTRICTION IN APPLICATIONS FILED UNDER 35 USC § 111; DOUBLE PATENTING 802: Basis for Practice in Statute and Rules

Meaning of “Independent” and “Distinct” o Independent: There is no disclosed relationship between the two or more inventions claimed.

The inventions are unconnected in design, operation or effect (not dependent on each other in any way).

o Distinct: There is a disclosed relationship between the two or more subjects in question, but they are capable of separate manufacture, use or sale as claimed, and are patentable over each other.

o Relationship: Two or more inventions are related if they are disclosed as connected by design, operation, or effects. Inventions that are not independent are related.

• Related inventions are distinct if the inventions as claimed are not connected in design, operation, or effect and when at least one invention is patentable over the other.

Definition of “Restriction” o Restriction is the practice of requiring an applicant to elect a single claimed invention for

examination when two or more independent inventions and/or two or more distinct inventions are claimed in an application.

Restriction Requirements * o If two or more independent and distinct inventions are claimed in a single application, the

examiner in his Office action shall require the applicant in his response to elect the invention to which the claim(s) shall be restricted.

o An applicant must then elect the invention to be reviewed in the existing application and may then file divisional application(s) for the invention(s) not elected.

o Where the applicant believes that the restriction is improper, s/he may petition to traverse the restriction. However, the applicant must still elect an invention to be reviewed, or else the petition will be considered non-responsive to the restriction requirement.

803: Restriction – When Proper*

Generally o Claims of a patent are properly restricted when they would support separate patents and are

either independent or distinct. o An examiner must give reasons and/or examples for the restriction requirement as a prima facie

showing of his burden but need not cite supporting documents. o An applicant may present evidence to rebut the examiner’s prime facie showing.

Markush Claims o A Markush claim recites alternatives in a defined group.

• Example: “…selected from the group consisting of A, B and C.” o Generally, the members of a defined Markush group belong to a recognized physical or chemical

class. However, a group may be chosen as long as there is at least one recognizable common feature among them.

o Unity of an invention exists where the compounds included within a Markush group: • Share a common utility; and • Share a substantial structural feature disclosed as being essential to that utility.

o In an application containing a Markush-type claim that encompasses at least two independent or distinct inventions, the examiner may require a provisional election of a single species prior to examination on the merits.

Transitional Applications o Generally

• An applicant may be allowed to have more than one distinct invention (or species) in a single application reviewed if the application has been pending for 3 three years or longer as of June 8, 1995.

• An applicant will not be permitted to have such additional invention examined if: The requirement was made in the application or in an earlier application relied on

under 35 USC §§ 120, 121, or 365(c) prior to April 8, 1995; No restriction requirement was made with respect to the invention(s) in the

application or earlier application due to actions by the applicant; or The required fee for examination of each additional invention was not paid.

o Linking Claim Allowable • When divided inventions in a transitional application are rejoined because a linking claim

is allowable (see MPEP 809), and the applicant paid the fee for the additional invention, the applicant may request a refund of that fee.

Page 52: TOL Patent Bar Outline

Patent Bar Outline © theOtherLives

* Important to know for the patent bar. 52

o Generic Claim Allowable • Where claims to multiple species were divided and then rejoined due to an allowable

generic claim, an applicant may request a refund for the fee paid for the previously divided species.

Nucleotide Sequences o Nucleotide sequences that encode different proteins are structurally distinct chemical

compounds. Generally, a maximum of 10 nucleotide sequences (depending on their lengths) will be allowed in an application without restriction.

o Applications claiming only a combination of nucleotide sequences will generally not be subject to a restriction requirement.

• The presence of one novel and nonobvious sequence within the combination will render the entire combination allowable.

• Even if a single sequence in the combination is not found to be novel and nonobvious, the combination may still be allowable.

o Applications containing only composition claims reciting different combinations of individual nucleotide sequences will be subject to a restriction requirement, and the applicant will be required to elect one combination.

804: Definition of Double Patenting*

Definition o Double patenting results when the right to exclude granted by a first patent is unjustly extended

by the grant of a later issued patent or patents. o This doctrine seeks to avoid unjustly extending patent rights at the expense of the public. o In order for there to be double patenting, there must be at least one common inventor and/or be

either commonly assigned/owned or non-commonly assigned/owned by subject to a joint research agreement.

o Two types of double patenting: • “Same Invention” – Based on 35 USC § 101, which states in the singular that an

inventor “may obtain a patent”. • “Non-Statutory Type” – Based on a judicially created doctrine primarily intended to

prevent prolongation of the patent term by prohibiting claims in a second patent not patentably distinguishable from claims in a first patent.

o “Domination”: Where one patent or application “dominates” a second patent or application when the first patent or application has a broad or generic claim which fully encompasses or reads on an invention defined in a narrower or more specific claim in another patent or application. Note that domination is different than double patenting.

o Instances where double patenting issue can be raised: • Between an issued patent and one or more applications: Where an inventor/owner has

already secured the issuance of the first patent, and an examiner must determine whether the grant of a second patent would give rise to an unjustified extension of the rights granted in the first patent.

• Between copending applications – Provisional rejections: “Provisional rejections” can be made between co-pending applications where the examiner determines that there will be a double patenting conflict when one of them issues. This allows the examiner and the applicant to address the potential problem without having to wait for one of the patents to issue.

Non-Statutory double patenting rejections: o Includes rejections based on either one- or two-way determination of

obviousness. o Could also include a rejection other than the usual “obviousness-type”

double patenting rejection (but this is rare). o Can be avoided by filing a terminal disclaimer in the application or

proceeding in which the rejection is made. o Obviousness type non-statutory double patenting rejection: Is present if

any claim in the application defines an invention that is merely an obvious variation of an invention claimed in a commonly owned patent.

o Two-way obviousness test is used when the applicant could not have filed the claims in a single application and there is an administrative delay.

o One-way obviousness test is appropriate when there is no administrative delay.

Statutory double patenting rejections:

Page 53: TOL Patent Bar Outline

Patent Bar Outline © theOtherLives

* Important to know for the patent bar. 53

o Prevents two patents from issuing on the same invention even if there is a terminal disclaimer.

o A terminal disclaimer cannot be used to obviate a statutory double patenting rejection.

• Between one or more applications and a published application – Provisional rejections: Applies to a published application and a new application by the same inventor (at

least one in common) or same assignee. Since the published application has not yet issued as a patent, the examiner is

permitted to make a “provisional” rejection on the ground of double patenting when the published application has not been abandoned and claims pending therein conflict with claims of the application being examined.

• Reexamination proceedings: A double patenting issue may raise a substantial new question of patentability of

a claim of a patent, and thus be addressed in a reexamination proceeding. Double patenting may exist where a reference patent or application and the

patent under reexamination are filed by at least one common inventor or owner/assignee.

o Requirements of a double patenting rejection (including provisional rejections) • Double patenting rejections are either statutory or non-statutory, depending on the

relationship of the inventions being claimed. • Generally, double patenting rejections are not appropriate in divisional applications

resulting from a restriction requirement made on a parent application. • Statutory double patenting – 35 USC § 101: Prevents the “same” invention from being

patented twice by the same inventive entity/owner. Question to ask is: Is the same invention being claimed twice? “Same invention” means identical subject matter.

Test: Whether a claim in the application could be literally infringed without literally infringing a corresponding claim in the patent. Is there an embodiment of the invention that falls within the scope of one claim but not the other?

Result: If there is such an embodiment, then identical subject matter is not defined by both claims and statutory double patenting would not exist.

• Non-statutory double patenting: Prevents unjust extension of patent rights granted by the first patent where the second patent sought does not necessarily claim the “same” invention but would extend the rights of the first patent.

Obviousness type: A non-statutory obviousness-type double patenting rejection is appropriate where the conflicting claims are not identical, but at least one examined application claim is not patentably distinct from the reference claim(s) because the examined application claim is either anticipated by, or would have been obvious over, the reference claim(s).

o The analysis employed in an obviousness-type double patenting determination parallels that used for a 35 USC § 103(a) rejection (see MPEP 2132), which are the following Graham factual inquiries*:

Determine the scope and content of a patent claim relative to a claim in the patent at issue;

Determine the differences between the scope and content of the patent claim as determined above and the claim in the application at issue;

Determine the level of ordinary skill in the pertinent art; and Evaluate any objective indicia of nonobviousness.

One-way obviousness: If the application at issue is the later filed application or both are filed on the same day, only a one-way determination of obviousness is needed in resolving the issue of double patenting.

Two-way obviousness: This is used when the later filed application is the patent. A two-way test is to be applied only when the applicant could not have filed the claims in a single application.

o In a two-way obviousness determination, it is necessary to apply the Graham factors twice – once with the application claims as the claims in issue and once with the patent claims as the claims in issue.

o An obviousness-type double patenting rejection will be appropriate only where each analysis compels a conclusion that the invention defined in the claims in issue is an obvious variation of the invention defined in the claim in the other patent/application.

o Contrast between double patenting rejection and rejections based on prior art

Page 54: TOL Patent Bar Outline

Patent Bar Outline © theOtherLives

* Important to know for the patent bar. 54

• A double patenting rejection must rely on a comparison with the claims in an issued or to-be-issued patent, whereas an anticipation or obviousness rejection based on the same patent under 35 USC § 102(e)/103(a) relies on a comparison with what is disclosed (whether or not claimed) in the same issued or to-be-issued patent.

• A terminal disclaimer cannot be used to obviate a rejection based on 35 USC § 102(e)/103(a) prior art.

o Double patenting does not relate to international applications that have not yet entered the national stage.

Disclaimers o A disclaimer may be used to disclaim or dedicate part of the patent term. o An examiner may require a disclaimer if a particular claim is found to be the same as a claim in

another issued patent. o A disclaimer of a portion of a patent’s term is necessary to ensure that no claimed invention

receives longer than a set term. • A statutory disclaimer disclaims the whole term of one or more claims in a patent. • A terminal disclaimer (37 CFR § 1.321) gives up a terminal portion of the term.

Used for non-statutory type double patenting rejections. Not effective in overcoming statutory double patenting rejections. Terminal disclaimers operate with respect to all claims in a patent.

Avoiding a Double Patenting Rejection o Statutory: Can be avoided by amending the conflicting claims so that they are not coextensive in

scope. Remember that a terminal disclaimer cannot be used to obviate a statutory double patenting rejection. Further, a 37 CFR § 1.131 (Affidavit or declaration of prior invention) affidavit cannot overcome this type of rejection.

o Non-statutory: Can be avoided by filing a terminal disclaimer in the application or proceeding in which the rejection is made. The use of a 37 CFR § 1.131 affidavit in overcoming a double patenting rejection is inappropriate because the claim or claims in the application are being rejected over a patent that claims the rejected invention.

Commonly Owned Inventions of Different Inventive Entities; Non-Commonly Owned Inventions Subject to a Joint Research Agreement

o Remember that double patenting rejections apply to applications/patents where there is at least one inventor in common, as well as applications/patents that are commonly owned or subject to a joint research agreement.

805: Effect of Improper Joinder in Patent

No patent can be held void for the improper joinder of inventions claimed therein. 806: Determination of Distinctness or Independence of Claimed Inventions*

General principles relating to distinctness: o Where inventions are independent, restriction to one is ordinarily proper; o Where inventions are related as disclosed but are distinct as claimed, restriction may be proper; o Where inventions are related as disclosed but are not distinct as claimed, restriction is never

proper; and o A reasonable number of species may be claimed when there is an allowable claim generic

thereto. Single Embodiment, Claims Defining Same Essential Features

o A provisional election of a single species may be required where only generic claims are presented and the generic claims recite such a multiplicity of species that an unduly extensive and burdensome search is necessary.

Genus and/or Species Inventions o Generally, an application with claims directed to different embodiments or species that could fall

within the scope of a generic claim will be subject to restriction between species. However, where an allowable generic claim is included that links a reasonable number of species, a double patenting rejection may be improper.

o Species may be independent or related inventions • Where species under a claimed genus are not connected in any of design, operation or

effect under the disclosure, the species are independent inventions. o Definition of a generic claim

• Generic claims should require no material element additional to those required by the species claims, and each of the species claims must require all the limitations of the generic claim.

o Claims limited to species

Page 55: TOL Patent Bar Outline

Patent Bar Outline © theOtherLives

* Important to know for the patent bar. 55

• Claims are different inventions; therefore, claims can never be species. • The scope of a claim is limited to a single disclosed embodiment. However, a claim may

alternatively encompass two or more of the disclosed embodiments and thus be designated a generic or genus claim.

o Restriction between mutually exclusive species • If it can be shown that two or more inventions are independent, the applicant will be

required to restrict the presented claims to one of the independent inventions. Examples: Different modes of operation, different functions or different effects are

independent. If two inventions within an application consist of a process and an apparatus, and

if the apparatus cannot be used to practice the process or any part thereof, they are independent.

Where species under a genus are independent. Related Inventions

o Where two or more related inventions are claimed, it must be determined whether they are distinct. If so, restriction may be proper. If they are not distinct, restriction is never proper. If non-distinct inventions are claimed in more than one application or patent, double patenting must be held, except where the additional applications were filed in response to a restriction requirement (in which case, the Office has already determined they are NOT the same invention).

o Combination and subcombination • A combination is an organization of subcombinations or elements.

o Subcombinations usable together • Two or more claimed subcombinations, disclosed as usable together in a single

combination, and which can be shown to be separately usable, are usually restrictable when the subcombinations do not overlap in scope and are not obvious variants.

o Process and Apparatus • A process and an apparatus may be shown to be distinct when:

The process as claimed may be practiced on another apparatus; The apparatus can be used to practice a materially different process; The apparatus is not the obvious apparatus for making the product and can be

used to make other products; The product claimed can be made by another materially different apparatus.

• A process of making and a product made by the process can be shown to be distinct inventions if the following can be shown:

That the process as claimed is not an obvious process of making the product and the process as claimed can be used to make other and different products; and

That the product as claimed can be made by another and materially different process.

• An apparatus and a product made by the apparatus can be shown to be distinct inventions if either or both of the following can be shown:

That the apparatus as claimed is not an obvious apparatus for making the product and the apparatus as claimed can be used to make other and different products; and

That the product as claimed can be made by another and materially different apparatus.

• A product and process of using the product can be shown to be distinct inventions if either or both of the following can be shown:

The process of using the product as claimed can be practiced with another materially different product; or

The product as claimed can be used in a materially different process. Independent Inventions

o Inventions as claimed are independent if there is no disclosed relationship between the inventions (if they are unconnected in design, operation and effect).

808: Reasons for Insisting Upon Restricting

Every requirement to restrict has two aspects: o The reasons why each invention as claimed is either independent or distinct from the others; and o The reasons why there would be a serious burden on the examiner if restriction is not required.

An election of species should not be required if the species claimed are considered clearly unpatentable (obvious) over each other.

Election of species should be required prior to a search on the merits: o In all applications containing claims to a plurality of species with no generic claims; and

Page 56: TOL Patent Bar Outline

Patent Bar Outline © theOtherLives

* Important to know for the patent bar. 56

o In all applications containing both species claims and generic or Markush claims.

809: Linking Claims Linking claims are those that enable joinder between two or more otherwise divisable claims. The most common types of linking claims which, if allowed, act to prevent restriction between inventions

that can otherwise be shown to be divisible, are: o Genus claims linking species claims; o A claim to the necessary process of making a product linking proper process and product claims; o A claim to “means” for practicing a process linking proper apparatus and process; and o A claim to the product linking a process of making and a use (process of using).

The applicant must be advised of the allowance of generic claim and be given a time limit of 1 month (not less than 30 days) to conform all of the claims to the nonelected species to fully embrace an allowed generic claim or the examiner will cancel the claims to each nonconforming species by an examiner’s amendment and pass the application to issue.

810: Action on the Merits

Generally, in an application when only a non-final written requirement to restrict is made, no action on the merits is given.

If an action is given on the merits, it must be given on all the claims. A one-month shortened statutory period will be set for reply when a restriction requirement is made

without action on the merits. 811: Time for Making a Requirement

Restriction is proper at any stage of prosecution up to the final action. A second requirement may be made when it becomes proper, even though there was a prior requirement with which the applicant complied.

812: Who Should Make the Requirement

The requirement should be made by an examiner who would examine at least one of the inventions. Telephone Restriction Practice

o When an examiner determines that a requirement for restriction should be made, the examiner should formulate a draft of the restriction and telephone the attorney or agent of record to request an oral election, with or without traverse.

o No telephone communication need to be made when the requirement is complex, the application is being prosecuted by the applicant pro se, or the examiner knows from past experience that an election will not be made by telephone.

818: Election and Reply

An applicant who receives a non-final Office action containing restriction requirements between two groups of claims should reply distinctly pointing out the detailed reasons why the applicant believes the examiner erred in requiring a restriction between the two groups and additionally state which claim/group/species to elect.

Election Fixed by Action on Claims: Election becomes fixed when the claims in an application have received an action on their merits by the Office.

Traversing the Rejection – 37 CFR § 1.143 o When the restriction or rejection is thought to be wrong and is objected to, the applicant must

attempt to “traverse” the rejection. In traversing, an applicant is still required to make an election or else the traverse will be considered non-responsive to the restriction/rejection.

o Filing a traverse: • Is not used in connection with responding to an official action; • Must set forth all objections to the restriction/rejection and not simply state a

disagreement; • Will retain the applicant’s right to petition after the examiner restricts the restriction

requirement.

Page 57: TOL Patent Bar Outline

Patent Bar Outline © theOtherLives

* Important to know for the patent bar. 57

CHAPTER 900: PRIOR ART, CLASSIFICATION AND SEARCH 901: Prior Art

Abandoned Applications o A published abandoned application is available as prior art as of its publication date.

Pending Applications o Pending applications that have not been published are generally preserved in confidence.

Canceled Matter o Any matter that is cancelled from the application file wrapper of a US patent will be considered

prior art as of the date of patent issuance (NOT the filing date) since the application will then be open to the public.

Co-pending Abandoned Applications o If the patent refers to and relies on the disclosure of a co-pending abandoned application, the

disclosure will be considered a prior art reference. o If a reference patent claims the benefit of a co-pending but abandoned application disclosing the

same subject matter as the patent, the abandoned application may be considered prior art. The reference date of the common subject matter is the filing date of the abandoned application.

Foreign Patents o In general, a foreign patent and the contents of its application should not be cited as a reference

until an examiner can confirm its date of patenting or publication by reviewing a copy of the document.

Page 58: TOL Patent Bar Outline

Patent Bar Outline © theOtherLives

* Important to know for the patent bar. 58

Page 59: TOL Patent Bar Outline

Patent Bar Outline © theOtherLives

* Important to know for the patent bar. 59

CHAPTER 1000: MATTERS DECIDED BY VARIOUS U.S.P.T.O. OFFICIALS 1002: Petitions to the Director

Petitions can generally not be made on appealable matters. Remember that petitions are made to the Director, and appeals are made to the Board of Patent Appeals and Interferences.

The mere filing of a petition will not stay the period for replying to an examiner’s action, nor act as a stay of other proceedings.

Any petition that is not filed “within 2 months from the action complained of may be dismissed as untimely.”

The 2-month time period in 37 CFR § 1.181(f) applies to petitions under any section that does not specify the time period within which a petition must be filed (i.e. 37 CFR § 1.182 and 1.183).*

Often, the action complained of (i.e. a requirement for a new drawing) is included in the same letter as an action on the merits of the claims, the latter having a 3-month period for reply.

o In this case, if an applicant requests reconsideration of the requirement for a new drawing, the examiner’s action on the request, if adverse, establishes the beginning of the 2-month period for filing the petition.

o The petition must be filed within this period even though the period for a reply to the rejection of the claims may extend beyond the 2-month period.

[Specific types of petitions reviewed by the Office Director, as well as specific technology center Directors are listed in MPEP 1002 and 1003.]

Page 60: TOL Patent Bar Outline

Patent Bar Outline © theOtherLives

* Important to know for the patent bar. 60

Page 61: TOL Patent Bar Outline

Patent Bar Outline © theOtherLives

* Important to know for the patent bar. 61

CHAPTER 1100: STATUTORY INVENTION REGISTRATION (SIR) AND PREGRANT PUBLICATION (PG PUB) 1101: Request for SIR

A SIR can be considered a hybrid of a patent and a publication, as no rights are bestowed on the applicant, but others are restricted from gaining rights to the invention.

o A published SIR will be treated the same as a US patent for defensive purposes. This tactic is often used by the federal government and rarely used by individuals or companies. A request for SIR (or a request to change a SIR) may be filed at the time of filing a non-provisional

application or later during the pendency of a non-provisional application. The fee depends on the time of filing the SIR request.

The application to be published must include: o Specification with claim(s); o Oath or declaration; and o Drawings (when necessary).

An SIR will be granted and published if the application: o Meets the requirements of 35 USC § 112 (specification); o Complies with the printing requirements; o Includes agreement from the inventor(s) to waive the right to receive a patent; and o Includes the proper fee.

1105: Review of Final Refusal to Publish SIR

An applicant who is dissatisfied with a final refusal to publish a SIR for reasons other than non-compliance with § 112 may file a petition for review with the appropriate Technology Center Director.

An applicant who is dissatisfied with a decision finally rejecting claims under § 112 may obtain review by filing an appeal with the BPAI.

1109: Withdrawal of SIR request

An applicant may file a request to withdraw a SIR with a non-provisional application containing a SIR request. If the examiner receives the withdrawal request before a Notice of Intent to Publish SIR has been mailed, the examiner will first ascertain whether there is an outstanding rejection under § 112.

o If so, a timely reply to the rejection will be required using a Response to Request to Withdraw Request for a SIR form.

o If the response is timely received, the request will normally be granted. ` Any request to withdraw an SIR filed after a Notice of Intent to Publish SIR has been sent (as of the

mailing date of the notice) must be in the form of a petition and include a fee. An original SIR application can be abandoned in favor of a continuing application for a patent, claiming

the filing date of the earlier filed application, by filing an express abandonment of the original application and a timely request or petition to withdraw the request for a SIR prior to publication of the SIR. See 37 CFR § 1.296 (Withdrawal of request for publication of SIR).

1111: SIR Publication and Effect

A published SIR will be treated the same as a US patent for all defensive purposes and will be usable as a reference (prior art) as of its filing date in the same manner as a patent.

An applicant’s waiver to patent rights of the invention is effective as of the date of publication of the SIR. The holder of a SIR will not be able to file a reissue application to recapture the rights, including the right

to exclude others from making, using, selling, offering to sell, or importing the invention, that were waived by the initial publication of the SIR.

1120: Eighteen-Month Publication of Patent Applications

Generally, non-provisional utility and plant applications filed on or after November 29, 2000 are published promptly after the expiration of a period of 18 months from the earliest filing date for which benefit is sought.

The Office will generally publish: o Utility and plant applications filed under 35 USC § 111(a) on or after November 29, 2000; and o Non-provisional applications having entered the national stage from an international application

filed on or after November 29, 2000. o The following are not published:

• Provisional applications • Design applications • Reissue applications

o Exceptions to automatic 18-month publication include applications that are: • No longer pending, as when:

Page 62: TOL Patent Bar Outline

Patent Bar Outline © theOtherLives

* Important to know for the patent bar. 62

The application has already issued as a patent. (The Office, however, will not stop the publication until the patent actually issues.)

The application is abandoned. • Subject to secrecy order. • Filed with a non-publication request.

o The Office will not publish an application until it includes: • The basic filing fee; • An English translation, if required; • An executed oath or declaration; • A specification; • An abstract; • Drawings (if necessary); and • A sequence listing (if necessary).

The publication date is 18 months from the earliest of the following: o The US filing date o The international filing date o The filing date of an earlier application if the application claims the benefit of the earlier

application. The Office always lists the projected publication date on the filing receipt for every complete patent

application. The Office generally starts preparing for the publication 4 months in advance of the projected publication

date. Therefore, communications received during this 4-month window may not be processed in time to affect the publication date.

1121: Content of a Patent Application Publication

The patent application publication is generally based on: o The patent application papers and drawings deposited on the filing date of the application; o The executed oath or declaration submitted to complete the application; o Any subsequently filed application papers and drawings submitted in reply to a preexamination

notice requiring: • A title and abstract in compliance with 37 CFR § 1.72; • Application papers in compliance with 37 CFR § 1.52; • Drawings in compliance with 37 CFR § 1.84; or • A sequence listing in compliance with 37 CFR §§ 1.821-1.825.

o The correspondence address for the application according to Office records at the time the publication process was initiated;

o Any amendments that expedite the publication process as long as they are submitted in time, including:

• Amendments to the specification that are reflected in a substitute specification; • An amendment to the abstract; • Amendments to the claims that are reflected in a complete claim listing; and • Amendments to drawings.

Appendices, other than those containing sequence listings or certain tables, are not printed if they are contained on pages located after the claims.

If the applicant would like the assignee information to be published, such information must be provided on either an ADS or on the application transmittal letter.

1122: Requests for Nonpublication*

Where an applicant does not intend his invention to be the subject of a foreign or international application or any other agreement that requires publication, s/he may request that the application not be published by filing a nonpublication request.

The request will be honored if it is in compliance with the following: o The request for nonpublication must be submitted with the application upon filing (this

requirement cannot be waived); o The request for nonpublication must state in a conspicuous manner that the application is not to

be published; o The request must contain a certification that the invention disclosed in the application has not

been and will not be the subject of an application filed in another country, or under a multilateral international agreement, that requires 18-month publication; and

o The request is signed in compliance with 37 CFR § 1.33(b) (Amendments and Other Papers for Correspondence Concerning Patent Applications).

Page 63: TOL Patent Bar Outline

Patent Bar Outline © theOtherLives

* Important to know for the patent bar. 63

1123 & 1124: Rescission of a Nonpublication Request and Notice of Foreign Filing* An applicant may rescind a nonpublication request at any time. The application will then be treated as

though the request was never made. If applicant filed a nonpublication request and then decided to file a counterpart application, applicant

must file either: * o A request to rescind the nonpublication request before filing the counterpart application; or o A notice of foreign filing no later than 45 days after the filing date of the counterpart application*.

• If applicant fails to provide notice within this time period, the application will be abandoned.

• When an application is so abandoned, the applicant’s sole remedy to restore the application is by filing a petition to revive the abandoned application on the basis of unintentional delay – NOT unavoidable delay. Petition requires:

Fee; and Statement that the delay in submitting the notice of foreign filing was

unintentional. 1125: Express Abandonment to Avoid Publication

Applicant may file for express abandonment under 37 CFR § 1.138(c) in order to abandon the application to avoid publication.

o Such request must be filed in time for the Office to remove the application from the publication process, otherwise it will be published.

1126: Publication Fees

Publication fee is due within 3 months of the Notice of Allowance (unless pre-paid). If the publication fee is not paid during this time, the application will go abandoned.

There is no discount for small entity status for the publication fee. 1128: Availability of Published Applications

Members of the public cannot obtain physical access to any published pending applications. Rather, the public may retrieve any available information electronically through the PAIR system.

1129: Request for Early Publication

Earlier publication is possible by submitting a request under 37 CFR § 1.219 along with the publication fee.

An applicant may wish to publish early when s/he suspects someone of infringement in order to get provisional damages.

1130: Republication and Correction of Patent Application Publications

An applicant wishing to correct errors in a patent application publication or to republish with an amended specification, etc., may submit a request for republication, including:

o A copy of the application in compliance with the EFS requirements; o A publication fee; and o A processing fee.

Applications that are no longer pending (such as those abandoned or already issued as patents) will not be republished.

If the applicant wishes to have corrected in a publication a material mistake made by the Office, s/he may submit a request for corrected publication:

o Within 2 months from the date of the patent application publication (non-extendable); and o Identify the Office’s material mistake in the publication.

Examples of material mistakes include: o The publication did not include claims that were included in the originally-filed specification and

not canceled by a preliminary amendment. o The publication did not include a part of the specification that provides support for the published

claims. o The publication did not include any of the drawings originally filed. o The publication did not include the benefit claim to a prior-filed non-provisional application where

the specific reference was timely submitted in the first sentence(s) of the specification or ADS.

Page 64: TOL Patent Bar Outline

Patent Bar Outline © theOtherLives

* Important to know for the patent bar. 64

1132: Requests for Redacted Publication If an application includes a more extensive description than any previously filed corresponding foreign

application, the applicant may file a request for a redacted publication under 37 CFR § 1.217, such that the less-extensive description is published by the Office.

A request for a redacted publication under 37 CFR § 1.217 requires: o A redacted copy of the application in compliance with the Office EFS requirements within 16

months after the earliest filing date for which a benefit is sought. o A certified copy of each foreign filing application that corresponds to the US application for which

a redacted copy is submitted. o A translation of each such foreign filed application that is in a language other than English, and a

statement that the translation is accurate. o A marked up copy of the application showing the redactions in brackets. o A certification showing that the redacted copy of the application eliminates only the part or

description of the invention that is not combined in any counterpart application. 1133: Voluntary Publication

Utility and plant applications filed before November 29, 2000 will not be published under 35 USC § 122(b). However, an application may request publication by submitting a request that includes:

o A copy of the application in compliance with the EFS requirements; o A publication fee; and o A processing fee.

1134: Third Party Inquiries and Correspondence in a Published Application

No protests or public use proceedings may be initiated after the publication of an application without the express written consent of the applicant.

Third parties may submit patents or publications under 37 CFR § 1.99 for consideration in a pending published application within 2 months of the publication or before the Notice of Allowance is sent out, whichever is earlier. (Republication does not restart this 2-month period.)

Submission under 37 CFR § 1.99 may NOT include: o More than 10 total references; o Explanations of the patents or publications; o Documents other than patents or publications; or o Markings or highlights on the patents or publications.

Page 65: TOL Patent Bar Outline

Patent Bar Outline © theOtherLives

* Important to know for the patent bar. 65

CHAPTER 1200: APPEAL* 1201: Introduction

Where patent claims are denied because of prior art or other patentability issues, the questions raised relate to the merits. Remember that questions raised as to procedure are petitionable (to the Director), but those raise on the merits are appealable (to the BPAI).

1204: Notice of Appeal

When a claim has been twice rejected, an applicant may file a Notice of Appeal, accompanied by the required fee, within the specified time period.

o It is not required that the claims be finally rejected, just that they be twice rejected. o If any claim was rejected in a parent application and then again in an offspring application, the

claim will count as being twice rejected, and the applicant can choose to appeal at that time*. After the applicant files a Notice of Appeal, s/he has two months to submit an Appeal Brief.

o When an appellant files an Appeal Brief without filing a Notice of Appeal, the Office will treat the brief as both as long as it is filed in the time period set in the last Office action.

1205: Appeal Brief

Time for Filing Appeal Brief o The appeal brief and fee must be submitted within two months of filing the Notice of Appeal (the

Office date of receipt or the date of “Express Mail” date in if this type of submission is used). o The two-month time period may be extended under 37 CFR § 1.136(a) (Extensions of time).

• A petition may be submitted to the Technology Center, along with a fee. o For an appeal in an ex parte reexamination proceeding, the time period can only be extended

under the provisions of 37 CFR § 1.550(c) (Conduct of ex parte reexamination proceedings). o When an application is revived after abandonment for failure on the part of the appellant to take

appropriate action after final rejection, and the petition to revive was accompanied by a notice of appeal, appellant has 2 months, from the mailing date of the Director’s affirmative decision on the petition, in which to file the appeal brief. The time period for filing the appeal brief may be extended under 37 CFR §1.136.

o Failure to timely file an appeal brief * • Although failure to file the brief and the required appeal brief fee within the permissible

time will result in dismissal of the appeal, if any claims stand allowed, the application does not become abandoned by the dismissal, but is returned to the examiner for action on the allowed claims*.

• If there are no allowed claims, the application is abandoned as of the date the brief was due. Claims that have been objected to as dependent from a rejected claim do not stand allowed.

• In a reexamination proceeding failure to file the brief will result in the issuance of the certificate under 37 CFR § 1.570 or § 1.997.

• If the application goes abandoned, applicant can petition to have it revived by the general route.

• If the appeal is dismissed but the application is not abandoned because there is at least one allowed claim, applicant may file a petition to reinstate the claims and the appeal. A fee must accompany the petition and the applicant must also file:

An RCE (under 37 CFR § 1.114), accompanied by a submission and a fee. For a design application, a CPA is appropriate instead of an RCE; or

An appeal brief and associated fee to reinstate the appeal. • Where the dismissal of the appeal is believed to be in error, a petition pointing this out

may be sufficient. Appeal Brief Content

o Only one copy of the appeal brief is required and it must address the following issues: • Exactly who the real party in interest is; • Any other related appeals and interferences – identified by application number and filing

date; • Status of each claim – identifying whether each one has been allowed, rejected,

canceled, etc.; • Status of the amendments; • A summary of the claimed subject matter; • A concise statement about each ground of rejection to be reviewed on appeal;

Page 66: TOL Patent Bar Outline

Patent Bar Outline © theOtherLives

* Important to know for the patent bar. 66

• The appellant’s contention with respect to each ground of rejection presented and the basis for those contentions, including citations of authorities, statutes, and part of the record relied upon;

• All arguments which the appellant wants the Board to consider must be presented in the Appeal Brief;

• Claims appendix, including a copy of the claims involved in the appeal; • Evidence appendix; and • Related proceedings appendix.

Non-Compliant Appeal Brief and Amended Brief o In the case of a noncompliant appeal brief, the applicant will be given one month or 30 days

(whichever is longer) from the date of the mailing of the non-compliance notice to file an amended brief.

1206: Amendments and Affidavits or Other Evidence Filed With or After Appeal

Amendments are not submitted as a matter of right. Amendments filed after the filing of a Notice of Appeal, but prior to the date of filing an Appeal Brief,

may only be admitted to: o Cancel claims; o Comply with any requirement of form expressly set forth in a previous action; o Present rejected claims in better form for consideration on appeal; or o Amend the specification or claims upon a showing of good and sufficient reasons as to why the

amendment is necessary and was not earlier presented. Amendments filed on or after the filing of an Appeal Brief may only be admitted to:

o Cancel claims (where such cancelation does not affect the scope of any other pending claim in the proceeding); or

o Rewrite dependent claims into independent form. Affidavits or other evidence submitted after filing of a Notice of Appeal, but prior to an appeal brief, may

be admitted if the examiner determines that: o The affidavit or other evidence overcomes all rejections under appeal; and o A showing of good and sufficient reasons why the affidavit or other evidence is necessary and

was not earlier presented has been made. 1207: Examiner’s Answer

An examiner may do one of three things after reviewing an Appeal Brief: o Reopen prosecution to enter a new ground of rejection with approval from the supervisory patent

examiner; o Withdraw the final rejection and allow the application if the examiner determines that the

rejections have been overcome and no new ground of rejection is appropriate; or o Maintain the appeal by conducting an appeal conference and draft an examiner’s answer, which

may include a new ground of rejection. Appeal Conference

o An appeal conference is mandatory in all cases in which an acceptable brief has been filed, except where the examiner determines that the appeal cannot go forward.

o The participants of the appeal conference should include: 1) the examiner charged with preparation of the examiner’s answer, 2) a supervisory patent examiner, and 2) a third examiner (a “conferee”).

Contents of Examiner’s Answer o An examiner’s answer will be prepared following the conference, and will be written within two

months of the receipt of the appeal brief by the examiner. • The answer will contain a response to the arguments in the brief, note any errors in the

appellant’s claims, and state any rejections that are to be withdrawn. • If the examiner’s answer contains a new ground of rejection, it must give appellant a two-

month time period to reply to the new ground of rejection. New Ground of Rejection in Examiner’s Answer

o If the examiner’s answer contains a new ground of rejection, the appellant must, within 2 months of receiving the answer, either:

• Reopen the prosecution of the application (the appeal will be treated as withdrawn); or • Maintain the appeal by filing a reply brief in compliance with 37 CFR § 41.37, which

addresses each new ground of rejection. o If prosecution is reopened:

• The appellant should file a reply in compliance with 37 CFR § 1.111 that addresses each new ground of rejection set forth in the examiner’s answer;

Page 67: TOL Patent Bar Outline

Patent Bar Outline © theOtherLives

* Important to know for the patent bar. 67

• The examiner may make the next Office action final if the new ground of rejection was: Necessitated by amendment; or Based on info presented in an IDS

o If the appeal is maintained, the reply brief must include: • An identification page (appellant’s name, application number, application filing date, title

of the invention, name of the examiner, art unit of examiner, and title of the paper, which should be “Reply Brief”);

• A status of claims brief; • The grounds of rejection to be reviewed on appeal page(s); and • Argument pages.

o Extensions of time under 37 CFR § 1.136(a) are not applicable to the 2-month time period for reply.

o New grounds of rejection are not limited to rejections made in response to arguments presented in the appeal brief.

Reopening of Prosecution After Appeal o Appellant may not request to reopen prosecution if the examiner’s answer does not have a new

ground of rejection. Supplemental Examiner’s Answer

o The examiner may provide a supplemental examiner’s answer to the reply brief, which may NOT include new grounds of rejection.

o The appellant may file another reply brief within 2 months from the mailing of the supplemental examiner’s answer.

o The examiner may provide a supplemental examiner’s answer in response to: • A reply brief that raises new issues; • A remand by the Board for further consideration of a rejection under 37 CFR § 41.50(a);

and • A remand by the Board for other purposes that are not for further considerations of a

rejection under 37 CFR § 41.50(a). 1208: Reply Brief and Examiner’s Response to Reply Brief

The appellant may file a reply brief within 2 months from the mailing fate of the examiner’s answer or supplemental answer.

The appellant is required to file a reply brief or a reply under 37 CFR § 1.111 when the examiner’s answer contains a new ground of rejection.

If the reply brief is in compliance, the examiner must acknowledge receipt. Additionally, the examiner may:

o Withdraw the final rejection and reopen prosecution to respond to the reply brief; or o Furnish a supplemental examiner’s answer responding to any new issue raised in the reply brief.

1209: Oral Hearing

An oral hearing should be requested only when the appellant considers such a hearing necessary for a proper presentation of the appeal.

It may be requested within 2 months of the date the examiner’s answer was sent. The request must be filed in its own writing, and it must include a fee. If the request for the oral hearing is improper, the appeal will be assigned for consideration and a decision

on the briefs will be made without an oral hearing. In the oral hearing, the appellant may only rely on evidence that has been previously entered and

considered by the examiner and present arguments that have been relied upon in the appeal brief or reply brief.

The primary examiner may request to be present at an oral hearing. 1210: Actions Subsequent to Examiner’s Answer but Before Board’s Decision

Jurisdiction does not pass to the board just because a Notice of Appeal has been filed; the Board must accept the appeal before jurisdiction is passed to it.

Jurisdiction is normally transferred from the examiner to the Board: o 2 months after the examiner’s answer, plus mail room time, if no reply brief has been timely filed; o After a supplemental examiner’s answer, pursuant to remand from the Board; o After the examiner has notified the appellant by written communication that the reply brief has

been entered and considered and that the application will be forwarded to the Board. 1211: Remand by Board*

Page 68: TOL Patent Bar Outline

Patent Bar Outline © theOtherLives

* Important to know for the patent bar. 68

The Board has the authority to remand a case to the primary examiner when necessary, such as when the pertinence of the references cited is not clear.

Remand by Board for Further Consideration of rejection o A supplemental examiner’s answer written in response to a remand by the Board for further

consideration of a rejection may set forth a new ground of rejection. o If a supplemental examiner’s answer is written in response to a remand by the Board for further

consideration of a rejection, the applicant must either reopen prosecution or maintain appeal (see above for requirements for each) in order to avoid sua sponte dismissal of the appeal as to the claims subject to the rejection for which the Board has remanded the proceeding.

Remand by Board to Consider Amendment o The Board has no obligation to consider new or amended claims submitted while it has

jurisdiction of the appeal. However, the Board may remand the application to the primary examiner for consideration of the proposed amendment.

Remand by Board to Consider Affidavits or Declarations o Affidavits or declarations filed with the filing of a Notice of Appeal but before jurisdiction passes to

the Board will be considered for entry only if the appellant makes the necessary showing as to why they are necessary and not earlier presented.

Remand by Board for Further Search o A remand to the examiner extends the total pendency of an application and may necessitate an

extension of the patent term under 35 USC § 154(b) (Contents and term of patent; provisional rights).

1212: Board requires Appellant to Address Matter

The Board is authorized to require an appellant to clarify the record by addressing any matter deemed appropriate for a reasoned decision on the appeal, this may include:

o The applicability of particular case law that has not been previously identified as relevant to an issue in the appeal;

o The applicability of prior art that has not been made of record. Appellant will be given a non-extendable time period for reply, and failure to respond during that time will

result in dismissal of appeal. 1213: Decision by Board

The Board of appeals may decide to affirm or reverse the primary examiner’s decision or the Board may find a new ground of rejection.

The Board may suggest an amendment for the applicant to overcome a specific rejection, but the Board is not required to overturn the rejection if such an amendment is made.

New Grounds of Rejection by Board o The decision to review allowed claims is a discretionary one, and the Board is not required to

review such claims, nor should any inference be drawn from allowed claims that are not rejected by the Board.

Publication of and Public Access to Board Decision o Any Board decision is available for public inspection if rendered in a file open to the public. o If the Board decision is rendered in a file that is not open to the public, and if the Director believes

that special circumstances warrant publication, then the decision may be published or made available for public inspection.

1214: Procedure Following Decision by Board*

After the decision has been made by the board, a copy is mailed to the appellant. Once the copy of the decision has been mailed, the applicant will have 2 months to:

o Request a rehearing; or o File an amendment.

The 2-month time period is NOT extendable under 37 CFR § 1.136(a), but may be under 37 CFR § 1.136(b) or 37 CFR § 1.550(c) in the case of a reexamination.

Procedure Following New Ground of Rejection by Board o Following a new ground of rejection raised by the Board, the appellant has 2 months to either:

• Request a rehearing; or • Reopen prosecution before the examiner by submitting an appropriate amendment

and/or new evidence. o An appropriate amendment is one that amends one or more of the rejected claims or substitutes

new claims to avoid the art or reasons provided by the Board.

Page 69: TOL Patent Bar Outline

Patent Bar Outline © theOtherLives

* Important to know for the patent bar. 69

o An amendment that merely adds new claims without either amending the rejected claims or substituting new claims for the rejected claims is not appropriate.

o If the appellant submits an argument without either an appropriate amendment or new evidence as to any of the claims rejected by the Board, it will be treated as a request for rehearing.

Rehearing o Arguments not raised in the briefs before the Board and evidence not previously relied upon in

the brief are not permitted in the request for rehearing except: • Upon a showing of good cause, appellant may present a new argument based on a

recent decision of the Board or a Federal Court; and • New arguments responding to a new ground of rejection.

o If the board decides that no claims stand allowed*, then: • The proceedings in the application are terminated; and • The application is no longer considered as pending, and it will be abandoned.

1215: Withdrawal or Dismissal of Appeal If at least one allowed claim remains in the application, an application is not abandoned if the appeal is

dismissed. Once the appellant has filed a Notice of Appeal, the appellant may also request that prosecution be

reopened for the following situations: o In response to a new ground of rejection made in an examiner’s answer, appellant may file a

reply that addresses the new ground of rejection within 2 months from the mailing of the examiner’s answer.

o In response to a supplemental examiner’s answer that is written in response to a remand by the Board for further consideration of a rejection, the appellant may file a reply that addresses the rejection in the supplemental answer within 2 months of its mailing.

Withdrawal of Appeal o An appellant may withdraw an appeal at any time. However, if no claims stand allowed, the

withdrawal will serve as abandonment of the application. Dismissal of Appeal*

o If no appeal brief is filed within the proscribed period of time, the appeal will be dismissed. o If no claims stand allowed at this point, the application will be abandoned as of the date the brief

was due. o If claims stand allowed in an application, the failure to file a brief and consequent dismissal of the

appeal is to be treated as a withdrawal of the appeal and of any claim not standing allowed. • The allowed claims will be passed to issue.

o Claims that are allowable except for their dependency from rejected claims will be treated as if they were rejected.

1216: Judicial Review

An applicant who is dissatisfied with a decision by the Board may appeal the decision to the US Court of Appeals for the Federal Circuit or may have a remedy by civil action against the Commissioner in US District Court for D.C.

By filing an appeal to the Fed Circuit, failure to prosecute the appeal, such as by an applicant’s failure to file a brief, may result in the dismissal of the appeal by the court.

Page 70: TOL Patent Bar Outline

Patent Bar Outline © theOtherLives

* Important to know for the patent bar. 70

Appeal Steps and Relevant Time Periods

Page 71: TOL Patent Bar Outline

Patent Bar Outline © theOtherLives

* Important to know for the patent bar. 71

CHAPTER 1300: ALLOWANCE AND ISSUE 1301: Substantially Allowable Application, Special

When an application is in condition for allowance, except as to matters of form, the application will be considered special and prompt action taken to require correction of formal matters.

1302: Final Review and Preparation for Issue

A Notice of Allowability is sent to the applicant when the application has been allowed and is expected to patent.

An interference search will be performed once an application is ready for allowance. Examiner’s Amendments and Changes

o Examiners may make informal amendments if the application file is a paper file. Such informal amendments include:

• Misspelled words; • Disagreement of a noun with its verb; • Inconsistent “case” of a pronoun; • Disagreement between a reference character as used in the description and on the

drawing, but only if the examiner is certain of the propriety of the change; • Correction of reversed figure numbers; • Other obvious minor grammatical errors; and • Informalities in the application, other than the ones noted above, or of purely grammatical

nature. o The amendment or cancelation of claims by a formal examiner’s amendment is permitted when

passing an application to issue where these changes have been authorized by the applicant (or attorney/agent) in a telephone or personal interview.

o Substantive changes made to the abstract or the specification must be made with the applicant’s approval.

The examiner will make a statement supplying the reasons why the application was allowed if it is not obvious from the prosecution.*

1303: Notice of Allowance

Undelivered o In the case where a Notice of Allowance is returned and a new notice is sent, the date of sending

the notice must be changed in the file to agree with the date of such re-mailing. Not Withheld Due to Death of Inventor

o The Notice of Allowance will not be withheld due to death of the inventor if the executor or administrator has not intervened.

1304: Amendments After D-10 Notice

“Secrecy order” applications are not sent to issue even when all of the claims have been allowed. Instead of mailing a Notice of Allowance, a D-10 Notice is sent to the applicant.

1305: Jurisdiction

Jurisdiction over the patent passes from the examiner to the Director after the Notice of Allowance is sent. 1306: Issue Fee

The issue fee and any required publication fee are shown on the Notice of Allowance, and these are due 3 months from the date of the Notice of Allowance. The Director has no authority to extend the time for paying the issue fee.

Deferring Issuance of a Patent o It is public policy that a patent issue in regular course once the issue fee is paid. However, the

Office may defer issuance up to 1 month upon request in the absence of extraordinary circumstances or requirement of the regulations that would dictate a longer period.

Practice After Payment of Issue Fee; Receipt of Issue Notification o Utility and reissue patents are issued within about 4 weeks after the issue fee and publication fee

(if required) are received in the Office. o Applicants should file any necessary amendments, assignments, petitions, IDSs or other papers

prior to the date of issue fee payment, as the Office cannot ensure that any paper filed after issue fee payment will reach the appropriate PTO official before the date the application issues as a patent.

1308: Withdrawal from Issue

Page 72: TOL Patent Bar Outline

Patent Bar Outline © theOtherLives

* Important to know for the patent bar. 72

Withdrawal from Issue at the Initiative of the Applicant o Prior to the Payment of Issue Fee

• Applicant must petition the Director under 37 CFR § 1.313(a) or file an RCE under 37 CFR § 1.114 in order to have an application withdrawn from issue.

• If applicant files an RCE, accompanied by fee and a submission, s/he need not pay the issue fee to avoid abandonment of the application.

• Applicant may also file a continuing application on or before the day the issue fee is due and permit the parent application to become abandoned for failure to pay the issue fee (35 USC § 151).

• A petition to withdraw from issue must 1) show good and sufficient reason for withdrawing the application, and 2) include a withdrawal fee.

o After the Payment of Issue Fee • 37 CFR § 1.313(c): Once the issue fee has been paid, the application will not be

withdrawn from issue upon petition by the applicant for any reason except: Unpatentlability of one of more claims, which petition must be accompanied by

an unequivocal statement that one or more claims are unpatentable, an amendment to such claim or claims, and an explanation as to how the amendment causes such claim or claims to be patentable;

Consideration of a request for continued examination in compliance with § 1.114; or

Express abandonment of the application. Such express abandonment may be in favor of a continuing application.

• In addition, applicant should identify some specific and significant defect in the allowed application before it will be withdrawn from issue.

• Petitions under 37 CFR § 1.313(c) to have an application withdrawn after payment of the issue fee should be directed to the Office of Petitions.

Withdrawal from Issue at the Initiative of the Office o 35 USC § 151 and 37 CFR § 1.313(b) do not authorize the USPTO to withdraw an application

from issue after payment of the issue fee for any reason except: • A mistake on the part of the Office; • A violation of 37 CFR § 1.56 or illegality in the application; • Unpatentability of one or more claims; or • For interference.

Rejection After Allowance o If a claim was allowed, it can only be rejected by the primary examiner.

Page 73: TOL Patent Bar Outline

Patent Bar Outline © theOtherLives

* Important to know for the patent bar. 73

CHAPTER 1400: CORRECTING PATENTS 1400: General

A patent may be corrected/amended in one of four ways: o Reissue o Issuance of a Certificate of Correction, which becomes part of the patent o Disclaimer o Reexamination (discussed in MPEP 2200 and 2600)

1401: Reissue

The provisions of 35 USC § 251 permit the reissue of a patent in order to correct an error in the patent deeming it wholly or partly inoperative or invalid, which was made without any deceptive intention and provide the criteria for the reissue patent.

1402: Grounds for Filing

There must be at least one error in the patent for it to qualify for reissue. Such an error must be one that causes the patent to be wholly or partly inoperative or invalid due to:

o A defective specification or drawing that is substantive in nature; or o The applicant claiming more or less than s/he had a right to claim in the patent.

The most common bases for filing a reissue are: o The claims are too narrow or too broad; o The disclosure contains inaccuracies; o Applicant failed to or incorrectly claimed foreign priority; and o Applicant failed to make reference to or incorrectly made reference to prior copending

applications. Errors such as spelling, grammar or typography that do not make the patent inoperative or invalid do not

warrant a reissue. o These types of errors may be corrected in a reissue that is brought for an acceptable reason. o These types of errors are generally fixed by a Certificate of Correction.

A reissue will not be allowed where the applicant simply failed to file a divisional application on the non-elected invention in a restricted application.

Correction of inventorship is easier to correct using a Certificate of Correction, which will be issued if all parties are in agreement and inventorship is not contested.

A reissue may be filed in order to correct a failure to adequately claim benefit under 35 USC § 120 or 119(a)-(d) in an earlier filed copending US patent application. However, the courts have not addressed the issue of correcting such a failure under § 119(e) via reissue.

o The Office may grant a reissue for adding or correcting a benefit claim under 35 USC §119(e) that requires the benefit claim to a provisional application to be submitted during the pendency of the application.

1403: Diligence in Filing

A reissue that is filed within 2 years of the grant of the original patent will not normally be rejected for lacking diligence.

A reissue application can be granted a filing date without an oath or declaration, the basic filing fee, search fee, or examination fee. A period of time will be given to correct these deficiencies, however, a surcharge will be required.

1405: Reissue and Patent Term

Generally, if a patent is reissued, it will be reissued for the remainder of the term of the original patent. The term of a patent will never be lengthened through a reissue. However, one may be shortened through

reissue by way of a terminal disclaimer. o For example, a patent term will not be lengthened by filing a reissue to delete a benefit claim

under 35 USC § 120. The schedule of maintenance fees on a patent is not altered by the filing of a reissue application.

1410: Content of Reissue Application

An application for reissue must contain the same parts required for an original patent application, as well as the additional requirements for a reissue.

An oath or declaration must be submitted by all inventors. When a patent is requested to be reissued:

o The rights to the original patent must be surrendered; o New fees must be paid (same as those required if the application were an original); and

Page 74: TOL Patent Bar Outline

Patent Bar Outline © theOtherLives

* Important to know for the patent bar. 74

o All assignees must provide written consent. Where the reissue application does not seek to enlarge the scope of any claims of the original patent, the

reissue oath may be made and sworn to by the assignee of the entire interest. Reissue Application, Oath or Declaration, and Consent of All Assignees

o If an inventor is to be added in a reissue application, a proper reissue oath or declaration including the signatures of all of the inventors is required.

o If one or more inventors are being deleted in a reissue application, an oath or declaration must be supplied over the signatures of the remaining inventors.

o Consent to the Reissue • In regard to reissues, it will be presumed that an assignee does exist unless otherwise

stated. • If all the assignees cannot give consent for the filing of the reissue, a petition may be sent

to the Office of Petitions. A reissue will be examined upon acceptance of the petition, but it will neither be allowed nor issued until the consent of all the assignees is given.

• Where a continuation reissue application is filed with a copy of the assignee consent from the parent reissue application, and the parent reissue application is not to be abandoned, the copy of the consent will not be accepted.

• Where a divisional reissue application is filed with a copy of the assignee consent from the parent reissue application, regardless of whether or not the parent reissue application is to be abandoned, the copy of the consent will not be accepted.

o Proof of Ownership of Assignee • An assignee who consents to the filing of a reissue must establish their ownership by:

Filing in the reissue application documentary evidence of a chain of title from the original owner to the assignee; or

Specifying in the record of the reissue application where such evidence is recorded in the Office.

1411: Form of Specification

A full copy of the printed patent should be used to provide the abstract, drawings, specification and claims of the patent for the reissue application.

Amendments may be made by physically incorporating the changes within the specification or by providing a separate amendment paper.

Certificate of Correction or Disclaimer in Original Patent o All changes that were made to an original patent by Certificate of Correction will be incorporated

into a reissue application. New Matter

o New matter is not allowed in a reissue application. The claims in the reissue application must be for subject matter that the applicant had the right to claim in the original patent.

1412: Content of Claims

Reissue Claims Must be for Same General Invention o The reissue claims must be for the same invention disclosed – though not necessarily claimed -

in the original patent. o The presence of some disclosure in the original patent should indicate that the applicant intended

to claim – or considered the material now claimed – to be his or her invention. • However, in most cases, the mere failure to claim a disclosed embodiment in the original

patent would not be grounds for prohibiting a claim to that embodiment in the reissue. o Where a restriction requirement was made in an application and applicant permitted the elected

invention to issue as a patent without the filing of a divisional application on the non-elected invention(s), the non-elected invention(s) cannot be recovered by filing a reissue application.

Recapture of Canceled Subject Matter o A reissue will not be granted to “recapture” claimed subject matter that was surrendered in an

application to obtain the original patent. o Three Step Test for Recapture

1. The first step is to determine whether and in what aspect the reissue claims are broader than the patent claims.

2. The second step is to determine whether the broader aspects of the reissued claim related to surrendered subject matter.

3. The third step is for the court to determine whether the reissued claims were materially narrowed in other respects to avoid the recapture rule.

Broadening Reissue Claims

Page 75: TOL Patent Bar Outline

Patent Bar Outline © theOtherLives

* Important to know for the patent bar. 75

o A reissue for broadening the scope of claims in the original patent must be filed within two years of the grant of the original patent.

o An applicant may not enlarge the scope of the claims in a patent during reexamination. However, since a dependent claim is construed to contain all the limitations of the claim from which it depends, the dependent claim must be at least as narrow as the independent claim from which it depends and thus does not broaden the patent.

o A claim would be considered broadened if the patent owner would be able to sue any party for infringement who previously could not have been sued for infringement.

o A claim that has been broadened in a reissue as compared to its scope in the patent is not a broadened reissue claim if it is narrower than, or equal in scope to, any other claim that appears in the patent.

o If the claims are broadened, entities that were carrying out acts or planning to carry out acts that would otherwise be infringement may have the right to continue those acts.

o Exception to the two-year rule: A broadened claim can be presented in a reissue application after the two-year period even though the broadened claim presented after the two years is different than the broadened claim presented within the two year period.

• Where any intention to broaden is indicated in the reissue application within the two years, a broadened claim can subsequently be presented in the reissue after two years.

• If an intention to broaden is indicated in a parent reissue application within the two years, a broadened claim can be presented in a continuing reissue application after two years.

Correction of Inventorship o Correction of the misjoinder of inventors has been held to be a ground for reissue. However, a

Certificate of Correction will be an easy way to do this where inventorship is the only correction being made.

o Where a reissue application seeks to correct inventorship in the patent and the inventors sign the reissue oath/declaration, the correct inventive entity must sign the reissue oath/declaration.

o An assignee of partial interest in a patent may not file a reissue application to correct inventorship where the other co-owner did not join in the reissue application and has not consented to the reissue proceeding.

o Where a reissue to correct inventorship also changes the claims to enlarge the scope of the patent claims, the signature of the correct inventive entity is needed.

1413: Drawings

If there are no changes to the drawing of an original patent, the same drawings may be used for the reissue application. However, a request must be submitted in order to have the drawings transferred.

Where a change to the drawings is desired, applicant must submit a replacement sheet for each sheet of drawing containing a figure to be revised.

1414: Content of Reissue Oath/Declaration

Reissue oaths or declarations must contain the following: o A statement that the applicant believes the original patent to be wholly or partly inoperative or

invalid - • By reason of a defective specification or drawings, or • By reason of the patentee claiming more or less than patentee had the right to claim in

the patent; o A statement of at least one error which is relied upon to support the reissue application (such as

the basis for the reissue); o A statement that all errors which are being corrected in the reissue application up to the time of

filing of the oath/declaration arose without any deceptive intention on the part of the applicant; and

o The information required by 37 CFR § 1.63 (Oath or Declaration). When desiring to claim foreign priority, the oath/declaration in a reissue application must claim foreign

priority even though the priority claim was made in the original patent. Supplemental Reissue Oath/Declaration

o A supplemental oath/declaration is required where any “error” under 35 USC § 251 has been corrected and the error was not identified in the original reissue oath/declaration.

o In the supplemental reissue oath/declaration there is no need to state an error which is relied upon to support the reissue application if an error to support a reissue has been previously and properly stated in a reissue oath/declaration in the application and the error is being corrected in the reissue.

o The supplemental reissue oath/declaration must state an error which is relied upon to support the reissue application only where one of the following is true:

Page 76: TOL Patent Bar Outline

Patent Bar Outline © theOtherLives

* Important to know for the patent bar. 76

• The prior reissue oath/declaration failed to state an error; • The prior reissue oath/declaration attempted to state an error but failed to do so properly;

or • All errors under 35 USC § 251 (Reissue of Defective Patents) stated in the prior reissue

oath/declaration are no longer being corrected in the reissue application. o The supplemental oath must be submitted before the allowance. o If the applicant needs to correct an error after the allowance of the reissue application, a

supplemental reissue oath/declaration must accompany the requested correction stating that the error to be corrected arose without any deceptive intention on the part of the applicant.

o A supplemental oath/declaration will be required where: • The application is otherwise in condition for allowance; • Amendments or other corrections of errors in the patent have been made subsequent to

the last oath/declaration filed in the application; or • At least one of the amendments or other corrections fixes an error.

A reissue application is subject to the same duty of disclosure (and may require an IDS) as a regular application.

While the filing of an IDS with a reissue is not required, it is strongly encouraged. Submission of a supplemental reissue oath/declaration to obviate a rejection cannot be deferred by an

applicant until the application is otherwise in condition for allowance. A supplemental oath/declaration will be required where:

o The application is otherwise in condition for allowance; o Amendments or other corrections of errors in the patent have been made subsequent to the last

oath/declaration filed in the application; or o At least one of the amendmentss or other corrections fixes an error.

1417: Claim for Benefit Under 35 USC § 119(a)-(d)

A claim for the benefit of an earlier filing date in a foreign country under 35 USC § 1199(a)-(d) must be made in a reissue application.

Any application in which a claim for foreign priority is made must specify: o The application number of the foreign application; o The foreign country; and o The day, month, and year of the filing of the foreign application.

No additional certified copy of the foreign application is necessary if a claim for the benefit of an earlier filing date in a foreign country under 35 USC § 119(a)-(d) is made in a reissue application as well as in the application on which the original patent was granted.

1430: Reissue Files Open to the Public and Notice of Filing Reissue Announced in Official Gazette

Reissue applications are open to public inspection. The filing of a reissue application – excluding CPAs – is announced in the Official Gazette.

1441: Two-Month Delay Period Reissue applications will be acted on in advance of other applications (will be “made special”). Generally, a reissue will not be acted on sooner than two months after the announcement is made in the

Official Gazette. o This delay period allows members of the public to review the reissue application and prepare/file

a protest if desired. An applicant may file a petition under 37 CFR § 1.182 to avoid this delay. Protesting a Reissue Application

o A protest may be filed during the pendency of a reissue application, prior to the date of the mailing of the Notice of Allowance.

o Where a final rejection has been issued or the prosecution on the merits has been otherwise closed, a petition under 37 CFR § 1.182 along with the required fee for entry of the protest are required.

o If the protest is a “reissue litigation” protection, it is important that the protest be filed early if the protestor wishes the protest to be considered at the time the Office first acts on the reissue application.

1442: Special Status

All reissue applications are taken up as “special” and will be processed ahead of other special applications unless suspended due to litigation. All issues not deferred will be treated and responded to immediately.

Evidence of concurrent litigation will result in suspension of the application until: o A stay of the litigation is in effect;

Page 77: TOL Patent Bar Outline

Patent Bar Outline © theOtherLives

* Important to know for the patent bar. 77

o The litigation has been terminated; o There are no significant overlapping issues between the application and the litigation; or o It is the applicant’s desire that the application be examined at that time.

Reissue applications involved in “stayed litigation” will be taken up for action in advance of other reissue applications, and the two-month delay period may be waived.

1448: Fraud, Inequitable Conduct, or Duty of Disclosure Issues

The Office does not investigate issues of fraud, inequitable conduct or duty of disclosure in reissue applications.

Applicant’s statement in the reissue oath or declaration of lack of deceptive intent will be accepted as dis-positive except in special circumstances such as an admission or judicial determination of fraud, inequita-ble conduct, or violation of the duty of disclosure.

1449: Protest Filed in Reissue Where Patent is in Interference

The Special Program Law Office (SPLO) will be notified if a reissue application related to a patent is involved in an interference proceeding, and the SPLO will notify all parties relating to the proceeding.

SPLO will make sure the Office does not allow claims in the reissue that are unpantentable over the pending interference counts or found unpatentable in the interference proceeding.

If a reissue and a reexamination are copending, a decision will be made to either merge the two so that a decision can be made on both at once, or else to suspend one proceeding until the other is resolved.

A reissue application can be employed to provoke an interference if the reissue application: o Adds copied claims that are not present in the original patent; o Amends claims to correspond to those of the patent or application with which an interference is

sought; or o Contains at least one error (not directed to provoking an interference) appropriate for the reissue.

An examiner may request a reissue applicant to add a claim to provoke an interference, but a reissue applicant cannot present added or amended claims to provoke an interference if the claims were deliberately omitted from the patent.

If a reissue application is filed while the original patent is in an interference proceeding, the reissue applicant must promptly notify the Board of the filing or the reissue application within 20 days from the filing date of the reissue application.

1450: Restriction and Election of Species Made in Reissue Application

In a reissue, an examiner is permitted to require a restriction between claims newly added and those in the original application, where the added claims are directed to an invention separate and distinct from the invention(s) defined by the original patent claims.

If restriction is required, the subject matter of the original patent claims will be held to be constructively elected unless a disclaimer of all the patent claims is filed in the reissue application, which cannot be withdrawn by the applicant.

The criteria for making a restriction are the same as those used in an original application, including those for requiring an election of species.

1451: Divisional Reissue Applications; Continuation Reissue Applications; Where the Parent is Pending

The original patent may be divided into several divisional reissues. There is no requirement that a family of divisional reissue applications issue at the same time; however, it

is required that they contain a cross reference to each other in the specification. The same patent claim cannot be presented for examination in more than one of the divisional reissue

applications, as a pending claim, in either its original or amended versions. 1452: Request for Continued Examination of Reissue Application

RCE is available for a reissue application. Effective May 29, 2000, an applicant in a reissue application may file a request for continued examination

of the reissue application, if the reissue application was filed on or after June 8, 1995. An RCE will simply prolong prosecution of the reissue, not start a new one.

1453: Amendments to Reissue Applications

Remember that no reissue patent will be granted to enlarge the scope of an original patent unless it is filed within two years.

No amendment may be made on an expired patent. No amendment may be made to introduce new matter. The Specification

Page 78: TOL Patent Bar Outline

Patent Bar Outline © theOtherLives

* Important to know for the patent bar. 78

o All amendments that include any deletions or additions must be made by submission of the entire text of each added or rewritten paragraph with markings, except that an entire paragraph of specification text may be deleted by a statement deleting the paragraph without presentation of the text of the paragraph.

o The applicant must indicate the precise point where each amendment is made. o All bracketing and underlining is made in comparison to the original patent, not in comparison to

any prior amendment in the reissue application. Thus, all paragraphs which are newly added to the specification of the original patent must be submitted as completely underlined each time they are re-submitted in the reissue application.

The Claims o For each claim that is being amended by the amendment being submitted (the current

amendment), the entire text of the claim must be presented with markings. o For each new claim added to the reissue by the amendment being submitted (the current amend-

ment), the entire text of the added claim must be presented completely underlined. o A patent claim should be canceled by a direction to cancel that claim, there is no need to present

the patent claim surrounded by brackets; and o A new claim (previously added in the reissue) should be canceled by a direction to cancel that

claim. o Each amendment submitted must set forth the status of all patent claims and all added claims as

of the date of the submission. A submission will be considered non-responsive if not all statuses are provided.

o Additionally, each claim amendment must be accompanied by an explanation of the support in the disclosure of the patent for the amendment. The explanation is required for complete responsiveness.

The Drawings o Amendments to the original patent drawings are not permitted, and any changes to drawings

must be by way of 37 CFR § 1.173(b)(3) (Reissue specification, drawings, and amendments). 1454: Appeal Brief

The requirements for appeal briefs apply to reissues in the same manner as original applications. However, the claims on appeal presented in an appeal brief for a reissue application should include all

underlining and bracketing necessary to reflect the changes made to the patent claims during the prosecution of the reissue application.

When it is necessary to amend the reissue application in order to place the application in condition for allowance, the examiner may:

o Request that application provide the amendments; or o Make the amendments, with the applicant’s approval, by a formal examiner’s amendment.

1457: Design Reissue Applications and Patents

Generally, the rules for filing a design reissue are the same as those for a utility application. 1460: Effect of Reissue

The reissued patent will be viewed as if the original patent had been originally granted in the amended form provided by the reissue.

“Absolute” intervening rights are available for a party that “prior to the grant of a reissue, made, pur-chased, offered to sell, or used within the United States, or imported into the United States, anything patented by the reissued patent,” and “equitable” intervening rights may be provided where “substantial preparation was made before the grant of the reissue.” See BIC Leisure Prods., Inc. v. Windsurfing Int’l, Inc., 1 F.3d 1214.

1480: Certificates of Correction – Office Mistake

Generally, a Certificate of Correction is used for minor mistakes (like typos) or missing inventor name(s) in issued patents on the part of the patent Office or the applicant, through error and without deceptive intent.

A Cert of correction may be used in order to correct an issued patent as long as the filing was made without deceptive intent in the following situations:

o To correct inventorship; and o A correction for failure to claim priority.

A preferred embodiment materially affecting the scope of a patent omitted in an application is not minor enough to be corrected by a Certificate of correction, since it affects the scope and meaning of claims.

Page 79: TOL Patent Bar Outline

Patent Bar Outline © theOtherLives

* Important to know for the patent bar. 79

The Office has the authority to decline a Certificate of Correction even where the mistake is by the Office, such as where the meaning is still obvious in light of the mistake. Therefore, such requests should only be made for errors of consequence.

1481: Certificates of Correction – Applicant’s Mistake

A Certificate of Correction may be used to correct mistakes by the applicant that are of a clerical or typographical nature or minor character under 37 CFR § 1.323. Changes that constitute new matter or require reexamination are not available by Certificate of Correction.

Corrections that cannot be made by Certificate of Correction must be made by reissue. A petition under 37 CFR § 1.48(a) is appropriate to request a correction of inventorship in a non-

provisional application while a petition under 37 CFR § 1.324 is the appropriate vehicle to correct inventorship in a patent.

1485: Handling of Request for Certificates of Correction

Certificates of Correction will not be issued for patents involved in interferences. 1490: Disclaimers

A disclaimer is a statement filed by an owner (in part or in entirety) of a patent or application, in which said owner relinquishes certain legal rights to the patent.

There are two types of disclaimers: o Statutory disclaimer: The applicant must disclaim or dedicate to the public the entire term of a

certain part of the patent. • In a pending application, entire claims cannot be disclaimed, but rather must be

cancelled. • Since a statutory disclaimer is made in order to relinquish rights, it cannot be made as a

vehicle for adding or amending claims. o Terminal disclaimer: The applicant must disclaim or dedicate to the public the terminal portion of

the entire patent. • The term to be disclaimed must be stated.

The owner of the patent or application can sign a disclaimer, and a person empowered by the owner to sign the disclaimer can also sign it.

For a disclaimer to be accepted, it must be signed by the proper party as follows: o A disclaimer filed in an application must be signed by:

• The applicant where the application has not been assigned; • The applicant and the assignee where each owns a part interest in the application; • The assignee where assignee owns the entire interest in the application; or • An attorney or agent of record.

o A disclaimer filed in a patent or a reexamination proceeding must be signed by either: • The patentee, or • An attorney of agent of record.

Withdrawing a Terminal Disclaimer o Before issuance of a patent, applicant may file a petition under 37 CFR § 1.182 to request

withdrawal. o Post-issuance, there is no available mechanism for withdrawal.

Page 80: TOL Patent Bar Outline

Patent Bar Outline © theOtherLives

* Important to know for the patent bar. 80

Page 81: TOL Patent Bar Outline

Patent Bar Outline © theOtherLives

* Important to know for the patent bar. 81

CHAPTER 1500: DESIGN PATENTS 1501: Statutes and Rules Applicable

Design patent applications are not included in the Patent Cooperation Treaty (PCT). 1502: Definition of a Design

The subject matter claimed in a design application is a design embodied in or applied to an article of manufacture and not the article itself.

A design that is the subject of a patent is inseparable from the article to which it is applied and cannot exist alone merely as surface ornamentation.

Both design and utility patents may be obtained on a single invention if the inventiveness resides in both its utility and ornamental appearance.

Some key differences between design and utility patents include: o The term of a utility patent is 20 years from the US filing date (or earliest effective US filing date),

while the term of a design patent is 14 years from the date of grant; o Maintenance fees are required for utility patents but not for design patents; o Design patent applications include only a single claim while utility patents can have multiple; o Restriction between plural, distinct inventions is discretionary on the part of the examiner in utility

patent applications, while this is mandatory in design applications; o Foreign priority can be obtained for the filing of utility patent applications up to 1 year after the first

filing in any country subscribing to the Paris Convention, while this period for design patents is only 6 months;

o Utility applications may claim the benefit of a provisional application, while a design patent may not;

o A Request for Continued Examination (RCE) may be filed on utility and plant patents but not on design patent applications;

o Continued Prosecution Applications (CPAs) are only available for design applications; and o Design applications are not subject to application publication.

1503: Elements of a Design Patent Application

35 USC § 171 governs rules for design patent applications. A design patent application has essentially the same elements as a utility application, with the essential

elements of 1) a drawing, and 2) a claim in a specific form. Specification

o Preamble and title: A preamble, if included, should state the name of the applicant, the title of the design, and a brief description of the nature and intended use of the article in which the design is embodied.

o Description: Generally, only a brief description of the drawing is necessary. o Claim: Only a single claim is allowed in design applications and is usually written as “the

ornamental design for the article which embodies the design or to which it is applied as shown.” o Statements that describe or suggest another embodiment of the claimed design having a shape

and appearance that would be evident from the one shown are permitted in the specification of an issued design patent

Drawing o Every design patent application must include either a drawing or a photograph of the claimed

design (a single design application cannot contain both). o There must be no inconsistencies among the views of drawings or else they will be objected to. o The drawings or photos should contain a sufficient number of views to disclose the complete

appearance of the design claimed. o Design drawings must not have any broken lines; they should all be solid since everything in the

claimed design is important. 1504: Examination

Ornamentality, novelty, nonobviousness, enablement and definiteness are necessary prerequisites to the grant of a design patent.

Three kinds of patentable design include: o Designs for an ornament, impression, print or picture applied to an article of manufacture (surface

indicia), o Designs for the shape or configuration of an article of manufacture, and o A combination of these two.

The following are not patentable: o A picture standing alone.

Page 82: TOL Patent Bar Outline

Patent Bar Outline © theOtherLives

* Important to know for the patent bar. 82

o A design containing offensive subject matter. A design patent claiming an article that will be hidden from view during its use will likely not be patentable

due to lacking ornamentality. A design must be original and not merely a simulation of an existing object or person. Such a claim will be

rejected under 35 USC § 171 as nonstatutory subject matter in that the claimed design lacks originality. The Graham factual inquiries applicable to evaluating obviousness for utility patent applications also

apply to design applications. As a reminder, these are: o Determining the scope and content of the prior art; o Ascertaining the differences between the claimed invention and the prior art; o Resolving the level of ordinary skill in the art; and o Evaluating any objective evidence of nonobviousness (“secondary considerations”).

Rejections under 35 USC § 171 may be based on: o A design visible in its ultimate end use which is primarily functional based on the evidence of

record; or o A design not visible in its ultimate hidden end use which is itself evidence that the design is

primarily functional. To overcome a rejection based on 35 USC § 171:

o Submit a declaration under 37 CFR § 1.132 over applicant’s signature clearly explaining, specifically and in depth, which areas of the claimed design were created for primarily ornamental reasons.

o Show that the design was done with “thought of ornament.” o Submit a declaration from a representative of the company that commissioned the design (would

show intent behind creation of the design). Restriction

o General restriction practice applies to design patent applications. o A restriction will be required in a design application that contains more than one patentably

distinct design. o Design patents are independent if there is apparent relationship between two or more separate

articles disclosed in the drawings. o To overcome a design restriction, the embodiments have to meet the following requirements:

• They must have overall appearances with basically the same design characteristics; and • The differences between the embodiments must be sufficient to patentably distinguish

one design from the other. Double Patenting

o Determining whether there is a double patenting issue in a design application involves making the following inquiries:

• Is the same design being claimed twice? If yes, then a rejection should be given under 35 USC § 171 on the grounds of

“same invention.” If not, are the designs directed to patentably distinct variations of the same

inventive concept? o If yes, then a rejection based on non-statutory type double patenting

should be given. o For a “same invention” type double patenting rejection (statutory) based on two designs, identical

designs with identical scope must be twice claimed. o For an “obviousness” type double patenting rejection (non-statutory) for design patent

applications, the examiner will apply the Graham factual inquiries in comparing the overall appearance of the claimed design in the application with the overall appearance of the claimed design in the conflicting application or patent.

• A terminal disclaimer may be used to overcome this type of rejection. Foreign Priority

o The provisions of 35 USC § 119(a) – (d) apply to design patent applications. However, in order to obtain the benefit of an earlier foreign filing date, the US application must be filed within 6 months of the earliest date on which any foreign application for the same design was filed, as opposed to 12 months for utility applications.

o In the absence of a statement in the design application as originally filed incorporating by reference the disclosure of an earlier filed application, the disclosure in a continuing application may not be amended to conform to that of the earlier filed application for which priority is claimed. Material must be explicitly incorporated.

1505: Allowance and Term of Design Patent

The term of a design patent is 14 months from the date of issuance.

Page 83: TOL Patent Bar Outline

Patent Bar Outline © theOtherLives

* Important to know for the patent bar. 83

1509: Reissue of a Design Patent

The term of a design patent may not be extended by reissue. See MPEP 1400 for rules governing reissues. A reissue design application must be filed with a copy of all drawing views of the design patent regardless

of whether the views are being cancelled or amended in the reissue. 1510: Reexamination

General reexamination rules apply to design applications. See MPEP 2200. 1511: Protest

General protest rules apply to design applications. See MPEP 1900. 1512: Relationship Between Design Patent, Copyright and Trademark

An ornamental design may be copyrighted as a work of art or receive a trademark and may also be covered by a design patent if it qualifies.

Page 84: TOL Patent Bar Outline

Patent Bar Outline © theOtherLives

* Important to know for the patent bar. 84

Page 85: TOL Patent Bar Outline

Patent Bar Outline © theOtherLives

* Important to know for the patent bar. 85

CHAPTER 1600: PLANT PATENTS 1601: Introduction: The Act, Scope, Type of Plants Covered

35 USC § 161 says that one who “invents or discovers and asexually reproduces any distinct and new variety of plant, including cultivated sports, mutants, hybrids, and newly found seedlings, other than a tuber propagated plant or a plant found in an uncultivated state, may obtain a patent therefore.”

Plants capable of sexual reproduction (or from seed) are not excluded from consideration if they themselves were asexually reproduced.

Asexually propagated plants are those that are reproduced by means other than from seeds, such as by the rooting of cuttings, by layering, budding, grafting or inarching.

“Tuber” means a short, thickened portioned of an underground branch (such as the Irish potato and Jerusalem artichoke). These are not patentable.

Other organisms that are not patentable as plants include: o Bacteria o Plants that are not invented or discovered in a cultivated state and asexually reproduced o Plants that are not obvious

1602: Rules Applicable

37 CFR § 1.161 - The rules relating to applications for patent for other inventions or discoveries are also applicable to applications for patents for plants except as otherwise provided.

1603: Elements of a Plant Application

The elements of a plant application, if applicable, should appear in the following order: o Plant application transmittal form o Fee transmittal form o Application data sheet o Specification o Drawings (in duplicate) o Executed oath or declaration

The components of each of these elements are the same as for utility applications. 1605: Specification and Claim

The specification should include: o A complete detailed description of the plant and the characteristics thereof that distinguish it from

known, existing plants; o The origin or parentage and the genus and species designation of the plant variety sought to be

patented; o The Latin name of the genus and species of the plant claimed should be stated and preceded by

the heading; o Where and in what manner the variety of plant has been asexually reproduced; and o A distinctive reference to color, where color is a distinguishing feature.

Only the entire plant can be patented. Therefore, only one claim is necessary and only one claim is permitted.

1606: Drawings

If color drawings or photos are submitted, two copies are required. If the required copies are not submitted at the time of filing, a filing date will still be accorded the

application. However, the drawings must be submitted before the application is forwarded to the examiner.

The requirements for photos for utility patent applications are waived for plant applications. 1607: Specimens

Specimens need not be submitted unless the examiner requests them. Other

The language of the claim must refer to a “new and distinct variety of plant.” Absolutely no claim should be directed toward a new variety of flower or fruit. The specification should not include statements with laudatory expressions (such as “prettier roses”).

Page 86: TOL Patent Bar Outline

Patent Bar Outline © theOtherLives

* Important to know for the patent bar. 86

Page 87: TOL Patent Bar Outline

Patent Bar Outline © theOtherLives

* Important to know for the patent bar. 87

CHAPTER 1700: MISCELLANEOUS 1702: Restrictions on Practice in Patent Matters*

Employees of the PTO may not prepare, prosecute or even aid in the prosecution of any patent application during their employment and for an additional 2 years afterward unless cleared by the PTO.

1703: The Official Gazette

The Official Gazette for patents (as opposed to the section for trademarks) is published every Tuesday and reports the reexamination certificates, reissues, plant patents, utility patents, and design patents issued, as well as SIRs.

1706: Disclosure Documents

The Office will accept disclosure documents as evidence of date of conception and store them for two years. However, it’s encouraged that a provisional application be filed instead.

The disclosure documents are not part of any patent application filing and therefore will not have any bearing on application filing date or protection of intellectual property. A patent application must be filed for an inventor to obtain protection for his invention.

The Office guarantees that disclosure documents will be kept in confidence and only used to determine date of conception.

Page 88: TOL Patent Bar Outline

Patent Bar Outline © theOtherLives

* Important to know for the patent bar. 88

Page 89: TOL Patent Bar Outline

Patent Bar Outline © theOtherLives

* Important to know for the patent bar. 89

CHAPTER 1800: PATENT COOPERATION TREATY (PCT) * 1801: Basic PCT Principles

A PCT application provides a mechanism for an applicant to file a single “international application” in a single language.

An application filed via this method goes through two stages: o The international stage, where the application is filed as a PCT application; and o The national stage, where the application enters local prosecution of a PTO-member designated

country. The PCT offers an alternative route to filing foreign applications in the offices of the foreign countries that

are PCT members, and PCT filing does not preclude taking advantage of the priority rights and other advantages provided under the Paris Convention and WTO agreement on Trade-Related Aspects of Intellectual Property (TRIPS Agreement).

The applicant must designate at least one member state upon filing the PCT application, and all designations must be made at the time of filing (cannot be added later).

o However, an applicant may make preliminary designations at the time of filing and then will have 15 months to convert preliminary designations to regular.

In most instances, a national US application is filed first, and a PCT application is then filed on the same invention within a year, claiming the benefit of the US national application filing date.

In a continuation or CIP application, a specific reference to the parent application must be included in an application data sheet or in the first sentence of the specification.

Important Players in the PCT Process o Receiving Office (RO)

• This is the office where an applicant files an international application. • US residents and nationals choose from either the USPTO or the WIPO. • An international application must contain the designation of at least one Contracting State

in which patent protection is desired. • The RO will grant an international filing date, collect fees, handle informalities by direct

communication with the applicant, and monitor all corrections. • By 13 months from the priority date, the RO should prepare and transmit a copy of the

international application, called the Search Copy, to the ISA; and the RO then forwards the original, called the Record Copy, to the IB. (See ISA and IB details below).

o International Searching Authority (ISA) • This Office conducts a prior art search of the inventions claimed in international

applications and issues an International Search Report (ISR). • For international applications filed after January 1, 2004, the ISA also issues a Written

Opinion. • Generally, US applicants choose the US to act as the ISA; alternatively, they often

choose the European Patent Office (EPO). • The international search report and the written opinion will normally both be issued by the

USA within 3 months from the receipt of the search copy (this usually occurs 16 months after the priority date).

The search report merely contains a listing of the prior art references. The written opinion discusses whether the ISA believes each claim satisfies the

conditions of novelty, non-obviousness and industrial applicability. • Once complete, the ISA transmits a copy of the search report and written opinion to the

IB. o International Bureau (IB)

• The WIPO in Geneva performs the duties of the IB. • The IB maintains the master file of all the international applications and acts as the

publisher and central coordinating body under the PCT. • After the search report has been transmitted to the IB, applicant has two months to

amend claims directly with the IB.

Page 90: TOL Patent Bar Outline

Patent Bar Outline © theOtherLives

* Important to know for the patent bar. 90

• Applicants must submit a certified copy of the priority document (or the prior US national application) to which the international application claims priority directly to the IB or receiving office no later than 16 months after the priority date.

• The IB will normally publish the international application along with the international search report and any amended claims 18 months from the priority date.

• The written opinion will be made publicly available 30 months from the priority date. o Designated Office (DO)

• The DO is the national office acting for each of the Contracting States designated in an international application (for example, the USPTO).

• No later than the expiration of 30 months from the priority date in most contracting states, the DO must receive:

A copy of the international application, A translation thereof, and The national fee.

• However, some countries still expect to receive the above listed items within 20 months of the priority date.

• For international applications filed on or after Jan 1, 2004, the applicant should file the demand with the IPEA before the expiration of the latest:

3 months from the date of transmission of the international search report and the written opinion to the applicant, or

22 months from the priority date of the international application. o Elected Office (EO)

• The national office acting for the state or region elected under Chapter II. o International Preliminary Examining Authority (IPEA)

• The International Preliminary Examining Authority (IPEA) normally starts the examination process when it is in possession of:

The demand; The amount due; If the applicant is required to furnish a translation under PCT Rule 55.2, that

translation; Either the international search report or a notice of the declaration by the

International Searching Authority (ISA) that no international search report will be established; and

If the international application has a filing date on or after January 1, 2004, the written opinion established under PCT Rule 42bis.1.

• For international applications having an international filing date on or after January 1, 2004, the IPEA shall not start the international preliminary examination before the expiration of the later of three months from the transmittal of the international search report and written opinion; or the expiration of 22 months from the priority date unless the applicant expressly requests an earlier start.

• The IPEA establishes the International Preliminary Examination Report, which is now called the International Preliminary Report on Patentability (IPRP) for applications filed on or after January 1, 2004.

This report presents the final position as to whether each claim is novel, involves an inventive step and is industrially applicable by 28 months from the priority date.

A copy is sent to the IB and to the applicant. 1803: Reservations Under the PCT Taken by the US

If the US is the designated country in the international application, it will only be published after it enters the national stage and then again if it issues as a patent.

The US receiving office only receives applications in English.* 1805: Where to File an International Application*

An application can only be filed in the US RO if at least one of the applicants is a resident or national of the US and if the applicant(s) is the inventor(s).

An international application cannot be filed via fax. 1810: Filing Date Requirements

An international filing date is accorded on the date that the international application was received by the receiving office.

Page 91: TOL Patent Bar Outline

Patent Bar Outline © theOtherLives

* Important to know for the patent bar. 91

Where there are defects in an application, the RO will invite applicant to correct the deficiency within a set time limit.

o The international filing date will then be date on which a timely filed correction is received by the RO.

The USPTO is not competent to receive international applications that are not in the English language and, upon payment of a fee equal to the transmittal fee, the USPTO will forward such applications to the IB provided they are in a language accepted by the IB as receiving Office.

1812: Elements of the International Application

An international application must contain (in the following order): o A request; o A description; o One or more claims; o An abstract (may be submitted later in the filing); o One or more drawings (where necessary); o A sequence listing;

The required fee (may be submitted up to 1 month after filing in the US). An international application must designate at least one country and be in the appropriate language. Similar to the national restriction requirements, an invention claimed in an international application must

be in compliance with the unity of invention. 1819: Earlier International or International-Type Search

Certain ISAs refund part - or all - of the international search fee or reduce the amount of the fee where the search can be based partly or wholly on an earlier search.

1820: Signature of Applicant

Officers such as President, VP, Secretary, Chairman of the Board, etc. (but generally not the attorney) of an organization are presumed to have the authority to sign on behalf of the organization.

An international application must be signed by the applicant(s) or each of their assigns. 1828: Priority Claim and Document

An applicant may claim the priority of an application filed in or for a Contracting State that is a member of the WTO, even if that state is not party to the Paris Convention. The applicant must present identifying evidence of the earlier filed application.

o In countries that use an inventor’s certificate instead of a patent system, a PCT applicant can claim priority to the inventor’s certificate.

An applicant may correct or add a priority claim by a notice submitted to the RO or IB within 16 months from a changed priority date, but no later than 4 months from the international filing date.*

1832: License Request for Foreign Filing Under the PCT

A license for foreign filing is not required to file an international application in the US Receiving Office but may be required before the applicant or the US Receiving Office can forward a copy of the international application to a foreign patent office, the IB or other foreign authority.

Applications under secrecy order will not be forwarded to the IB until the secrecy order is lifted. 1836: Rectification of Obvious Mistakes

Mistakes in an international application can be rectified free of charge. The request for rectification must be addressed to the authority competent to authorize the rectification,

and it must be filed within 26 months form the priority date. 1842: Basic Flow Under the PCT

General Flow

Measuring Time Limits Under the PCT

o Time limits under the PCT are measured from the priority date of the application.

Page 92: TOL Patent Bar Outline

Patent Bar Outline © theOtherLives

* Important to know for the patent bar. 92

o The PCT application should be filed within 12 months of the first application (usually the US national application).

o An international search report and the written opinion must be established 3 months from the receipt of the search copy by the ISA, or 9 months form the priority date, whichever is later.

o An international preliminary examination is optional; the applicant may file a demand for one if he desires prior to the expiration of 3 months from the date of transmittal to the applicant of the International search report and written opinion or 22 months from the priority date.

1843: The International Search

The international search is a thorough, high quality search of the most relevant resources, resulting in the International Search Report and Written Opinion prepared by the ISA.

An international search will not be performed when the invention does not meet the general patentability requirements laid out in 35 USC § 101. For example, non-patentable subject matter includes computer programs, mere presentations of information, etc.

1850: Unity of Invention Before the International Searching Authority

Any international application must relate to one invention only or to a group of inventions so linked as to form a single general inventive concept.

With respect to a group of inventions claimed in an international application, unity of invention exists only when there is a technical relationship among the claimed inventions involving one or more of the same or corresponding special technical features.

Unity of invention is similar to national stage restriction requirements, but restriction requirements do not apply to international applications.

1853: Amendment Under PCT Article 19

The applicant has one opportunity to amend the claims only of the international application after issuance of the Search Report.

The amendments to the claims must be filed directly with the IB, usually within 2 months of the date of mailing of the Search Report.

1857: International Publication

The IB must send a copy of the published international applications to each of the designated Offices that have requested to receive the published application on the day of publication.

An international application may be used as prior art as of its international filing date, or an earlier US filing date for which benefit is properly claimed, under 35 USC § 102(e), if the international application:

o Was filed on or after Nov 29, 2000; o Designated the US; or o Was published under PCT Article 21(2) in English.

1859: Withdrawal of International Application, Designations, or Priority claims

An applicant can withdraw an application, designation of any country or any priority claim by notice to the RO or the IB before the expiration of 30 months from the priority date, and there is no fee to withdraw.

A notice of withdrawal must be signed by all the applicants, an appointed agent or an appointed common representative. The applicant who is considered to be the common representative may NOT sign such a notice on behalf of the applicants.

If the application is withdrawn prior to international publication, it will not be published.

1864-69: The Demand and Preparation for Filing of Demand See: 37 CFR 1.480. Demand for international preliminary examination. Once an applicant files an international application under PCT Chapter I, s/he has the right to file a

demand for preliminary examination under Chapter II. While Chapter I affords the applicant the benefit of an international search, including an international

search report and written opinion for applications filed on or after January 1, 2004, Chapter II affords the applicant examination of the description, claims and drawings to correct any defects, respond to observations, or add in negative findings of claims.

The filing of a demand shall constitute the election of all Contracting States which are designated and are bound by Chapter II of the Treaty on the international filing date.

Applicants will have one month to pay deficient fees plus a late payment fee, and there is no extension of this deadline.

The demand and fees should be submitted directly to the IPEA, and the IPEA will promptly notify the IB of the demand.

Page 93: TOL Patent Bar Outline

Patent Bar Outline © theOtherLives

* Important to know for the patent bar. 93

1871: Processing Amendments Amendments cannot add subject matter that goes beyond the disclosure of the international application

as originally filed. PCT Article 19 amendments are those made to the claims during the Chapter I search phase, and these

amendments can only be made after the search report has been established. PCT Article 34 amendments are those made to the description, claims and drawings during the Chapter II

examination phase 1878: Preparation of the Written Opinion

A written opinion must be prepared by the ISA at the same time the international search report is prepared in applications with an international filing date. Further written opinions may be prepared by the IPEA if necessary.

An invitation by the IPEA to applicant to reply to the examiner’s written opinion will normally set a 2-month time limit for reply except in situations where a 2-month limit would risk delaying the application for another deadline. In that case, a 1-month deadline may be used.

o There is no extension for the time to replay to an examiner’s opinion. One interview by either phone or in person is a matter of right, and it must be requested after the filing of

the demand and before the expiration of the limit for reply to an examiner’s opinion. The relevant date for the purpose of considering prior art is defined as:

o The international filing date; o Where the international application claims the priority of an earlier application and has an

international filing date which is within the priority period, the filing date of such earlier application (unless priority is not valid);

o Where the international application claims the priority of an earlier application and has an international filing date that is later than the date on which the priority period expired but within the period of 2 months from that date, the filing date of such earlier application, unless the authority considers that the priority claim is not valid.

1879: Preparation of the International Preliminary Examination Report

The international preliminary examination report is prepared by the IPEA and (as of January 1, 2004) must be established within:

o 28 months from the priority date; or o 6 months from the time provided for the start of international preliminary examination; or o 6 months from the date of receipt by the IPEA of the translation, whichever expires last.

If the IPEA elects not to establish a report, the examiner must specify that the report has not been established because:

o The application relates to subject matter that does not require international preliminary examination;

o The description, claims or drawings are so unclear that no meaningful opinion could be formed; o The claims are so inadequately supported by the description that no meaningful opinion could be

formed. 1893: National Stage (US National Application Filed Under 35 USC § 371)

An applicant who uses the PCT instead of filing a national application in the USPTO gains the benefit of: o A delay in time when papers must be submitted to the national offices; o An international search and a written opinion; o A delay in the expenditure of fees; o Additional time for research; o The option of obtaining international preliminary examination.

The national stage is unique compared to a domestic application in that: o It is submitted later (normally 30 months from the claimed priority date, compared to 12 months

for a domestic application claiming priority); o The status of the prior art is generally known before the national stage begins, and this is not

necessarily so in a domestic national application; o If the filing of an international application is to be taken into account in determining the

patentability or validity of any application for patent or granted patent, then special provisions apply.

One the national stage application has been taken up by the examiner, prosecution proceeds in the same manner as a domestic application except that the prior art date is the international priority date and the unity of invention requirement proceeds under rules of restriction requirement.

Correcting Deficiencies

Page 94: TOL Patent Bar Outline

Patent Bar Outline © theOtherLives

* Important to know for the patent bar. 94

o If the basic national fee has been paid and the copy of the international application (if required) has been received by expiration of 30 months from the priority date, but the required oath/declaration, translation, search fee, examination fee, or application size fee has not been filed prior to commencement of the national stage, the Office will send the applicant a notice identifying any deficiency and provide a period of time to correct the deficiency.

Correcting Inventorship o An oath or declaration that names an inventive entity different than that set forth in the

international application will not be accepted for purposes of entering the US national phase unless there is submitted:

• A statement from each person being added or deleted as an inventor that any error in inventorship in the international application occurred without deceptive intent on his/her part;

• The processing fee; and • The written consent of the assignee if an assignment has been executed by any of the

original named inventors. o Where an inventor who needs to correct cannot or will not do so, a petition under 37 CFR § 1.47

must be filed. Abandonment

o The application will be abandoned as to the US at 30 months from the priority date if the requirements for the submission of the basic national fee and a copy of the international application (where necessary) are not satisfied.

o If the requirements for an English translation of the international application, oath/declaration, search fee, examination fee and application size fee are not met within a time period set in a notice provided by the Office, then the application will become abandoned upon expiration of the time period set in the notice.

1895: A Continuation, Divisional, or Continuation-In-Part Application of a PCT Application Designating the US

It is possible to file a US national application under 35 USC § 111(a) during the pendency of the international application designating the US without the completing the requirements for entering the national stage, and this may be done by filing a divisional, continuation or CIP application.

CIP applications are generally filed in instances where applicants seek to add matter to the disclosure which is not supported by the disclosure of the international application as originally filed, as new matter may not be added to a US national stage application.

To obtain benefit under 35 USC §§ 120 and 365(c) of a prior international application designating the US, the continuing application must:

o Include a specific reference to the prior international application; o Be copending with the prior international application; and o Have at least one inventor in common with the prior international application.

A claim for foreign priority under 35 USC § 119(a)-(d) must be made in the continuing application in order to obtain the benefit of the filing date of the prior filed foreign application.

1896: The Differences Between a National Application Filed Under 35 USC 111(a) and a National Stage Application Submitted Under 35 USC § 371

Filing Date o Domestic National: For an application filed under 35 USC § 111(a), the filing date is the date the

USPTO receives a specification containing a description, at least one claim and any required drawings.

o National Stage: For a PCT international application, the filing date is the date the applicant satisfies Article 11 requirements and contains a description, a claim, names at least one applicant who is a resident or national of a PCT Contracting State, filed in the prescribed language and designating at least one Contracting State.

Effective Date as a Reference o Domestic National: Under § 102(e), a national application’s date of reference is its earliest

effective US filing date, taking into consideration any proper priority or benefit claim to prior US applications under 35 USC § 119(e) or 120 if the prior application(s) properly support(s) the subject matter to make the rejection.

o National Stage: Under § 102(e), an international application’s date of reference may be the international filing date if certain conditions are met, such as 1) the international filing date is after Nov 29, 2000, 2) it designates the US and 3) is published under PCT Article 21(2) in English.

Priority Requirements

Page 95: TOL Patent Bar Outline

Patent Bar Outline © theOtherLives

* Important to know for the patent bar. 95

o Domestic National: The applicant must provide the claim and the certified copy of the foreign priority application.

o National Stage: the WIPO must provide the certified copy of the priority application and the applicant must provide the claim.

Unity of Invention o Domestic National: Restriction practice is used. o National Stage: Unity of invention practice is used for the international application, but restriction

practice is used after the application enters the national stage. Filing Fees

o Each is subject its own prescribed filing fees. Reference to Application in Declaration

o Applicant’s oath or declaration is required to identify the specification to which it is directed. o The specification may be identified in a US national application filed under § 111(a) by reference

to an attached specification or by reference to the application number and filing date of the previously filed specification.

o Submissions under the National Stage may identify the specification in the same manner as for § 111(a) or may identify the specification by reference to the international application number.

Page 96: TOL Patent Bar Outline

Patent Bar Outline © theOtherLives

* Important to know for the patent bar. 96

Page 97: TOL Patent Bar Outline

Patent Bar Outline © theOtherLives

* Important to know for the patent bar. 97

CHAPTER 1900: PROTEST 1901: Protest Under 37 CFR § 1.291

A protest is a means for a third party to challenge whether a pending application should issue. A party obtaining knowledge of an application pending in the Office may file a protest against the application and bring attention to the facts that would make the granting of a patent improper.

Who Can Protest o Any member of the public can submit a protest against a pending application for which they have

relevant information. o Protests can be filed anonymously. Any member of the public or his attorney can file a protest

without naming the actual party Information that Can be Relied Upon in Protest

o Information useful for protesting includes information showing: • That the subject matter was publicly known or used in the US; • That the subject matter was used or on sale in the US for more than 1 year before the

filing date; • That the applicant abandoned the invention; • That the application does not meet statutory requirements; • Fraud or violation of the duty of disclosure; and • Whether the wrong or incomplete group of inventors are listed on the application.

o The types of evidence that may be used in a protest include: • Complaints • Answers • Depositions • Answers to interrogatories • Exhibits • Transcripts of hearings or trials • Court orders and opinions • Stipulations of the parties

o Arguments that cannot be relied upon in protest include those presenting fraud, inequitable conduct, or violation of duty of disclosure in protests.

How Protest is Submitted o The protestor should submit:

• Proper identification of the application being protested; • A listing of the patents, publications or other information relied upon; • A copy of each item listed; • A translation of non-English papers; • A concise explanation of the relevance of the references; and • Proof of service to the applicant by:

In-person delivery to applicant; Leaving the protest at applicant’s place of business or residence; Sending the protest by first class mail; or Publishing in the Official Gazette, as a last resort.

o If servicing the applicant is not possible, the entire protest must be mailed to the Office in duplicate.

o No fee is required when submitting a protest. When the Protest Should be Submitted

o A protest must be submitted prior to application publication, a Final Rejection or a Notice of Allowance.

o A protest may only be submitted after publication (but before a final rejection or notice of allowance) with permission of the applicant.

o Once the initial protest is filed, the protestor may not submit any additional information. o Protests may also be submitted during the pendency of a reissue application, ideally during the

first 2 months following announcement of the reissue in the Official Gazette. o Generally, one month is allowed for an applicant to comment on a protest before an examiner will

take action. Protestor Participation

o Protests are ex parte matters, so protestors are not permitted any involvement in the proceedings after submitting the protest.

o Like any third party, the protestor is not entitled to any information about the patent application unless granted permission by the applicant.

Page 98: TOL Patent Bar Outline

Patent Bar Outline © theOtherLives

* Important to know for the patent bar. 98

Page 99: TOL Patent Bar Outline

Patent Bar Outline © theOtherLives

* Important to know for the patent bar. 99

CHAPTER 2000: DUTY OF DISCLOSURE 2001: Duty of Disclosure, Candor, and Good Faith

Any information material to the patentability of the invention must be disclosed to the Office with candor and good faith.

o “Material” means information pertaining to the patentability of the subject matter in the application. Information is material when:

• It establishes a prima facie case of unpatentability to a claim; or • It refutes or is inconsistent with a position an applicant takes.

No patent will be granted on an application in which fraud on the Office occurred or the duty to disclosure was violated through bad faith or intentional misconduct. If fraud on the Office is discovered after a patent has issued, the patent will be invalidated.

Who has Duty to Disclose o The following have the duty of disclosure:

• Each inventor; • Each registered practitioner; • Each assignee; and • Every other person who is substantially involved in the preparation or prosecution of the

application. o The duty does not extend to typists, clerks (or similar personnel), or to corporations.

To Whom Duty of Disclosure is Owed o The duty of disclosure is owed to the PTO. o Individuals having information may disclose the information to an attorney, agent or inventor who

then will have the duty to disclose the relevant information to the Office. o The disclosure must be in writing.

Information Under 37 CFR § 1.56(a) and (b) o Information material to an invention includes patents, publications, and other information such as

that on possible prior public uses, sales, offers to sell, derived knowledge, a prior invention by another, inventorship conflicts, etc.

o Information known in the past may not fit under the duty to disclose because it may not be realized that the information is relevant to the present application.

o Information from a related litigation must be brought to the attention of the examiner. o Information merely favorable to patentability is not required under this rule.

Sources of Information o Sources include, generally, third parties and any searches performed by a person having the duty

of disclosure. 2003: Disclosure – When Made

The duty to disclose exists until the application becomes abandoned or the patent issues. If information is sent before the issue fee is paid, the application may be withdrawn from issue and

abandoned so that the relevant info may be considered in a continuing application. 2004: Aids to Compliance with Duty of Disclosure/Inequitable Conduct Issues

Aids to proper disclosure that a practitioner may consider giving to his client (the inventor) include: o A questionnaire for the applicant involving the disclosure; o Asking questions about inventorship; o Asking questions about the disclosure of the best mode; o Making sure the inventor knows the responsibilities involved with signing the oath; o Explaining the scope of the claims; o Evaluating the materiality of the prior art; o That the prior art is properly described; o An accurate specification; o Checking on other applications by the same inventor; o Submitting whatever may be relevant; o Submitting information promptly; o Highlighting the most relevant information; o Checking on CIP applications; o Any outside information; o Checking that all individuals are informed of their duty of disclosure; and o Recording and keeping discarded information that was not considered relevant.

Page 100: TOL Patent Bar Outline

Patent Bar Outline © theOtherLives

* Important to know for the patent bar. 100

2010: Office Handling of Duty of Disclosure/Inequitable Conduct Issues A determination of inequitable conduct or a deceptive intention by the applicant requires a high level of

proof. Generally, duty of disclosure issues under 37 CFR § 1.56 are handled by the courts, not the PTO. A finding of “fraud”, “inequitable conduct” or a violation of duty of disclosure with respect to any claim in

an application or patent, renders all the claims thereof unpatentable or invalid. 2012: Reissue Applications Involving Issues of Fraud, Inequitable Conduct, and/or Violation of Duty of Disclosure

Collateral Estoppel o Once a patent is declared invalid, a collateral estoppel barrier is created against any further

litigation and the PTO will not review any other matters concerning it.

Page 101: TOL Patent Bar Outline

Patent Bar Outline © theOtherLives

* Important to know for the patent bar. 101

CHAPTER 2100: PATENTABILITY - *This chapter will be tested heavily* 2105: Patentable Subject Matter – Living Subject Matter

The tests set forth by the court regarding patentable living subject matter are: o “The laws of nature, physical phenomena and abstract ideas” are not patentable subject matter; o A “nonnaturally occurring manufacture or composition of matter – a product of human ingenuity –

having a distinct name, character and use” is patentable subject matter; o “A new mineral discovered in the earth or a new plant found in the wild is not patentable subject

matter;” o “The production of articles for use from raw materials prepared by giving to these materials new

forms, qualities, properties, or combinations whether by hand labor or machinery” is “manufacture” under 35 USC § 101.

2106: Patentable Subject Matter Eligibility – 35 USC § 101

Generally, anything under the sun that is made by man may be patentable, including (broadly): o Machines: A concrete thing, consisting of parts or of certain devices and combinations of

devices. o Manufactures: The production of articles for use from raw or prepared materials by giving to

these materials new forms, qualities, properties or combinations, whether by hand labor or by machinery.

o Compositions of Matter: A composition of two or more substances or a composite article whether it be the result of chemical union or of mechanical mixture.

o Processes: Involve actions; a process, art or method and includes a new use of a known process, machine, manufacture, compositions of matter or material.

o In addition to the above-listed categories, claims may also be “product-by-process” claims, where the product claimed is defined by the process that makes it.

Utility Requirement o An invention must have practical applications in addition to being nonobvious and novel.

• Therefore, mere presentations of information (such as computer programs with no utility) are not patentable.

o Remember, an invention absolutely must satisfy the utility requirement of 35 USC § 101. • An invention that that does not operate to produce the results claimed by the patent

applicant is not a “useful” invention in the meaning of the patent law. However, the Federal Circuit has stated that the claimed invention must be totally incapable of achieving a useful result in order to violate § 101.

• The invention does not have to work flawlessly, it merely has to be capable of producing the claimed results.

• Inventions that assert to have utility in the treatment of human or animal disorders are subject to the same legal requirements for utility as inventions in any other field of technology.

Courts have held that the mere identification of a pharmacological activity of a compound that is relevant to an asserted pharmacological use provides an “immediate benefit to the public” and thus satisfies the utility requirements.

Rejection under § 101 o To properly reject a claimed invention under § 101, the Office must:

• Make a prima facie showing that the claimed invention lacks utility, and • Provide a sufficient evidentiary basis for factual assumptions relied upon in establishing

the prima facie showing. o The prima facie showing must contain the following elements:

• An explanation that clearly sets forth the reasoning used in concluding that the asserted specific and substantial utility is not credible;

• Support for factual findings relied upon in reaching this conclusion; and • An evaluation of all relevant evidence of record, including utilities taught in the closest

prior art. 2111: Claim Interpretation; Broadest Reasonable Interpretation*

Interpretation, Generally o During examination, pending claims must be “given their broadest reasonable interpretation

consistent with the specification.” • Such interpretation must be consistent with the interpretation that those skilled in the art

would teach.

Page 102: TOL Patent Bar Outline

Patent Bar Outline © theOtherLives

* Important to know for the patent bar. 102

o An examiner should not import limitations in the specification to the claims where the limitation is not part of the claims.

o An applicant may be his/her own lexicographer, meaning the applicant may ascribe any meaning s/he wishes to a particular term so long as it is clearly defined in the specification.

• Otherwise, the examiner will apply the plain meaning to the language presented. Effects of Preamble

o A preamble does not necessarily limit a claim. Such a determination is made on a case-by-case basis.

o Generally, where the preamble is necessary to give life and meaning to the claim, it may also limit it.

o Any terminology in the preamble that limits the structure of the claimed invention must be treated as a claim limitation.

Transitional Phrases* o Transitional phrases define the scope of a claim with respect to what unrecited additional

components or steps, if any, are excluded from the scope of the claim. o “Comprising”:

• Is inclusive or open-ended and does not exclude additional, unrecited elements or method steps;

• “Including” and “characterized by” are synonymous with comprising. o “Consisting essentially of”:

• Limits the scope of a claim to the specified materials or steps and those that do not materially affect the basic and novel characteristics of the claimed invention;

• For the purpose of searching for and applying prior art under 35 USC § 102 or 103, absent a clear indication in the specification or claims of what the basic and novel characteristics actually are, the term “consisting essentially of” will be construed as equivalent to “comprising.”

o “Consisting of”: • Excludes any element, step, or ingredient not specified in the claim (cannot add an

element or step). • When the phrase “consists of” appears in a clause of the body of a claim, rather than

immediately following the preamble, it limits only the element set forth in that clause; other elements are not excluded from the claim as a whole.

o Other transitional phases must be construed in light of the specification. 2112: Requirements of Rejection Based on Inherency; Burden of Proof

Inherency* o The express, implicit and inherent disclosures of a prior art reference may be relied upon in the

rejection of claims under 35 USC § 102 or 103. o Inherent features need not be recognized at the time of the invention; in need only be inherent. o Something that is old does not become patentable merely upon the discovery of a new property. o Under the principles of inherency, if a prior art device, in its normal and usual operation, would

necessarily perform the method claimed, then the method claimed will be considered anticipated by the prior art device.

o The fact that a certain result or characteristic may occur or be present in the prior art is not sufficient to establish the inherency of that result or characteristic.

Burden of Proof o Once a reference teaching a product appearing to be substantially identical is made the basis of a

rejection, and the examiner presents evidence or reasoning tending to show inherency, the burden shifts to the applicant to show a nonobvious difference.

2113: Product-by-Process Claims

Product-by-process claims are not limited to the manipulations of the recited steps, only the structure implied by the steps.

If the product in the product-by-process claim is the same as or obvious from a product of the prior art, the claim is unpatentable.

o Merely making an old product by a new process does not, in itself, warrant a patent. o However, if the new process results in an improvement over the old process – such as a more

pure product – the new product may be patentable as a product of the new process. 2114: Apparatus and Article Claims – Functional Language

Apparatus claims must be structurally distinguishable from the prior art. Remember that apparatus claims cover what a device is, not what a device does.

Page 103: TOL Patent Bar Outline

Patent Bar Outline © theOtherLives

* Important to know for the patent bar. 103

The manner of operating the device does not differentiate an apparatus claim from prior art. However, a prior art device can perform all the functions of the apparatus claim and still not anticipate the

claim if there is a structural difference. 2115: Material or Article Worked Upon by Apparatus

The material or article worked upon does not limit an apparatus claim. 2116: Material Manipulated in Process

All of the limitations of a claim must be considered when weighing the differences between the claimed invention and the prior art in determining the obviousness of a process or method claim.

2121: Prior Art; General Level of Operability Required to Make a Prima Facie Case

Prior Art is Presumed to be Operable/Enabling o When prior art anticipates or makes obvious all elements of the claimed invention, the burden is

on the applicant to provide facts showing inoperability of the prior art. o The level of disclosure required within a reference to make it an “enabling disclosure” is the same

regardless of the type of prior art. o Efficacy is NOT a requirement for prior art enablement as long as the reference provides an

enabling disclosure and anticipates a claimed invention in sufficient detail to enable a person of ordinary skill in the art to carry out the claimed invention.

Use of Prior Art in Rejections Where Operability Is in Question o § 102 Rejections and Addition of Evidence Showing Reference is Operable

• It is critical that the reference teach every element of a claim, but it is not necessary that the reference teach how to practice the invention. A rejection can still be made based on that reference if there is secondary evidence (other publication, patent, etc.) that teaches how to make or use the invention.

o § 103 Rejections and Use of Inoperative Prior Art • A non-enabling reference may qualify as prior art for the purpose of obviousness.

Compounds and Compositions – What Constitutes Enabling Prior Art o One of ordinary skill in the art must be able to make or synthesize the compound or composition

based on the disclosure. o A reference does not contain an “enabling disclosure” if attempts at making the compound or

composition were unsuccessful before the date of invention. Plant Genetics – What Constitutes Enabling Prior Art

o The disclosure must be sufficient to enable one of ordinary skill to grow and cultivate the plant. Apparatus and Articles – What Constitutes Enabling Prior Art

o A picture may constitute an enabling disclosure as long as the picture shows all of the claimed structural features and how they are put together.

2123: Rejection Over Prior Art’s Broad Disclosure Instead of Preferred Embodiments

Everything disclosed in a patent is available as prior art, even that which is not claimed by the inventor to be his own.

Nonpreferred and alternative embodiments constitute prior art; the mere disclosure of more than one alternative does not constitute a teaching away from any of these alternatives.

2124: Exception to the Rule that the Critical Reference Date Must Precede the Filing Date

In some circumstances a factual reference need not antedate the filing date, such as where a reference is cited to show a universal fact.

o Such facts include the characteristics and properties of a material or a scientific truism. 2125: Drawings as Prior Art

Drawings Can Be Used as Prior Art o Like other qualifying prior art, drawings and pictures can anticipate claims if they clearly show the

structure that is claimed, including all of the claimed structural features and how they are put together.

Proportions of Features in a Drawing Are Not Evidence of Actual Proportions When Drawings Are Not to Scale

o Where it is not indicated that the drawings are to scale or where a scale is not provided, arguments should not be based on measurement of the drawing features.

2126: Availability of a Document as a “Patent” for Purposes of Rejection Under 35 USC § 102(a), (b) and (d)

Page 104: TOL Patent Bar Outline

Patent Bar Outline © theOtherLives

* Important to know for the patent bar. 104

A secret patent is only available as a reference for § 102(a) or (b) as of the date that it is made available to the public. However, a secret patent is available as reference under § 102(d) as of the grant date since 102(d) applies where the applicant (or his/her representative or assign) applied for a patent for the same invention.

The date that a foreign patent is effective as a reference is usually the date patent rights are formally awarded to its applicant.

2127: Domestic and Foreign Patent Applications as Prior Art

Abandoned Applications, Including Provisional Applications o Abandoned applications can be used as prior art when disclosed to the public, such when

referenced in the disclosure of another patent, in a publication, or by voluntary disclosure. Applications Which Have Issued as Patents

o A § 102(e) rejection cannot rely on matter that was canceled from the application and thus did not get published in the issued patent.

o Canceled matter only becomes prior art as of the date the patent issues because that is when the application file history becomes available to the public.

o A § 102(b) rejection over a published application may rely on information that was canceled prior to publication.

Foreign Applications Open for Public Inspection (Laid Open Applications) o Laid open applications may constitute “Published” documents, such as when the specification is

announced in an official journal where anyone can obtain copies and is sufficiently open to the public.

2128: “Printed Publications” as Prior Art

A Reference is a “Printed Publication” if It is Accessible to the Public o The strict definitions of the words “printed” and “publication” are no longer valid in this context

given of the state of advanced technology by which information is made available. The following rules have been set for determining whether information constitutes public disclosure of a printed publication for the purpose of using it as prior art.

Electronic Publications as Prior Art o An electronic publication, such as an online database or webpage, qualifies as a “printed

publication within the meaning of § 102(a) and (b) provided the publication was accessible to persons concerned with the relevant art.”

o Electronic publications are available as prior art from the date of publication. • If the publication does not have a publication date associated with it, it cannot be used

under § 102(a) or (b). However, it may still be used to show the state of the art. o Like most other prior art, an electronic publication can be relied on for all that it teaches. o It is not necessary to show that anyone actually viewed the prior art online to use it as the basis

for a rejection as long as it was available to the public. Level of Public Accessibility Required*

o A thesis placed in a university library may qualify as prior art if sufficiently accessible to the public. • It has been held that a doctoral thesis which was indexed and shelved in a library was

sufficiently open to the public to meet the “printed publication” requirement. o An orally presented paper can constitute a “printed publication” if written copies are disseminated

or available without restriction. o Internal documents intended to be confidential are NOT “printed publications” regardless of how

many copies are distributed internally as long as there is an existing policy of confidentiality within the organization.

o Publicly displayed documents can constitute a “printed publication” even if the duration of display is for only a few days and the documents are not disseminated by copies or indexed in a library or database.

• The requirement is that persons or ordinary skill in the art can view it and are not precluded from copying it.

• The factors for determining whether posted information is a “printed publication” are: The length of time the display was exhibited; The expertise of the target audience; The existence (if any) of reasonable expectations that the material displayed

would not be copied; and The simplicity or ease with which the material displayed could have been copied.

o Note: In these days of camera cell phones and extremely portable digital cameras, the showing standard for copyability is not high.

Date Publication is Available as a Reference

Page 105: TOL Patent Bar Outline

Patent Bar Outline © theOtherLives

* Important to know for the patent bar. 105

o Date of accessibility can be shown through evidence of routine business practice, and specific evidence of the actual date a document becomes publicly available is not always necessary.

o A journal article or other publication becomes available as prior art on the date that it is received by a member of the public, such as an article distributed by mail.

2129: Admissions as Prior Art

Admissions by Applicant Constitute Prior Art o If an applicant identifies the work of another as prior art in the specification, the reference can be

relied upon for both anticipation and obviousness rejections regardless of whether it would otherwise qualify as prior art.

Jepson Claims o 37 CFR § 1.75(e): Where the nature of the case admits, as in the case of an improvement, any

independent claim should contain in the following order: • A preamble comprising a general description of all the elements or steps of the claimed

combination which are conventional or known, • A phrase such as "wherein the improvement comprises," and • Those elements, steps and/or relationships which constitute that portion of the claimed

combination that the applicant considers as the new or improved portion. o A claim in Jepson format is an implied admission that the subject matter of the preamble is the

prior work of another. o However, this implication can be overcome if the applicant provides a different reason for using

the Jepson style claim, such as to avoid a double patenting rejection. o Where a Jepson preamble references the applicant’s own work, it cannot be used against the

claims. Information Disclosure Statements (IDS)

o A reference is NOT taken as admitted prior art merely because it is listed in an IDS. 2131: Anticipation* – Application of 35 USC § 102(a), (b), and (e)

General Rule: One reference only may be used to anticipate a claim. o Remember: To anticipate a claim, the reference must teach every element of the claim. Put

differently, all elements must be taught by a single reference to be anticipated. • However, when a claim covers several structures or compositions, the claim may be

anticipated if any of the structures or compositions within the scope of the claim is disclosed in the prior art.

o Exception to the General Rule • A § 102 rejection over multiple references has been held to be proper when the extra

references are cited to: Prove the primary reference contains an “enabled disclosure;”

o Extra references and extrinsic evidence can be used to show that the primary reference contains an enabled disclosure, such as where one reference fully anticipates a machine or composition but does not provide an enabling disclosure of the invention.

Explain the meaning of a term used in the primary reference; or o Extra references or other evidence can be used to show meaning of a

term used in the primary reference but may not be used to expand the meaning of term(s) used in the primary reference.

Show that a characteristic not disclosed in the reference is inherent. o Additional evidence may be used to show an inherent characteristic of

the thing taught by the primary reference, such that it makes clear that the missing descriptive matter is necessarily present in the thing described in the reference.

Genus-Species Situations o A species will anticipate a claim to a genus.

• A reference that clearly names the claimed species anticipates the claim no matter how many other species are named.

• A generic chemical formula will anticipate a claimed species covered by the formula when the species can be “at once envisaged” from the formula.

Anticipation of Ranges o A specific example in the prior art that is within a claimed range anticipates the range.

• A claim that covers several compositions or a range – such as temperature or percent composition – will be anticipated if even one of the compositions or points in the range is disclosed in prior art.

Page 106: TOL Patent Bar Outline

Patent Bar Outline © theOtherLives

* Important to know for the patent bar. 106

o Prior art that teaches a range overlapping or touching the claimed range anticipates if the prior art range discloses the claimed range with “sufficient specificity.”

• What constitutes “sufficient specificity” must be determined on a case by case basis. For example, if the claims are directed to a narrow range and the prior art discloses a broad range where no specific examples of success have been shown in the narrow range, it may be reasonable to conclude that the narrow range is not sufficiently disclosed in the prior art. However, all of the facts will have to be assessed and weighed in each case.

o Prior art that teaches a value or range that is very close to but does not overlap or touch the claimed range does not anticipate the claimed range.

Secondary Considerations o While secondary considerations, such as unexpected results, are considered in determining

obviousness, such considerations are irrelevant in § 102 rejections. Nonanalogous or Disparaging Prior Art

o Arguments that a reference is nonanalogous, disparaging or is not recognized as solving the problem are irrelevant to § 102 rejections for anticipation.

o Further, the question whether a reference “teaches away” from the invention is inapplicable to an anticipation analysis.

2132: 35 USC § 102(a)

“Known or Used” o “Known or used” means publicly known or used. o Knowledge or use is accessible to the public if there was no effort to keep it in secrecy. o Another’s sale of a product made by a secret process can be a § 102(a) public use if the process

can be determined by examining the product. “In This Country”

o Only knowledge or use in the US can be used in a § 102(a) rejection. “By Others”

o Means any combination of authors or inventors different than the inventive entity. “Patented in This or a Foreign Country”

o Refer to section 2126, above. Publications as § 102(a) Prior Art

o A § 102(a) prima facie case is established if reference publication is the invention (or an obvious variant thereof) “by others” within 1 year of the filing date, unless it is stated within the publication that it is describing the applicant’s work.

o An applicant can rebut a prima facie case by showing that a reference’s disclosure was derived from applicant’s own work.

• An applicant’s own work disclosed within 1 year of filing the application cannot be used against him/her under § 102(a).

o A 37 CFR § 1.131 (swearing-back) rejection can be used to overcome a § 102(a) rejection, but of course it will only be effective as long as the reference is not a statutory bar under § 102(b), (c), or (d).

2133: 35 USC § 102(b)

Rules Regarding the 1-Year Provision o The 1-year grace period under § 102(b) is extended to the next working day if it would otherwise

end on a holiday or weekend. o The 1-year time bar under § 102(b) is measured from the US filing date.

Rejections of CIP Applications o For any claims in a CIP application that are not supported in the parent application, their effective

filing date is the date of the CIP. Rejections Based on Publications and Patents

o An applicant’s own work that was available to the public before the grace period may be used in a § 102(b) rejection.

o A § 102(b) rejection creates a statutory bar to patentability of the rejected claims and, therefore, cannot be overcome by affidavits/declarations, foreign priority dates, or evidence that the applicant himself invented the subject matter.

Rejections Based on “Public Use” o Test for “Public Use:

• The use of even one of the patent articles, which was accessible to the public or commercially exploited, may constitute a statutory bar to patentability.

o Types of Public Use that Constitute Statutory Bar • Public knowledge is not necessarily public use under § 102(b).

Page 107: TOL Patent Bar Outline

Patent Bar Outline © theOtherLives

* Important to know for the patent bar. 107

• Mere public knowledge may constitute a rejection under § 102(a) but not under § 102(b). • “Public use” and “non-secret use” are not necessarily synonymous. • Even if the invention is hidden, an inventor who puts a machine or article embodying the

invention in public view is barred from obtaining a patent as the invention is in public use. • However, there is no public use if the inventor restricted use to locations where there was

a reasonable expectation of privacy and the use was for his or her own enjoyment. • The presence or absence of a confidentiality agreement is not dispositive of the public

use issue, but it is one fact to be considered in case-by-case bases. • An invention is in public use if the inventor allows another to use the invention without

restriction or obligation of secrecy, even if it’s only one person and the article wasn’t necessarily used in public view (such as under a garment).

• Use by an independent third party is public use if it sufficiently informs the public of the invention or a competitor could reasonably ascertain the invention.

Rejections Based on “On Sale” o The Meaning of Sale

• A sale that bars patentability may be for-profit or not-for-profit. • A single sale may bar patentability. • Even a conditional sale – such as a condition on buyer satisfaction – may bar

patentability. • A sale of rights is not a sale of the invention and will not in itself bar a patent. • If the seller has control over the purchaser – such as a parent company and its subsidiary

– the sale will not be a bar to patentability. o Offers for Sale

• A rejected or unreceived offer for sale is enough to bar a patent since patentability can be barred once the offer is made. Likewise, delivery of the offered item is not required.

• It is not necessary that the seller have the item on-hand or show it to the purchaser in order for an offer for sale to bar patentability. However, the material terms of an offer for sale must be present.

o Sale by Inventor, Assignee or Others Associated with the Inventor in the Course of Business

• The activity of the sale does not need to be public. • An on-sale bar may be found even where the inventor’s consent was not given for the

sale. • Objective evidence of a sale or offer to sell is required for an on-sale bar to apply.

o Sales by Independent Third Parties • Sales or offers for sale by independent third parties will bar a patent. • Non-prior art publications can be used as evidence of sale before the critical date.

o The “Invention” • The Invention Must be Ready for Patenting

The invention need not have been reduced to practice in order for the on-sale bar to apply as long as 1) it was the subject of a commercial offer for sale, and 2) the invention was ready for patenting at the time it was offered for sale.

• Sale of a Process A process is not sold in the same way as a tangible invention, such as a

machine. The transfer of the “know how” of a process for which patent is sought under

terms of the sale does not in itself constitute an on-sale bar since the process is not actually performed merely through the sale.

o “In This Country” • § 102(b) applies to sale made in the US. • Generally, the on-sale bar does not apply where both the manufacture and delivery occur

in a foreign country. If one or the other takes place in the US, it will likely constitute on-sale bar.

o Permitted Activity; Experimental Use • If the invention was used publicly or sold primarily for experimentation, purely incidental

commercial exploitation may not constitute a statutory bar to patentability. However, market testing is NOT experimental use that qualifies for this

exception. • The commercial activity must legitimately advance development of the invention towards

completion. • Significant factors indicative of “commercial exploitation” include:

Preparation of various contemporaneous “commercial” documents;

Page 108: TOL Patent Bar Outline

Patent Bar Outline © theOtherLives

* Important to know for the patent bar. 108

Preparation of price lists; Display of samples to prospective customers; Demonstration of models or prototypes; Use of an invention where an admission fee is charged; Advertising in publicity releases, brochures, and various periodicals.

• Experimentation ends when the invention is actually reduced to practice. • Experimental testing is permitted even where the invention is available to the public as

long as the use testing is developmental and the inventor maintains sufficient control over the invention during testing by any third parties involved.

2134: 35 USC § 102(c)

General Rules o Under § 102(c), abandonment must be intentional, whether express or implied. o Where there is no evidence of expressed intent or conduct by inventor to abandon his invention,

delay in reapplying for patent after abandonment of a previous application does not constitute abandonment under § 102(c).

o A mere lapse in time will not bar a patent except where there is a priority contest under § 102(g) and the applicant suppressed, abandoned or concealed the invention.

2135: 35 USC § 102(d)

If the following 4 conditions are present, there will be a bar against patentability in the US under § 102(d): o The foreign application was filed more than 12 months prior to the effective US filing date; o The foreign application was filed by the same applicant as in the US; o The foreign patent or inventor’s certificate must have actually been granted prior to the US filing

date (but it need not be published); and o The same invention must be involved.

Where the anniversary date of the 12-month limit falls on a weekend or Federal holiday, it is extended to the following business day.

A CIP breaks the chain of priority as to foreign, as well as US patents, where the claims in the new application are not supported by the original disclosure.

§ 102(d) applies as of the grant date even if there is a period of secrecy after the granting of the patent. 2136: 35 USC § 102(e)*

Generally o You will undoubtedly encounter several questions regarding § 102(e), and therefore should know

this part of the statute word for word. Additionally, you should dedicate ample study time to ensure that you fully understand the practical applications of § 102(e) provisions, as detailed below.

Status of the US Application as a Reference o When there is no common assignee or inventor, a US application must issue as a patent or be

published as a SIR or as an application publication before it is available as prior art under § 102(e).

o When there is a common assignee or inventor, a provisional § 102(e) rejection over an earlier filed unpublished application can be made.

Content of the Prior Art Available Against the Claims o A § 102(e) rejection may rely on any part of the patent or application publication disclosure. o Reference must itself contain the subject matter relied on in the rejection. o The Supreme Court has authorized § 103 rejections based on § 102(e), in that US patents may

be used as of their filing dates to show that the claimed subject matter is anticipated or obvious. Critical Reference Date

o A reference’s foreign priority date under 35 USC §§ 119(a)-(d) and (f) cannot be used as the § 102(e) reference date since § 102(e) is explicitly limited to certain references “filed in the US before the invention thereof by the applicant.”

o For PCT applications, the international filing date is a US filing date for prior art purposes under § 102(e) if:

• The international application meets the following 3 conditions: The international filing date is on or after November 29, 2000; It designates the US; and Is published in English.

o The § 102(e) critical reference date where the reference claims benefit to a provisional application is the provisional application filing date.

Page 109: TOL Patent Bar Outline

Patent Bar Outline © theOtherLives

* Important to know for the patent bar. 109

o The filing date of a US parent application can only be used as the § 102(e) date if it supports the subject matter relied upon in the child.

Overcoming a Rejection Under § 102(e) o A § 102(e) rejection can be overcome by antedating the filing date or showing that disclosure

relied on is the applicant’s own work. o A § 102(e) rejection can be overcome by showing that the reference is describing applicant’s own

work. o The applicant need not show diligence or reduction to practice when the subject matter disclosed

in the reference is applicant’s own work. o Claiming of individual elements or subcombinations in a combination claim of the reference does

not itself establish that the patentee invented those elements. 2137: 35 USC § 102(f)

§ 102(f) says that a person is entitled to a patent unless he did not invent the subject matter sought to be patented.

Where it can be shown that an applicant “derived” an invention from another, a rejection under § 102(f) is proper.

o Derivation requires complete conception by another and communication to the alleged infringer. o The party alleging derivation does not have to prove an actual reduction to practice, derivation of

public knowledge, or derivation in this country. o § 102(f) may apply where § 102(a) and § 102(e) are not available statutory grounds for rejection.

Inventorship o Executors of an oath or declaration under 37 CFR § 1.63 are presumed to be the inventors. o An inventor must contribute to the conception of the invention. o As long as the inventor maintains intellectual dominion over making the invention, ideas,

suggestions, and materials may be adopted from others. o The inventor is not required to reduce the invention to practice. o The general requirements for joint inventorship apply to § 102(f).

2138: 35 USC § 102(g)

Ex Parte Application o § 102(g) may form the basis for an ex parte rejection if:

• The subject matter at issue has been actually reduced to practice by another before the applicant’s invention; and

• There has been no abandonment, suppression or concealment. Interference Practice

o § 102(g) is the basis of interference practice where priority must be determined between two applicants.

o This is an inter partes proceeding directed at determining who has priority to patent his invention when either two applications are filed for the same invention by different inventors or where one application is filed closely enough in time to an issued patent of the same invention by a different inventor.

• Remember that the US is a first-to-practice country, which means that an inventor may have priority if he was the first to conceive of the invention and diligently reduce it to practice even if someone else was able to patent it first.

• Sufficient evidence must be produced in interference proceedings to prove time of conception and reduction to practice.

“The Invention was Made in This Country” o § 102(g) applies only to inventions that have actually been made – conceived and reduced to

practice. o The invention must be made in the US or a NAFTA or WTO country.

“By Another Who has Not Abandoned, Suppressed or Concealed It” o During an interference proceeding, an inference of suppression or concealment may arise from

delay in filing a patent application. o Any suppression or concealment need not be attributed to the inventor. o Inference of suppression or concealment is rebuttable by showing activity directed to perfecting

the invention, developing additional species with the scope of the genus claim (like compounds) or preparing the application.

“Conception” o Conception is “the complete performance of the mental part of the inventive act.” In other words,

conception occurs when the inventor has thought through the invention well enough to describe it

Page 110: TOL Patent Bar Outline

Patent Bar Outline © theOtherLives

* Important to know for the patent bar. 110

sufficiently that one enabled in the art could reduce it to practice without excessive experimentation or use of inventive skill.

o As long as the inventor maintains intellectual domination over making the invention, ideas, suggestions and materials may be adopted from others.

o Conception requires contemporaneous recognition and appreciation of the invention. “Reduction to Practice”

o Reduction to practice may be actual or constructive. • Actual reduction to practice is the reduction of the concept to the functional form of the

invention. Requirements for actual reduction to practice include: The party constructed an embodiment or performed a process that met every

element of the interference count; and The embodiment or process operated for its intended purpose.

• Constructive reduction to practice consists of conception and filing of the patent application.

o In an interference proceeding, all limitations of a claim must be reduced to practice. o If there is no known utility of the invention, it cannot actually be reduced to practice.

“Reasonable Diligence” o The critical period for diligence for a first conceiver but second reducer begins not at the time of

conception of the first conceiver but just prior to the entry in the field of the party who was first to reduce to practice and continues until the first conceiver reduces to practice.

o The entire period during which diligence is required must be accounted for by either affirmative acts or acceptable excuses.

o Work relied upon to show reasonable diligence must be directly related to the reduction to practice.

o Diligence is also required in preparing and filing the patent application. 2142: Legal Concept of Prima Facie Obviousness – Relevant to § 103 Rejections

The legal concept of prima facie obviousness serves to allocate who has the burden of going forward with production of evidence in each step of the examination process.

Initially, the examiner has the burden of factually supporting a prima facie case of obviousness. If s/he does not do so, the applicant has no obligation to provide evidence of nonobviousness.

Analysis supporting a prima facie case of obviousness must be made explicit. Some examples of rationale for obviousness include:

o Combining prior art elements according to known methods to yield predictable results; o Simple substitution of one known element for another to obtain predictable results; o Use of known technique to improve similar devices in the same way; o Applying a known technique to a known device ready for improvement to yield predictability; o “Obvious to try” – choosing from a finite number of identified, predictable solutions, with a

reasonable expectation of success; o Known work in one field of endeavor may prompt variations of it for use in either the same field or

a different one based on design incentives or other market forces if the variations are predictable to one of ordinary skill in the art;

o Some teaching, suggestion, or motivation in the prior art that would have led one of ordinary skill to modify the prior art reference or to combine prior art reference teachings to arrive at the claimed invention.

Suggestion or Motivation to Modify the Reference o Obviousness can be established by combining or modifying the teachings of the prior art to

produce the claimed invention where there is some teaching, suggestion, or motivation to do so. o Where the teachings of the prior art conflict, the examiner must weigh the suggestive power of

each reference. o The fact that references can be combined or modified may not be sufficient to establish prima

facie obviousness. o A mere statement that the claimed invention is within the capabilities of one of ordinary skill in the

art is not sufficient by itself to establish prima facie obviousness. o If the examiner proposes a modification to the prior art to establish obviousness, it cannot render

the prior art unsatisfactory for its intended purpose. o Such a proposed modification cannot change the principle of operation of a reference.

Reasonable Expectation of Success is Required o Rationale presented by an examiner to support a finding of obviousness must show that all the

claimed elements were known in the prior art and one skilled in the art could have combined the elements as claimed by known methods with no change in their respective functions.

o Obviousness requires only a reasonable expectation of success.

Page 111: TOL Patent Bar Outline

Patent Bar Outline © theOtherLives

* Important to know for the patent bar. 111

o At least some degree of predictability is required. • Applicants may present evidence showing there was no reasonable expectation of

success. o Predictability is determined at the time the invention was made.

2144: Supporting a Rejection Under 35 USC § 103

Rationale may be in a reference, or reasoned from common knowledge in the art, scientific principles, art-recognized equivalents or legal precedent.

The expectation of some advantage is the strongest rationale for combining references. Legal precedent can provide the rationale supporting obviousness only if the facts in the case are

sufficiently similar to those in the application. An examiner may use rationale that is different from the applicant’s to show obviousness. The rationale to support a rejection under § 103 may rely on logic and sound scientific principle. Recall the Graham factors for an examiner to use for determining obviousness:

o Determine the scope and contents of the prior art; o Ascertain the differences between the prior art and the claims in issue; o Determine the level of ordinary skill in the pertinent art; and o Evaluate any evidence of secondary considerations.

2145: Consideration of Applicant’s Rebuttal Arguments

Once the examiner establishes a prima facie case of obviousness, the burden shifts to the applicant to provide a rebuttal.

Rebuttal evidence may include evidence of “secondary considerations,” such as: o Commercial success, o Long-felt but unsolved needs, and o Failure of others.

An applicant may submit evidence and arguments to rebut the finding of obviousness. However, an argument does not replace evidence where evidence is necessary.

Prima facie obviousness is not rebutted by merely recognizing additional advantages or latent properties present in the prior art.

An argument that the objects of two references cannot be physically combined will not rebut an examiner’s rejection because the test is what the combined teachings of those references would have suggested to those of ordinary skill in the art, not whether they can be physically combined.

Other examples of arguments that will not rebut an obviousness rejection include: o Arguing against references individually; o Arguing about the number of references combined; o Arguing limitations that are not claimed; o Arguing economic infeasibility; o Arguing about the age of references; o Arguing that prior art is nonanalogous.

2161: 35 USC § 112, First Paragraph; Three Separate Requirements

35 USC § 112, First Paragraph: “The specification shall contain a written description of the invention, and of the manner and process of making and using it, in such full, clear, concise, and exact terms as to enable any person skilled in the art to which it pertains, or with which it is most nearly connected, to make and use the same, and shall set forth the best mode contemplated by the inventor of carrying out his invention.”

o In summary, the specification must include a written description of the invention, enablement, and best mode of carrying out the claimed invention.

o These three requirements are separate and distinct from each other. Written Description

o The function of the written description requirement is to ensure that the inventor had possession of, as of the filing date, the specific subject matter later claimed by him or her.

o It is not material how the specification accomplishes this. o One of the policy considerations behind the written description requirement is to show that the

claimed inventor is actually the inventor of the invention being described and claimed. o This requirement serves both to satisfy the inventor’s obligation to disclose the technologic

knowledge upon which the patent is based, as well as to demonstrate that the patentee had possession of the invention that was claimed.

Enablement Requirement o The disclosure must adequately enable a person of ordinary skill in the art to make and use the

claimed invention without resorting to undue experimentation.

Page 112: TOL Patent Bar Outline

Patent Bar Outline © theOtherLives

* Important to know for the patent bar. 112

o The Test of Enablement • The standard applied to determine enablement is whether experimentation needed to

practice the invention is undue or unreasonable. • The fact that experimentation may be complex does not necessarily make it undue if the

art typically engages in such experimentation. • Some factors for determining whether experimentation is undue include:

The breadth of the claims; The nature of the invention; The state of the prior art; The level of one of ordinary skill; The level of predictability in the art; The amount of direction provided by the inventor; The existence of working examples; and The quantity of experimentation needed to make or use the invention based on

the content of the disclosure. o As long as the specification discloses at least one method for making and using the claimed

invention that bears a reasonable correlation to the entire scope of the claim, the enablement requirement is satisfied.

o It is not necessary that a working example is disclosed. An applicant may provide a prophetic example, which describes an embodiment of the invention based on predicted results.

Best Mode o The purpose of the best mode requirement is so the inventor cannot obtain a patent – and the

rights associated – while concealing from the public the best way to practice the invention. Patent rights are granted in exchange for the public to obtain the value of the invention.

o The requirement is that the best mode for using the invention as known by the inventor at the time of filing the application being disclosed. This duty ends at the date of filing.

o An examiner will assume that the best mode is presented unless s/he receives evidence to the contrary.

• It is not necessary for the applicant to explicitly state what the best mode is in the disclosure, but it must be included. For example, if the applicant presents more than one mode of use, s/he is not required to state which is best.

o If it is later discovered that the inventor knew of the best mode at the time of filing the application but did not disclose it, the patent will be invalidated.

• Active concealment or grossly inequitable conduct is not required to establish failure to disclose the best mode.

2171: 35 USC § 112; Second Paragraph; Two Separate Requirements

35 USC § 112: “The specification shall conclude with one or more claims particularly pointing out and distinctly claiming the subject matter which the applicant regards as his invention.”

There are two requirements in the Second Paragraph of § 112: o The claims must set forth the subject matter that applicants regard as their invention; and o The claims must particularly point out and distinctly define the metes and bounds of the subject

matter that will be protected by the patent grant. Subject Matter Which Applicants Regard as Their Own

o The invention presented in the claims will be presumed to be that of the applicant/inventor unless the examiner has reason to believe otherwise.

Claims Must Particularly Point Out and Distinctly Claim the Invention o The primary purpose of this provision is to ensure that the scope of the claims is clear so the

public is informed of the boundaries of what constitutes infringement of the patent. o Definite claim language must be analyzed in light of:

• The content of the particular application disclosure; • The teachings of the prior art; and • The claim interpretation that would be given by one possessing the ordinary level of skill

in the pertinent art at the time the invention was made. o The examiner must ensure that what is presented in the disclosure provides a clear warning to

the public of what constitutes infringement. o Some types of indefinite claims:

• A single claim that claims both an apparatus and the method steps of using the apparatus is indefinite under § 112, Second Paragraph.

• An attempt to claim a process without setting forth steps will generally be rejected for indefiniteness.

Page 113: TOL Patent Bar Outline

Patent Bar Outline © theOtherLives

* Important to know for the patent bar. 113

• Omnibus claims – those merely describing the inventions as “a device substantially as shown and described” – will be rejected as improper.

2181: Identifying a 35 USC § 112, Sixth Paragraph Limitation

35 USC § 112, Sixth Paragraph: “An element in a claim for a combination may be expressed as a means or step for performing a specified function without the recital of structure, material, or acts in support thereof, and such claim shall be construed to cover the corresponding structure, material, or acts described in the specification and equivalents thereof.”

A claim limitation will be presumed to invoke § 112, Sixth Paragraph, if it meets the following three-prong analysis:

o The claim limitations must use the phrase “means for” or “step for;” o The “means for” or “step for” must be modified by functional language; and o The phase “means for” or “step for” must not be modified by sufficient structure, material, or acts

for achieving the specified function. The specification need not describe the equivalents of the structures, material or acts corresponding to

the means- (or step-) plus-function element. A means- (or step-) plus function claim limitation satisfies 35 USC § 112, Second Paragraph if:

o The written description links or associates particular structure, materials, or acts to the function recited in a means- (or step-) plus function claim limitation, or

o It is clear based on the facts of the application that one skilled in the art would have known what structure, materials, or acts perform the function recited in a means- (or step-) plus function limitation.

o If means-plus-function language is employed in a claim, the specification must set forth an adequate disclosure showing what that language means.

2183: Making a Prima Facie Case of Equivalence

A prior art element is equivalent to the corresponding element disclosed in the specification. If the examiner finds that a prior art element performs the function specified in the claim, and is not

excluded by any explicit definition provided in the specification for an equivalent, the examiner should infer that the prior art element is an equivalent.

2186: Relationship to the Doctrine of Equivalents

The doctrine of eviqualents arises in the context of an infringement action. If an accused product of process does not literally infringe a patented invention, the accused product or

process may be found to infringe under the doctrine of equivalents. All words in a claim must be considered in judging the patentability of a claim against the prior art. The doctrine of equivalents operates to expand claim coverage beyond the literal scope of claim

language.

Page 114: TOL Patent Bar Outline

Patent Bar Outline © theOtherLives

* Important to know for the patent bar. 114

Page 115: TOL Patent Bar Outline

Patent Bar Outline © theOtherLives

* Important to know for the patent bar. 115

CHAPTER 2200: CITATION OF PRIOR ART AND EX PARTE REEXAMINATION OF PATENTS 2202: Citation of Prior Art

Prior art in the form of patents or printed publications may be cited to the Office for placement into the patent file without payment of a fee and may be made separate from a request for reexamination.

The main purpose for citing prior art is to alert the inventor and examiner that such art exists and should be considered when evaluating the validity of the patent claims.

2203: Persons Who May Cite Prior Art

The patent owner or any member of the public (individuals, as well as corporations or the government) may submit prior art citations of patents or printed publications to the Office.

The citer need not identify him/herself. o If confidence is desired, the citer must at least include an unsigned statement indicating that the

patent owner has been sent a copy of the cited papers, or a duplicate copy should be submitted to the Office.

A patent examiner should not cite materials in a patent file of their own initiative. 2204: Time for Filing Prior Art Citation

Citations may be filed anytime during the enforceability of a patent (length of patent term plus 6 years, under the statute of limitations for bringing an infringement action).

Citations made during the time a patent is under reexamination will not be admitted to the patent file until the reexamination is concluded unless the citation is made: 1) by the patent owner, 2) by an ex parte reexamination requester, 3) by an inter partes reexamination requester, 4) by an ex parte third party requester’s reply under 37 CFR § 1.535, or 5) as an enterable submission pursuant to 37 CFR § 1.948 in an inter partes reexamination proceeding.

2205: Content of Prior Art Citation

The prior art which may be submitted under 35 USC § 301 is limited to “written prior art consisting of patents or printed publications.”

Along with the citation is required an explanation of how the submitter considers the art to be pertinent and applicable to the patent, as well as an explanation of why it is believed that the prior art has a bearing on the patentability of any claim in the patent.

It is preferred that copies and English translations (where required) are included in the citation. Any citation made by a person other than the patent owner must include a statement that a copy of the

citation has been served on the patent owner, or a duplicate copy of the citation must be submitted to the Office for forwarding to the patent holder along with an explanation as to why service was not possible.

All prior art citations submitted should identify the patent in which the citation is to be placed by the patent number, issue date, and patentee.

A prior art citation is limited to the citation of patents and printed publications and an explanation of the pertinence and applicability of the patents and printed publications.

o A prior art citation cannot include a statement as to the claims violating 35 USC § 112, a statement as to the public use of the claimed invention, or a statement as to the conduct of the patent owner.

2207: Entry of Court Decision in Patent File

Copies of notices of suits and other proceedings involving the patent and copies of decisions or other court papers will be accepted at any time.

2209: Ex Parte Reexamination

Any person can file a request for ex parte reexamination. If a substantial new question of patentability is presented, a reexamination will be ordered.

When the prosecution of a reexamination proceeding is terminated, a reexamination certificate is issued, which indicates the status of all claims following the reexamination.

The basic characteristics of an ex parte reexamination include: o Anyone can request reexamination at any time during the period of enforceability of the patent; o Prior art considered during reexamination is limited to prior art patents or printed publications

applied under the appropriate parts of 35 USC §§ 102 and 103; o A substantial new question of patentability must be present for reexamination to be ordered; o If ordered, the actual reexamination proceeding is ex parte in nature; o Decision on the request must be made no later than 3 months from its filing and the remainder of

proceedings must proceed with “special dispatch” within the Office;

Page 116: TOL Patent Bar Outline

Patent Bar Outline © theOtherLives

* Important to know for the patent bar. 116

o If ordered, a reexamination proceeding will normally be conducted to its conclusion and the issuance of a reexamination certificate; and

o All reexamination and patent files are open to the public. 2210: Request for Ex Parte Reexamination

A request for examination includes: o A statement pointing out every new question of patentability; o A detailed explanation and identification of every claim sought; o A copy of every publication or patent referred to; and o A certification that the patent owner has been served with a copy of the request.

Although the fee can be paid later, the reexamination date is the date the complete fee is received o The fees for a reexamination proceeding include the fees for the request, for addition of claims,

for a request for an extension of time, for any appeal, brief and oral hearing. o No fee is due for the issuance of a reexamination certificate.

If a request filed by the patent owner includes a proposed amendment, excess claims fees may also apply.

2212: Persons Who May File a Request for Ex Parte Reexamination

Anyone may request ex parte reexamination. In an ex parte reexamination (as opposed to an inter partes reexamination):

o The requestor may not participate in any way. o The requestor may remain anonymous. o The third party need not serve the applicant.

A third party who initiates an ex parte reexamination can still participate after a substantial new question is confirmed, but the real party of interest must be identified.

2214: Content of Request for Ex Parte Reexamination

Required elements of a request for ex parte reexamination (37 CFR § 1.510(b)): o A statement pointing out each substantial new question of patentability based on prior patents

and printed publications. o An identification of every claim for which reexamination is requested, and a detailed explanation

of the pertinence and manner of applying the cited prior art to every claim for which reexamination is requested. If appropriate, the party requesting reexamination may also point out how claims distinguish over cited prior art.

o A copy of every patent or printed publication relied upon or referred to in paragraph (b)(1) and (2) of this section accompanied by an English language translation of all the necessary and pertinent parts of any non- English language patent or printed publication.

o A copy of the entire patent including the front face, drawings, and specification/claims (in double column format) for which reexamination is requested, and a copy of any disclaimer, certificate of correction, or reexamination certificate issued in the patent. All copies must have each page plainly written on only one side of a sheet of paper.

o A certification that a copy of the request filed by a person other than the patent owner has been served in its entirely on the patent owner at the address as provided for in § 1.33(c). The name and address of the party served must be indicated. If service was not possible, a duplicate copy must be supplied to the Office.

In order to obtain a reexamination request filing date, the entire fee must be submitted in addition to the five required elements listed above.

It is possible to obtain a refund of this fee if the request for reexamination is denied. 2216: Substantial New Question of Patentability

If a substantial new question of patentability based on prior patents and publications is found, a reexamination will be ordered.

If reexamination is ordered, the patent owner will have 2 months to file a statement. Questions relating to grounds of rejection other than those based on prior art patents or printed publi-

cations should not be included in the request and will not be considered by the examiner if included. 2217: Statement in the Request Applying Prior Art

Admissions by the patent owner as to any matter affecting patentability may be utilized to determine the scope and content of the prior art in conjunction with patents and printed publications.

2218: Copies of Prior Art

Page 117: TOL Patent Bar Outline

Patent Bar Outline © theOtherLives

* Important to know for the patent bar. 117

It is required that a legible copy of each patent or printed publication relied on or referred to in the request be filed with the request.

For non-English references, an English translation of all pertinent parts must be submitted. 2221: Amendments Included in Request by Patent Owner

If the patent owner is the requestor, s/he may include a proposed amendment with the request. Excess claims fee may apply.

2223: Withdrawal of Attorney or Agent

A request by an attorney or agent of record to withdraw from a patent will normally be approved only if at least 30 days remain in the period for a response.

2224: Correspondence

All requests for ex parte reexaminations mailed to the PTO should be marked “Mail Stop Ex Parte Reexamination” to distinguish them from inter partes reexamination requests.

A request for reexamination may not be sent by fax. Communications from the PTO to the patent owner will be directed to the first named, most recent

attorney or agent of record in the patent file, or to the patent owner’s address if no attorney or agent is of record.

2225: Untimely Paper Filed Prior to Order

After the filing of a request, no other papers may be submitted except: o Citations of patents or printed publications; o Another complete request; and o Notifications of the existence of prior or concurrent proceedings.

2229: Notice of Request for Ex Parte Reexamination in Official Gazette

Both reexamination requests that have been assigned a filing date and Director-initiated orders to reexamine made without a request will be announced in the Official Gazette.

2230: Constructive Notice to Patent Owner

If all attempts to deliver mail to the patent owner fail, the reexamination will proceed without actual notice to the patent owner.

2234: Entry of Amendments

Reexamination requests are assigned to examiners different from those who examined the patent application.

Although amendments will be entered for the purposes of examination, the amendments are not legally effective until the reexamination certificate is issued.

2236: Assignment of Reexamination

All reexamination files are normally open to inspection by the general public via public PAIR. 2240: Decision on Request

The following types of amendments will not be considered when deciding requests: o Amendments that have been presented with the request if by the patent owner; o Those that have been filed in a pending reexamination proceeding in which the certificate has not

been issued; and o Those that have been submitted in a reissue application on which no reissue patent has been

issued. If a second or subsequent request for ex parte reexamination is filed (by any party) while a first ex parte

reexamination is pending, then the second request must include new prior art that raises a substantial new question of patentability that is different from that raised in the pending first reexamination proceeding.

2241: Time for Deciding Request

The determination of whether or not to reexamine must be made within 3 months from the request’s filing date.

2242: Criteria for Deciding Request

Page 118: TOL Patent Bar Outline

Patent Bar Outline © theOtherLives

* Important to know for the patent bar. 118

If the prior art patents or printed publications raise a substantial question of patentability for at least one claim of the patent, then a substantial new question of patentability is present, unless that question has already been decided.

Where a request for reexamination of a patent is made before the conclusion of an earlier filed reexamination proceeding pending for that patent, the substantial new question of patentability may be raised with respect to any new or amended claim which has been proposed in the pending reexamination proceeding.

2246: Decision Ordering Reexamination

When granting a decision ordering a reexamination, where the question is raised, or where it is not clear that a patent or printed publication pre-dates the patent claims, a discussion should be provided as to why the patent or printed publication is in fact available against the patent claims.

Under some circumstances, a patent owner can petition that the patent not be reexamined, such as when the order for reexamination is not based on prior art patents and printed publications.

Petitioning is available when: o The reexamination order is not based on prior art patents or printed publications; o All claims of the patent were held to be invalid by a final decision of a Federal Court after all

appeals; o Reexamination was ordered for the wrong patent; o Reexamination was ordered based on a duplicate copy of the request; or o The reexamination order is based wholly on the same question of patentability raised by the prior

art previously considered in an earlier concluded examination of the patent by the Office. Petitioning is not available when:

o The examiner determines that the date of a reference is early enough that the reference constitutes prior art; or

o The examiner determines that a reference is a printed publication (i.e. that the criteria for publication have been met).

2247: Decision on Request for Reexamination, Request Denied

The request for reexamination will be denied if a substantial new question of patentability is not found based on patents or printed publications.

2248: Petition from Denial of Request

If a request for reexamination is denied, the requestor may petition for reconsideration within 1 month. The Director will review the petition and render a decision, which is final and non-appealable.

2249: Patent Owner’s Statement

The patent owner has no right to file a statement between the time of filing of the request and the order examination. Any amendments submitted during this period will not be considered.

If the reexamination is ordered, the patent owner will be given at least two months (from the date of owner’s service) to file a statement and any desired narrowing amendments to the patent claims. An extension of time may be granted.

Any statement filed must clearly point out why the patent claims are believed to be patentable, considering the cited prior art patents or printed publications alone or in any reasonable contribution.

Alternatively, the patent owner may file a waiver of statement filing, which may expedite reexamination. 2258: Scope of Ex Parte Reexamination

Rejections made under 35 USC § 102(f) or (g) based on the prior invention of another must be disclosed in a patent or printed publication.

New claims may be added in a reexamination, but not broader claims. A rejection on the prior public use or sale, insufficiency of disclosure, etc., cannot be made even if the

rejection relies on a prior patent or printed publication. Prior art patents or printed publication must be applied under an appropriate portion of 35 USC § 102

and/or §103 when making a rejection. The issue of double patenting is appropriate for consideration in a reexamination proceeding. Admission by the patent owners as to matters affecting patentability may be utilized in a reexamination

proceeding. A third party may not submit admissions that were supposedly made by the patent owner outside of the

record or the court. 2259: Res Judicata and Collateral Estoppel in Reexamination Proceedings

Page 119: TOL Patent Bar Outline

Patent Bar Outline © theOtherLives

* Important to know for the patent bar. 119

Since all claims finally held invalid by a Federal Court, after all estoppels, will be withdrawn from consideration and not reexamined during a reexamination proceeding, a rejection on the grounds of res judicata will not be appropriate in reexamination

2261: Special Status for Action

Reexamination proceedings have “special” status throughout their pendency in the Office. Any cases involved in litigation, whether reexamination or reissue, will have priority over all other cases.

2263: Time for Response

A shortened statutory period of 2 months will be set for responses to Office actions, except where the reexamination results from a court order or litigation, in which case the shortened statutory period will be set at 1 month.

2264: Mailing of Office Action

All actions in a third party requester ex parte reexamination will have a copy mailed to the third party requester.

2265: Extension of Time

The provisions of 37 CFR § 1.136(a) and (b) are not applicable to reexamination proceedings under any circumstances.

In reexaminations, extensions are available for cause only as written in 37 CFR § 1.550. Filing a response in a reexamination automatically extends the deadline by one month. A request for an extension must be filed:

o On or before the day on which action by the patent owner is due; and o It must set forth sufficient reasons for the extension.

Notification of the examiner’s ruling should reach the patent owner with sufficient time for the patent owner to consider the ruling and act on it.

2266: Responses

Where a patent owner’s submission contains a serious deficiency, such as an omission, and the period for response has expired, the patent owner will be notified of the deficiency and be given a time period for the omitted response, which must be provided by the new deadline to avoid termination of the proceedings.

A response by the patent owner will be considered not fully responsive to a non-final Office action where: o A bona fide attempt to respond to an examiner’s non-final action is filed; o before the expiration of the permissible response period; o but through an apparent oversight or inadvertence, some point necessary to a full response has

been omitted. In a reexamination proceeding, the patent owner cannot file an RCE under 37 CFR § 1.114. If a response to the final rejection is filed, the time period set in the final rejection is automatically

extended by 1 month if the response is the first response after the final rejection. o The period for response should be appropriately extended in the examiner’s advisory action if

there is insufficient time for the patent owner to consider the examiner’s ruling and act on it. 2273: Appeal from Ex Parte Reexamination

In an ex parte reexamination of a patent that issued from an original application filed on or after Nov 29, 1999, the patent owner may appeal to the Board only after a claim has been finally rejected.

A patent owner that is dissatisfied with the rejection of his claims may appeal to the Board for review of the rejection by filing a Notice of Appeal within the required time (the period set forth for response in the last Office action, normally 2 months).

A third party may not appeal under 37 CFR §§ 1.310 or 1.303(a). A reexamination under 35 USC § 302 is conducted ex parte after it is instituted. Third parties do not participate in ex parte reexaminations. It does not matter if a third party is dissatisfied with the decision of the Board, they cannot appeal.

2274: Appeal Brief

Where the appeal brief is not filed, but within the period allowed for filing the brief an amendment is presented which places the claims of the patent under reexamination in a patentable condition, the amendment may be entered. Amendments should not be included in the appeal brief.

An appeal brief must be filed within two months from the date of appeal. An applicant may file a petition requesting additional time to file an appeal brief (usually 1 month), and he

must give reasons for the request.

Page 120: TOL Patent Bar Outline

Patent Bar Outline © theOtherLives

* Important to know for the patent bar. 120

Failure to file the brief and/or the appeal fee within the permissible time will result in the dismissal of the appeal. The reexamination will then be terminated and a Notice of Intent to Issue Reexamination Certificate will be issued indicating the status of the claims at the time of appeal.

Where an appeal brief is defective, patent owner has 1 month to cure the defect(s). 2276: Oral Hearing

If patent owner (appellant) desires an oral hearing, appellant must file a written request, along with the fee, within two months (non-extendable) after the date of the examiner’s answer or supplemental examiner’s answer.

2280: Information Material to Patentability in Reexamination Proceeding

The duty of disclosure applies in reexamination proceedings. Any individual substantively involved in the reexamination proceeding may satisfy his or her duty by

disclosing the information to the attorney or agent having responsibility for the reexamination proceeding or to a patent owner acting in his or her own behalf.

2281: Interviews in Ex Parte Reexamination Proceedings

In ex parte proceedings, only ex parte interviews between the examiner and patent owner and/or the patent owner’s representative are permitted.

2283: Multiple Copending Ex Parte Reexamination Proceedings

In order for a second reexamination to be granted while one is still pending, the second request must raise a substantial new question of patentability.

If the second request is granted, the Central Reexamination Unit (CRU) will determine whether the two proceedings should be merged.

Where the merger decision indicates that an Office action will follow, the merged proceeding is returned to the examiner immediately after the decision for appropriate action.

2284: Copending Ex Parte Reexamination and Interference Proceedings

A patent being reexamined in an ex parte reexamination proceeding may be involved in an interference proceeding with at least one application, where the patent and the application are claiming the same pat-entable invention, and at least one of the application’s claims to that invention are patentable to the appli-cant.

The general policy of the Office is that a reexamination proceeding will not be delayed, or stayed, because of an interference or the possibility of an interference.

The reexamination proceeding itself can never be involved in an interference proceeding. 2285: Copending Ex Parte Reexamination and Reissue Proceedings

The general policy of the Office is that a reissue application examination and an ex parte reexamination proceeding will not be conducted separately at the same time as to a particular patent.

o The proceedings will either be merged, or one will be stayed until the other has concluded. In contrast to when a CPA of the reissue application is filed, if an RCE is filed, the reissue application is

not considered to be expressly abandoned and the merged proceeding will continue. 2286: Ex Parte Reexamination and Litigation Proceedings

The final decision by a US District Court finding a patent to be valid will have no binding effect during reexamination since the PTO may still find the claims of the patent to be invalid.

The PTO may discover new art and find the claims unpatentable as that art would raise a substantial new question of patentability.

A patentee could file a prior art statement under 35 USC § 301 or disclose prior art in a reissue application if the original patent (through error and without deceptive intent) is defective or claims more or less than what should be claimed.

The preponderance of evidence standard does not change in reexamination proceedings. 2287: Conclusion of Ex Parte Reexamination Proceeding

Upon conclusion of the ex parte reexamination proceeding: o The examiner will prepare a “Notice of Intent to Issue Ex Parte Reexamination Certificate.” o An examiner’s amendment will be prepared, if appropriate. o Where claims are found patentable, reasons will be given for each. o The examiner must also prepare the reexamination file so that the Office of Publications can

prepare and issue a certificate setting forth the results of the reexamination proceeding and the content of the patent following the proceeding.

Page 121: TOL Patent Bar Outline

Patent Bar Outline © theOtherLives

* Important to know for the patent bar. 121

2288: Issuance of Ex Parte Reexamination Certificate

Since abandonment is not possible in a reexamination proceeding, a reexamination certificate will be issued at the conclusion of the proceeding in each patent in which a reexamination proceeding has been ordered except where the reexamination has been concluded by vacating the reexamination proceeding or by the grant of a reissue patent on the same patent in which case the reissue patent also serves as the reexamination certificate.

The certificate will: o Cancel any patent claims determined to be unpatentable; o Confirm any patent claims determined to be patentable; o Incorporate into the patent any amended or new claims determined to be patentable; o Make any changes in the description approved during reexamination; o Include any statutory disclaimer or terminal disclaimer filed by the patent owner; o Identify unamended claims which were held invalid on final holding by another forum on any

grounds; o Identify any patent claims not reexamined; o Be dated on the day it is mailed to the patent owner at the address provided for in 37 CFR §

1.33(c) and a copy will be mailed to the third party requestor; and o Identify patent claims, dependent on amended claims, determined to be patentable.

Page 122: TOL Patent Bar Outline

Patent Bar Outline © theOtherLives

* Important to know for the patent bar. 122

Page 123: TOL Patent Bar Outline

Patent Bar Outline © theOtherLives

* Important to know for the patent bar. 123

CHAPTER 2500: MAINTENANCE FEES 2504: Patents Subject to Maintenance Fees

Maintenance fees are required to be paid three times throughout the full life of a utility patent issued after Dec 12, 1980.

o Maintenance fees do not apply to plant or design patents. Maintenance fees are not required to be paid on reissued patents if they weren’t required for the originally

issued patent, such as when an original utility patent was issued prior to December 12, 1980 and the reissue came after December 12, 1980.

2506: Times for Submitting Maintenance Fee Payments

Since maintenance fees are allowed to be adjusted every October, an applicant is not allowed to pay these in advance.

Payments may be submitted during the 6 months preceding each due date. Therefore, fees are allowed during:

o 3 to 3.5 years after grant o 7 to 7.5 years after grant o 11 to 11.5 years after grant

A payment made on the last day of this window will be considered timely made. There is a 6-month grace period following each due date, during which time the payment can be made

with a surcharge. If the last day for making a maintenance fee payment falls on a federal holiday or weekend, the payment

may be made on the following non-holiday or Monday. 2510: Submission of Maintenance Fee Payments and Documents

Maintenance fees may be paid by: o Internet (through the PTO’s Electronic Funds Transfer – EFT); o By mail; o By fax (when charged to a deposit account or credit card); or o By hand delivery to the Office of Finance.

2515: Information Required for Submission of Maintenance Fee Payment

Anyone may pay the maintenance fees and surcharges on a patent without proof of authority from the applicant.

A maintenance fee payment must include the patent number and the application number on which the maintenance fee is being paid.

If not all identifying information is submitted, the payment may not be accepted. If the patent expires due to lack of payment of maintenance fee or surcharge, the pantentee may proceed

under 37 CFR § 1.378 (see MPEP 2590) or file a petition under 37 CFR § 1.377 (see MPEP 2580), whichever is appropriate, in order to have the payment accepted as timely even though it lacked identifying information and wasn’t corrected within the grace period.

The following information should also be submitted for each patent on which a maintenance fee or surcharge is paid:

o The fee year; o The amount of the maintenance fee and any surcharge being submitted; o Any assigned customer number; and o Whether small status is being charged or claimed with the payment.

2520: Maintenance Fee Amounts

The following fees are due at the stated times and in the stated amounts: o 3.5 years after grant: $900. o 7.5 years after grant: $2,300 o 11.5 years after grant: $3,800

There is a 6-month grace period following the due dates listed above. During this period, the applicant can make the maintenance payment – with a surcharge – without the patent expiring.

If the maintenance payment is not made within the grace period, the patent will expire. 2522: Methods of Payment

A maintenance fee may be paid: o With cash; o With treasury notes; o With national bank notes;

Page 124: TOL Patent Bar Outline

Patent Bar Outline © theOtherLives

* Important to know for the patent bar. 124

o With post office money orders; o With certified checks; or o Using the PTO’s Electronic Funds Transfer system over the internet.

2531: Payment Late or Insufficient*

The payment will not be accepted if: o It is less than the required amount due; o It is paid after the expiration of the patent (after the grace period); o It is made in a manner other than those listed; or o The deposit account authorized to make the payment lacks sufficient funds to cover the costs

due. If the Office considers a payment to be late or insufficient, a Notice of Non-Acceptance of Patent

Maintenance Fee will be sent to the fee submitter (not necessarily the applicant). o Response to this notice is required prior to the expiration of the grace period in order to keep the

patent from expiring. o If the response is not made in time, then applicant must submit the appropriate petition in order to

get the patent reinstated (see below). 2550: Small Entity Status

If small entity status has been established, the applicant will receive a 50% discount on maintenance fees.

2575: Notices

The Office has no duty to notify the patentee when their maintenance fees are due. Therefore, the pantentee is entirely responsible for ensuring his fees are submitted on time.

If the maintenance fee is not paid within the grace period, the Office will mail a Notice of Patent Expiration to the fee address on file.

2580: Review of Decision Refusing to Accept and Record Payment of a Maintenance Fee Filed Prior to Expiration of Patent

37 CFR § 1.377 provides a mechanism for review of a decision refusing to accept and record payment of a maintenance fee filed prior to the expiration of a patent.

o A petition may be filed with the Director requesting that the fee be accepted. o A petition under this section must be filed within two months of the offending action.

A petition under this section is generally used when there was an error in the identifying information submitted with the maintenance fee.

A petition under this section would not be appropriate where the payer altogether failed to include required information with the payment.

2590: Acceptance of Delayed Payment of Maintenance Fee in Expired Patent to Reinstate Patent*

The Director may accept the payment of any maintenance fee due on a patent after the expiration of the patent if, upon petition, the delay in payment of the maintenance fee is shown to the satisfaction of the Commissioner to have been:

o Unavoidable or unintentional; and o If the surcharge required is paid as a condition of accepting payment of the maintenance fee.

Requirements to petition unavoidable delay: o Petitions to accept an unavoidably delayed payment of maintenance fees must include:

• The required maintenance fee; • The required surcharge; and • A showing that the delay was unavoidable by listing the:

Steps taken to ensure the timely payment of the maintenance fee; Date and the manner in which the patentee became aware of the expiration of

the patent; and Steps taken to file the petition promptly.

• The patentee’s lack of knowledge of the need to pay the maintenance fee and/or the failure to receive the maintenance fee reminder will never count as unavoidable delay.

o Requirements to petition unintentional delay: • Petitions to accept an unintentionally delayed payment of a maintenance fee must:

Be filed within 24 months after the 6 month grace period; Including the maintenance fee; Include the surcharge; and Include a statement that the delay in payment was unintentional.

Page 125: TOL Patent Bar Outline

Patent Bar Outline © theOtherLives

* Important to know for the patent bar. 125

There will be a lapse of time between a patent’s expiration (due to nonpayment of the maintenance fee) and when the petition to reinstate the patent is accepted. Patent rights are lost during this time.

Page 126: TOL Patent Bar Outline

Patent Bar Outline © theOtherLives

* Important to know for the patent bar. 126

Page 127: TOL Patent Bar Outline

Patent Bar Outline © theOtherLives

* Important to know for the patent bar. 127

CHAPTER 2600: OPTIONAL INTER PARTES REEXAMINATION (IPR) 2601: Introduction

The optional inter partes alternative to reexamination provides third parties with a greater opportunity to participate in reexamination proceedings while still maintaining the benefits of a reexamination over litigating in court.

The inter partes option (as opposed to ex partes) also provides the third party with appeal rights, as well as allowance to participate in the patent owner’s appeal to the Board.

For any inter partes reexamination (IPR) proceeding commenced on or after November 2, 2002, the third party requester also has the appeal rights to appeal to the Court of Appeals for the Federal Circuit and to participate in the patent owner’s appeal to the Federal Circuit.

By electing the optional inter parties reexamination process, the requestor must accept a statutory estoppel against subsequent review, either by the Office or by a Federal Court, of the issues that were or could have been raised in the reexamination proceeding. This limitation is intended to prevent third parties from using this form of reexamination to harass patent owners.

2602: Citation of Prior Art

Where an IPR proceeding is pending when a prior art citation is filed, the prior art will be considered when:

o It is cited during the period of enforceability of the patent; o The citation includes an explanation of the pertinence of the prior art; and o It is serviced to the patent owner (or sent in duplicate to the PTO).

2609: Inter Partes Reexamination (IPR)

Any third party requestor may request an IPR of a patent provided that: o The patent issued from an original application filed on or after Nov 29, 1999; o The fee is paid; and o A substantial new question of patentability is presented.

A third party requester may participate throughout the proceeding. There is an action closing prosecution (ACP) and a right of appeal notice (RAN) rather than a final

rejection. An IPR certificate is used to indicate the status of all claims following the reexamination. A decision on the request must be made no longer than three months from its filing date, and the

remainder of the proceedings must proceed with “special dispatch” within the PTO. The scope of the patent claims cannot be enlarged by amendment. Reexamination and patent files are open to the public as an electronic file. When the prosecution of an IPR proceeding is termination, an IPR certificate is issued and the

reexamination proceeding is concluded. 2611: Time for Requesting Inter Partes Reexamination (IPR)

An IPR may be filed during the enforceability of a patent (lifetime of patent plus 6 years following expiration).

If litigation is instituted within the period of the statute of limitations, requests for inter partes reexamination may be filed after the statute of limitations has expired, as long as the patent is still enforceable against someone.

2612: Persons Who May File a Request

Third parties may initiate IPRs. Parties that are likely to use IPR include:

o Licensees o Potential licensees o Infringers o Potential exporters o Patent litigants o Interference applicants o International Trade Commission respondents

For inter partes proceedings, the real name of the requestor will not be kept in confidence. 2614: Content of Request for IPR

The request should identify the patent by stating the patent number. Patents and printed publications that provide a substantial new question of patentability must be listed in

the reexamination request.

Page 128: TOL Patent Bar Outline

Patent Bar Outline © theOtherLives

* Important to know for the patent bar. 128

A copy of each cited patent or printed publication as well as a translation of each non-English document is required.

o A copy of any prior art disclaimer, certificate of correction, or reexamination certificate issued for the patent should also be included with the request.

The requester must include a certification that a copy of the reexamination request has been served on the patent owner, as well as a certification that the requester is not precluded from filing the request for reexamination.

2615: Fee for Requesting IPR

The entire fee required under 37 CFR § 1.915 must be paid before the IPR request will be accepted. If a request for IPR is denied, the fee will be refunded to the requester.

2616: Substantial New Question of Patentability

A request for IPR must include a statement pointing out a substantial new question of patentability regarding at least one claim in the patent at issue based on cited patents and publications.

Questions relating to grounds of rejection other than those based on prior art patents or printed publi-cations should not be included in the request and will not be considered by the examiner if included (questions as to on sale, fraud, etc.).

2617: Statement in the Request Applying Prior Art

The prior art cited may only consist of prior art patents or printed publications. The requester must present an explanation of how the cited patents or printed publications are applied to

all claims that the requester considers to merit reexamination based on patents or printed publications. Admissions by the patent owner as to any matter affecting patentability may be utilized to determine the

scope and content of the prior art in conjunction with patents and printed publications. o An admission per se may not be basis of a substantial new question of patentability. o Any admission submitted by the patent owner is proper. A third party, however, may not submit

admissions of the patent owner made outside the record of the file or a court record. Such a submission would be outside the scope of reexamination.

Only patents and printed publications that were available as of the filing date of the patent at issue may be cited as prior art.

o If the patent is the result of a continuing or other type of offspring application, the prior art date is the filing date of the continuing or offspring application, not the parent.

2624: Correspondence

A request for IPR may not be sent by fax. Any letters sent to the Office relating to the reexamination proceeding should identify:

o The proceeding by number of the patent undergoing reexamination; o The reexamination request control number assigned and name of the examiner; and o The examiner’s Art Unit.

2627: Incomplete Request for Inter Partes Reexamination

Request papers that fail to satisfy all the requirements of 37 CFR § 1.915 are incomplete and will not be granted a filing date.

2640: Decision on Request

Where a request for reexamination is filed on a patent after a reissue for that patent has already issued, reexamination will be denied because the patent on which the request for reexamination is based has been surrendered.

o A new request for reexamination, including and based on the specification and claims of the reissued patent, must be filed.

The second or subsequent request for reexamination may raise a substantial new question of patentability with respect to any new or amended claim that was proposed in the first (or prior) pending reexamination proceeding.

2641: Time for Deciding a Request

The determination of whether to reexamine must be made within 3 months of the filing date of the request (or the preceding business day if on a weekend or holiday).

2648: Petition from Denial of Request

If a requester is not satisfied with the denial of a request for IPR, they may seek review of a petition to the Director under 37 CFR § 1.181 (Petition to the Director).

Page 129: TOL Patent Bar Outline

Patent Bar Outline © theOtherLives

* Important to know for the patent bar. 129

2654: Conduct of IPR Proceedings

Once IPR is ordered, a first Office Action on the merits will be given. Each time the patent holder responds to an Office Action, the third party requester may respond.

o Both the patent owner and the requestor will be sent copies of the Office Actions. Lack of response from the patent owner will not delay reexamination.

2656: Prior Art Patents and Printed Publications Reviewed by Examiner in Reexamination

In addition to patents and printed publications cited by the requester in an IPR, the examiner must also consider patents and publications:

o Cited by another reexamination requester; o Cited by the patent owner under a duty of disclosure; o Discovered by the examiner in searching; o Of record in the patent file from earlier examination; and o Of record in the patent file from any 37 CFR § 1.501 (Citation of Prior Art in Patent Files)

submission prior to date of an order if it complies with 37 CFR § 1.98 (Content of Information Disclosure Statement).

2658: Scope of IPR

Inter partes reexamination (IPR) differs from ex partes reexamination (EPR) in the following ways: o In IPR, the requestor must be identified; o In IPR, the requestor may participate in the proceedings; and o In IPR, there is an estoppel effect that prevents the requester from raising issues not raised in the

IPR in future proceedings. However, the substance of the proceedings is the same for IPR and EPR. In any case, the scope of the claims cannot be enlarged through reexamination proceedings.

2660: First Office Action

The first Office action on the merits will ordinarily be mailed together with the order granting reexamination.

Where all the patent claims are found patentable in the first action, the examiner will issue an Action Closing Prosecution (ACP).

Where the examiner finds that one or more of the patent claims should be rejected, a rejection will be made.

2661: Special Status for Action

Any cases involved in litigation, whether they are reexamination proceedings or reissue applications, will have priority over all other cases.

Reexamination proceedings not involved in litigation will have priority over all other cases except for reexaminations or reissues involved in litigation.

2665: Extension of Time for Patent Owner Response

An extension of time in an IPR is possible under 37 CFR § 1.956 for sufficient cause and for a reasonable time specified.

2666: Patent Owner Response to Office Action

Where a third party requester does not timely file written comments on a patent owner response, any subsequent submission of comments on that response will be refused consideration. The third party requester does not, however, lose any rights as to commenting on future patent owner responses.

Where the third party requester fails to make a timely appeal or the third party requester’s appeal is dismissed, the third party requester loses further rights as the appellant in the appeal.

The patent owner and/or the third party requester will be given a time period of 30-days or one month from the mailing date of the notice of defective paper or the time remaining in the response/comments period set in the last Office action, whichever is longest, to correct the defect in a submission.

If the patent owner amends the patent to add claims during a reexamination and fails to pay an excess claims fee (if owed), the third party will only be able to comment on the new claims within 30 days of the date if/when the fee is paid by the patent owner.

If the failure to comply with the notice of defective paper results in a patent owner’s failure to file a timely and appropriate response to any Office action, the prosecution of the reexamination proceeding will be terminated.

Page 130: TOL Patent Bar Outline

Patent Bar Outline © theOtherLives

* Important to know for the patent bar. 130

2671: Examiner Action Following Response/Comments or Expiration of Time for Same After the response by the patent owner and any third party comments, the patent under reexamination will

be reconsidered. Although the ACP has may attributes similar to a “final rejection”, it is not a final action, and it cannot be

appealed from. An appeal can only be taken after an examiner issues a Right of Appeal Notice (RAN; a final office action,

which presents a final decision to reject the claims). o An amendment filed after a RAN will not be entered.

A reexamination proceeding may result in the final cancelation of claims from the patent and the patent owner does not have the right to continue the proceeding by refiling under 37 CFR § 1.53(b) or 1.53(d) nor by filing an RCE.

The examiner will hold a “patentability review conference” just before issuance of the ACP and just before issuance of the RAN.

2673: Examiner Consideration of Submissions After ACP and Further Action

No amendment other than canceling claims, where such cancelation does not affect the scope of any other pending claims in the proceeding, can be made in an IPR proceeding after the RAN except as provided in 37 CFR § 1.981 (Reopening After a Final Decision by the BPAI) or as permitted by 37 CFR § 41.77(b)(1). (Decisions and Other Actions by the Board).

When the case is taken up for action, the patent owner is given 30 days (or one month, if longer) to make a 37 CFR § 1.951(a) (Options after Office ACP in IPR) submission after ACP. If filed, the third party requestor will have 30 from the date of notice of the § 1.951(a) filing to make a § 1.951(b) submission.

o Where both the 37 CFR 1.951(a) and (b) submissions have been received, the case should be taken up for action as soon as possible.

2674: Appeal in Reexamination

An appeal by either party can only be taken after the RAN has been issued. o Remember that no amendments are allowed after the RAN has issued.

Both parties will have a one-month period in which to file an appeal once the RAN has issued. If neither party appeals, the reexamination is terminated and a certificate will issue. If one party appeals, the other party has 14 days to file a cross appeal. Appeal Procedure:

o Appeal brief: Due two months from notice of appeal o Respondent’s brief: Due one month from service of appeal brief o Examiner’s answer: No new grounds for rejection (or reopen prosecution) o Rebuttal brief: Due one month from examiner’s answer o Request for oral hearing: Due 2 months from examiner’s answer o Oral argument: 20 minutes for examiner and 30 minutes for each party o Decision by board o Petition for rehearing: Due one month from initial decision o Appeal to federal circuit by patent owner

2687: Notice of Intent to Issue IPR Certificate and Conclusion of Reexamination Proceedings

Upon conclusion of the IPR proceeding, the examiner must complete a Notice of Intent to Issue IPR Certificate (NIRC).

An NIRC is proper: o When there is not timely response by the patent owner to an Office action requiring a response.

Generally this is when all claims are under rejection and all claims will be canceled. o After a RAN where no party to the reexamination files a notice of appeal. o After the filing of a notice of appeal, where all parties who filed a notice of appeal fail to timely file

an appellant brief. o After a final decision by the Board, where there is no further timely appeal to the Board, where

there is no further timely appeal to the Court of Appeals or for a rehearing by the board. o After the Federal Court appeal process has been completed and the case is returned to the

examiner.

Page 131: TOL Patent Bar Outline

Patent Bar Outline © theOtherLives

* Important to know for the patent bar. 131

CHAPTER 2700: PATENT TERMS AND EXTENSIONS 2701: Patent Term

For a utility or plant patent, grant shall be for a term beginning on the date on which the patent issues and ending 20 years from the date on which the application for the patent was filed in the US or, if the application contains a specific reference to an earlier filed application or applications under § 120, 121, or 365(c), from the date on which the earliest such application was filed.

Design patents have a term of 14 years from the date of the patent grant. Patents in force on June 8, 1995, or that issued from an application that was filed before June 8, 1995,

have a term that is the greater of the 20-year term or 17 years from the patent grant. Continuing Applications: A patent granted on a continuation, divisional, or continuation-in-part application

that was filed on or after June 8, 1995, will have a term that ends 20 years from the filing date of earliest application for which a benefit is claimed.

International Applications: A patent granted on an international application filed on or after June 8, 1995 and which enters the national stage will have a term that ends 20 years from the filing date of the international application. A continuation or a continuation-in-part application claiming benefit under 35 USC § 365(c) of an international application filed under 35 USC § 363 designating the United States will have a term which ends twenty years from the filing date of the parent international application.

Foreign priority: An application claiming foreign priority has a term that ends 20 years from the filing date of the application in the United States and not the prior international application. Priority under § 119, 365(a) or 365(b) is not taken into account when determining the term of a patent.

2710: Term Extensions or Adjustments for Delays Within the USPTO Under 35 USC § 154

Utility and plant patents issuing on applications filed on or after June 8, 1995, but before May 29, 2000, are eligible for the patent term extensions.

Utility and plant patents issuing on applications filed on or after May 29, 2000 are eligible for the patent term adjustments.

Plant and utility patents issuing on applications filed before June 8, 1995 which have a term that is the greater of the “twenty-year term” or seventeen years from patent grant are not eligible for term extension or adjustment due to delays in processing the patent application by the United States Patent and Trademark Office.

There are no patent term adjustment provisions for design patents. 2720: Applications Filed Between June 8, 1995 and May 28, 2000

The 20-year term for patents issued between June 8, 1995 and May 28, 2000 may be extended for a maximum of 5 years for delays in the issuance of the patent.

Delays must be due to: o Interferences o Secrecy orders o Appeals

If the patent issues with a different patent term extension value than that indicated on the Notice of Allowance or Office computer records, the patentee may seek correction of the information by filing a request for a Certificate of Correction.

If the applicant disagrees with the PTO’s determination of whether to extend the patent, he may request review by petitioning under 37 CFR § 1.181.

2730: Applications Filed On or After May 29, 2000; Grounds for Adjustment

37 CFR § 1.702 governs the bases for patent term adjustments: o 1.702(a): A patent is entitled to Patent Term Adjustments (PTA) if the Office fails to perform

certain acts of examination within specified time periods. o 1.702(b):

• A patent is entitled to PTA if the Office fails to issue a patent within 3 years of the actual filing date of the application.

• For international applications, the actual filing date is the date that national stage commences.

o 1.702(c): A patent is entitled to PTA if the patent is delayed by an interference. o 1.702(d): A patent is entitled to PTA if the patent is delayed by a secrecy order. o 1.702(e): A patent is entitled to PTA if the patent is delayed by an appeal.

No submission or petition on behalf of a third party concerning a PTA under 35 USC § 1.54(b) (Parts of Application to be Filed Together; Filing Receipt) will be considered by the Office.

Page 132: TOL Patent Bar Outline

Patent Bar Outline © theOtherLives

* Important to know for the patent bar. 132

the2750: Patent Term Extensions for Delays at Other Agencies Under 35 USC § 156 The right to a patent term extension may be based on regulatory agency review (such as FDA approval). 35 USC § 156 was designed to create new incentives for the research and development of certain

products subject to pre-market government approval by a regulatory agency by allowing additional time to the term of a patent that was delayed while awaiting such approval.

An application for extension of a patent term under 35 USC § 156 must be submitted by the recorded owner or its agent within the 60 day period beginning on the date the product received permission for commercial marketing or use under the provision of law under which the applicable regulatory review period occurred for commercial marketing or use.

2751: Eligibility Requirements

There are three laws that discuss the details of patent term extensions for delays at other agencies: o 35 USC § 156, which describes exactly what types of patents may be extended and the extension

conditions; o 37 CFR § 1.710, which describes the patents that may be extended; and o 37 CFR § 1.720, which describes the conditions for the extension.

Generally, patents claiming the following types of inventions may qualify for an extension: o A human drug product o Medical devices o Food or color additive first approved for marketing or use after Sept 24, 1984 o Animal drug o Veterinary biological product first approved for marketing or use after Nov 16, 1988.

2754: Filing Date

The filing date for an application for patent term extension is the date the complete application is received in the USPTO or filed pursuant to certificate of mailing provisions or Express Mail provisions.

A request for patent term extension cannot be sent by fax. If the application meets the requirements of 37 CFR § 1.741, the application filing date will be established

even if the application is held to be informal under 37 CFR § 1.740.